ChaseDream
标题: 【每日逻辑练习第二季】【1-6】 [打印本页]
作者: corneliaflower 时间: 2011-11-24 22:29
标题: 【每日逻辑练习第二季】【1-6】
接过daisy的班,花儿也成为小分队发帖一员~
今天是thanks giving day,要感恩CD,感恩逻辑小分队,感恩XDJM们,更要感恩努力过付出着驶向成功的自己。
跟帖做题方法请见:
http://forum.chasedream.com/GMAT_CR/thread-613902-1-1.html
【精练】
1. Human beings have cognitive faculties that are
superior to those of other animals, and once humans
become aware of these, they cannot be made happy
by anything that does not involve gratification of
these faculties.
Which one of the following statements, if true, most
calls into question the view above?
———Bible Weaken Question Problem Set NO.1
(A) Certain animals—dolphins and chimpanzees,
for example—appear to be capable of
rational communication.
(B) Many people familiar both with intellectual
stimulation and with physical pleasures
enjoy the latter more.
(C) Someone who never experienced classical
music as a child will usually prefer popular
music as an adult.
(D) Many people who are serious athletes consider
themselves to be happy.
(E) Many people who are serious athletes love
gourmet food.
【逻辑链】
21.(28551-!-item-!-188;#058&003690)
Traces of cultivated emmer wheat have been found among the earliest agricultural remains of many archaeological sites in Europe and Asia.The only place where the wild form of emmer wheat has been found growing is a relatively narrow strip of southwest Asia.Since the oldest remains of cultivated emmer wheat yet found are from village sites in the same narrow strip, it is clear that emmer wheat was first domesticated somewhere in that strip.
Which of the following, if true, most strengthens the argument?
(A) The presentday distribution of another wild wheat, einkorn, which was also domesticated early in the development of agriculture, covers a much larger area of southwest Asia.
(B) Modern experiments show that wild emmer wheat can easily be domesticated so as to yield nearly as well as traditionally domestic strains.
(C) At the time when emmer wheat was first cultivated, it was the most nutritious of all the varieties of grain that were then cultivated.
(D) In the region containing the strip where wild emmer wheat has been found, climatic conditions have changed very little since before the development of agriculture.
(E) It is very difficult, without genetic testing, to differentiate the wild form of emmer wheat from a closely related wild wheat that also grows in southwest Asia.
22.(28829-!-item-!-188;#058&003748)
In the late 1980s, the population of sea otters in the North Pacific began to decline.There are two plausible explanations for the decline:predation, possibly by killer whales, or disease.Of these two, disease is the more likely, since a concurrent sharp decline in populations of seals and sea lions is believed to have been caused by disease, and diseases that infect these creatures are likely to be able to infect sea otters also.
Which of the following, if true, most seriously weakens the reasoning?
(A) Killer whales in the North Pacific usually prey on seals and sea lions but will, when this food source is scarce, seek out other prey.
(B) There is no indication that the sea otter population at any North Pacific location declined in the 1980s because of substantial numbers of sea otters migrating to other locations.
(C) Along the Pacific coast of North America in the 1980s, sea otters were absent from many locations where they had been relatively common in former times.
(D) Following the decline in the population of the sea otters, there was an increase in the population of sea urchins, which are sea otters' main food source.
(E) The North Pacific populations of seals and sea lions cover a wider geographic area than does the population of sea otters.
23.(29107-!-item-!-188;#058&003854)
From 1978 to 1988, beverage containers accounted for a steadily decreasing percentage of the total weight of household garbage in the United States.The increasingly widespread practice of recycling aluminum and glass was responsible for most of this decline.However, although aluminum recycling was more widely practiced in this period than glass recycling, it was found that the weight of glass bottles in household garbage declined by a greater percentage than the weight of aluminum cans.
Which of the following, if true of the United States in the period 1978 to 1988, most helps to account for the finding?
(A) Glass bottles are significantly heavier than aluminum cans of comparable size.
(B) Recycled aluminum cans were almost all beverage containers, but a significant fraction of the recycled glass bottles had contained products other than beverages.
(C) Manufacturers replaced many glass bottles, but few aluminum cans, with plastic containers.
(D) The total weight of glass bottles purchased by households increased at a slightly faster rate than the total weight of aluminum cans.
(E) In many areas, glass bottles had to be sorted by color of the glass before being recycled, whereas aluminum cans required no sorting.
24.(29155-!-item-!-188;#058&003857)
Tiger beetles are such fast runners that they can capture virtually any nonflying insect.However, when running toward an insect, the beetles intermittently stop, and then, a moment later, resume their attack.Perhaps they cannot maintain their pace and must pause for a moment's rest; but an alternative hypothesis is that while running tiger beetles are unable to process the resulting rapidly changing visual information, and so quickly go blind and stop.
Which of the following, if discovered in experiments using artificially moved prey insects, would support one of the two hypotheses and undermine the other?
(A) When a prey insect is moved directly toward a beetle that has been chasing it, the beetle immediately turns and runs away without its usual intermittent stopping.
(B) In pursuing a moving insect, the beetles usually respond immediately to changes in the insect's direction, and pause equally frequently whether the chase is up or down an incline.
(C) The beetles maintain a fixed time interval between pauses, although when an insect that had been stationary begins to flee, the beetle increases its speed after its next pause.
(D) If, when a beetle pauses, it has not gained on the insect it is pursuing, the beetle generally ends its pursuit.
(E) When an obstacle is suddenly introduced just in front of running beetles, the beetles sometimes stop immediately, but they never respond by running around the barrier.
作者: corneliaflower 时间: 2011-11-24 22:29
逻辑链KEY DACC
精练解释
Question #1. Weaken. June 1999 LSAT, Section #2, #15. The correct answer choice is (B)
This is a nice straightforward question to start the problem set. The conclusion of the argument appears at
the end of the stimulus: human beings “cannot be made happy by anything that does not involve
gratification of these [cognitive] faculties.” To weaken the argument we must show that individuals can be
made happy without gratification of the cognitive faculties. If you do not know the meaning of
“cognitive,” the problem can be challenging. Cognitive means “relating to the mental process of knowing,
including reasoning and judgment.” In other words, cognitive faculties are thinking and analyzing, etc.
Answer choice (A): This answer attempts to attack the first premise, but fails. Although it is fantastic news
that dolphins and chimps can rationally communicate, this fact has no impact on the argument at hand.
Even though they have this communication ability, human cognitive faculties can still be superior.
Answer choice (B): This is the correct answer, and a somewhat risqué one at that. By showing that many
people enjoy the physical more than the cognitive, the answer shows that people can be made happy by
gratification of something other than cognitive faculties. Cognitive faculties, being mental in nature, are of
course distinct from physical pleasures.
Additionally, this answer has the benefit of addressing the phrase in the stimulus regarding awareness of
cognitive faculties: “once humans become aware of these...” In this answer, unlike others, the individuals
are known to be familiar with cognitive faculties. While we believe that recognition of cognitive faculties
is inherent in adults (or some of the named types in other answers, such as serious athletes, who by
definition would have to be teens or adults), this answer is stronger because it explicitly addresses the
issue.
Answer choice (C): A preference for a certain type of music is likely a cognition-driven preference, and
this preference is expressed by an adult who would certainly be aware of cognitive faculties. And, since no
suggestion is made that individuals can be made happy without gratification of the cognitive faculties, this
answer is incorrect.
Answer choice (D): This can be an attractive answer at first, but it depends on the assumption that the
serious athletes are happy due to their athletic endeavors. However, that connection is not explicitly stated,
and it could be that the serious athletes are happy because of some gratification of their cognitive faculties,
in their respective sport or otherwise.
Answer choice (E): This answer is similar to answer choice (D). A gourmet is a connoisseur of food and
drink, and a connoisseur is a person with deep or special knowledge of a subject. In this sense, there
would be a cognitive element to the enjoyment of gourmet food. As such, this answer may serve to slightly
strengthen the argument because it shows that an individual with experience with the non-cognitive still
retains a love of the cognitive.
作者: xeyyxzty 时间: 2011-11-24 22:42
1.background information:Human beings have cognitive faculties that are superior to those of other animalspremise:once humans become aware of these
conclusion:they cannot be made happy by anything that does not involve gratification of these faculties
is there a link between happiness and cognitive faculties?
(A) Certain animals—dolphins and chimpanzees, for example—appear to be capable of rational communication.
--the premise and conclusion are all about humans--irrelevant
(B) Many people familiar both with intellectual stimulation and with physical pleasures enjoy the latter more.
--weaken--right
(C) Someone who never experienced classical music as a child will usually prefer popular music as an adult.
--irrelevant
(D) Many people who are serious athletes consider themselves to be happy.
--support--if the athletes are happy because of the love of sports they play~right
(E) Many people who are serious athletes love gourmet food.
--???--slightly strengthen
做对了~可是思路又错了!!!
To weaken the argument we must show that individuals can be made happy without gratification of the cognitive faculties.
Cognitive means “relating to the mental process of knowing, including reasoning and judgment.”
B:Additionally, this answer has the benefit of addressing the phrase in the stimulus regarding awareness of cognitive faculties: “once humans become aware of these...” In this answer, unlike others, the individuals are known to be familiar with cognitive faculties. While we believe that recognition of cognitive faculties is inherent in adults (or some of the named types in other answers, such as serious athletes, who by definition would have to be teens or adults), this answer is stronger because it explicitly addresses the issue.
这道题~还是需要多看几遍呀
2.background information:Traces of cultivated emmer wheat have been found among the earliest agricultural remains of many archaeological sites in Europe and Asia.The only place where the wild form of emmer wheat has been found growing is a relatively narrow strip of southwest Asia.
premise:the oldest remains of cultivated emmer wheat yet found are from village sites in the same narrow strip,conclusion:emmer wheat was first domesticated somewhere in that strip.is there a link between the location of the wild form of emmer and the place first domesticated it???what about other continents?(A) The present day distribution of another wild wheat, einkorn, which was also domesticated early in the development of agriculture, covers a much larger area of southwest Asia.--another wild wheat--irrelevant
(B) Modern experiments show that wild emmer wheat can easily be domesticated so as to yield nearly as well as traditionally domestic strains.--the difficulty --irrelevant
(C) At the time when emmer wheat was first cultivated, it was the most nutritious of all the varieties of grain that were then cultivated.--the nutrition--irrelevant
(D) In the region containing the strip where wild emmer wheat has been found, climatic conditions have changed very little since before the development of agriculture.--since before什么意思?--right?排除法
(E) It is very difficult, without genetic testing, to differentiate the wild form of emmer wheat from a closely related wild wheat that also grows in southwest Asia.
--weaken思路不对~选项不懂![](/static/legacy-emoticon/5.gif)
3.background information:In the late 1980s, the population of sea otters in the North Pacific began to decline.There are two plausible explanations for the decline: predation, possibly by killer whales, or disease.premise: a concurrent sharp decline in populations of seals and sea lions is believed to have been caused by disease, and diseases that infect these creatures are likely to be able to infect sea otters also.
conclusion:Of these two, disease is the more likely
what about killer whales? it doesn't mention it~~~
"are likely to be able to"~and is the disease fatal to sea otters?
(A) Killer whales in the North Pacific usually prey on seals and sea lions but will, when this food source is scarce, seek out other prey.
--seek sea otters--right
(B) There is no indication that the sea otter population at any North Pacific location declined in the 1980s because of substantial numbers of sea otters migrating to other locations.
--slightly support or irrelevant
(C) Along the Pacific coast of North America in the 1980s, sea otters were absent from many locations where they had been relatively common in former times.
--it isn't an explanation--irrelevant
(D) Following the decline in the population of the sea otters, there was an increase in the population of sea urchins, which are sea otters' main food source.
--it must happened--irrelevant
(E) The North Pacific populations of seals and sea lions cover a wider geographic area than does the population of sea otters.
--irrelevant
4.background information:From 1978 to 1988, beverage containers accounted for a steadily decreasing percentage of the total weight of household garbage in the United States.The increasingly widespread practice of recycling aluminum and glass was responsible for most of this decline.
premise: aluminum recycling was more widely practiced in this period than glass recycling
conclusion: it was found that the weight of glass bottles in household garbage declined by a greater percentage than the weight of aluminum cans.
the container of glass is heavier than that of aluminum~
(A) Glass bottles are significantly heavier than aluminum cans of comparable size.
--right
(B) Recycled aluminum cans were almost all beverage containers, but a significant fraction of the recycled glass bottles had contained products other than beverages.
--the products are irrelevant--it's only about containers
(C) Manufacturers replaced many glass bottles, but few aluminum cans, with plastic containers.
--?--right
(D) The total weight of glass bottles purchased by households increased at a slightly faster rate than the total weight of aluminum cans.
--a slightly faster rate & a greater percentage--no effect
(E) In many areas, glass bottles had to be sorted by color of the glass before being recycled, whereas aluminum cans required no sorting.
--irrelevant
A缺少条件吧~
5.Which of the following, if discovered in experiments using artificially moved prey insects, would support one of the two hypotheses and undermine the other?
background information:Tiger beetles are such fast runners that they can capture virtually any nonflying insect
premise:Perhaps they cannot maintain their pace and must pause for a moment's rest
an alternative hypothesis: while running tiger beetles are unable to process the resulting rapidly changing visual information, and so quickly go blind and stop.
conclusion:when running toward an insect, the beetles intermittently stop, and then, a moment later, resume their attack.
猜不出来~~~
(A) When a prey insect is moved directly toward a beetle that has been chasing it, the beetle immediately turns and runs away without its usual intermittent stopping.
--support neither--no blind and no rest
(B) In pursuing a moving insect, the beetles usually respond immediately to changes in the insect's direction, and pause equally frequently whether the chase is up or down an incline.
--support the first, weaken the second--right
(C) The beetles maintain a fixed time interval between pauses, although when an insect that had been stationary begins to flee, the beetle increases its speed after its next pause.
--pause for a rest and attack-no information about whether it goes blind--right
(D) If, when a beetle pauses, it has not gained on the insect it is pursuing, the beetle generally ends its pursuit.
--irrelevant
(E) When an obstacle is suddenly introduced just in front of running beetles, the beetles sometimes stop immediately, but they never respond by running around the barrier.
--blind, but not support the first one
呃...为什么B错呢?
作者: daisyの小夢想 时间: 2011-11-24 23:04
顶~~!!!!
大家感恩节快乐^^
感谢一路有大家陪伴。
作者: bananazoo 时间: 2011-11-24 23:09
感恩节快乐~
P: human are superior to those of other animals.
C: Once human find this, they will not be satisfied with anything that did by these animals.
G: they will still like these animals, like pets.
And sth those any has may be superior than human.
(A) Certain animals—dolphins and chimpanzees,
for example—appear to be capable of
rational communication.只是说明了有些动物不笨,并不能代表人类对他们满意
(B) Many people familiar both with intellectual
stimulation and with physical pleasures
enjoy the latter more.正确,人们更关心的是物理上的乐趣,这是全身心的愉悦么,这种力量大过了智慧力量,不就反驳了人们对动物不满意这句话~
(C) Someone who never experienced classical
music as a child will usually prefer popular
music as an adult.无关么?
(D) Many people who are serious athletes consider
themselves to be happy.这个只能说明。。。
(E) Many people who are serious athletes love
gourmet food.无关了。。。
P:E wheat have found in E & A
P:The wild form of E wheat was only found in a A place,and there is a relatively strip.
C:the oldest cultivated E wheat was found in the same place, E wheat was first domesticated in that strip.
G: only these place find these.
Answer: D
作者: Richardyu 时间: 2011-11-25 02:26
![](/static/legacy-emoticon/0.gif)
作者: fox0923 时间: 2011-11-25 04:14
顶~~
欢迎花儿加入~~大家都感恩节快乐~话说我刚刚order了两条jeans~~很期待啊![](/static/legacy-emoticon/13.gif)
作者: fox0923 时间: 2011-11-25 05:01
精练-----------------20s------------------WeakenP:Human have cognitive faculties superior to animals.
C: Therefore, humans cannot be happy unless these things involve the gratification of these faculties.
Prephrase: Sometimes it's hard to realize these faculties for human.
Analysis:
(A) Certain animals—dolphins and chimpanzees,
for example—appear to be capable of
rational communication.
----------------------------------------capable of rational communication cannot explain if human can be happy with or without involvement of gratification of cognitive faculties.
(B) Many people familiar both with intellectual
stimulation and with physical pleasures
enjoy the latter more.
----------------------------------------can be the contender, 因为cognitive faculties is the opposite of physical pleasure, 所以如果是注重后者,那么就说明了人们不需要cognitive faculties也是可以happy的。这样就weaken了。
(C) Someone who never experienced classical
music as a child will usually prefer popular
music as an adult.
----------------------------------------感觉好像是在human还没有意识到他们自己有cognitive faculties的时候就已经很happy了。感觉似乎虽然不能直接weaken conclusion,但是可以weaken conclusion造成的原因,即那个premise,但是我记得好像weaken是不应该直接weaken premise的,除非是中间有链接可以,像这种类似于restate premise的内容好像是不能够的。不确定阿~~
(D) Many people who are serious athletes consider
themselves to be happy.
------------------------------------------seriousness = cognitive faculties. and this finally causes happiness??? i do not think so.
(E) Many people who are serious athletes love
gourmet food.
---------------------------------------irrelevant.
不是B就是C,完了以后答案选B,
C的解释是:A preference for a certain type of music is likely a cognition-driven preference, and this preference is expressed by an adult who would certainly be aware of cognitive faculties. And, since no suggestion is made that individuals can be made happy without gratification of the cognitive faculties, this answer is incorrect.
从这里看,似乎要weaken,就直接weaken conclusion,对premise不要有改动才好。求confirm???
作者: Jane412 时间: 2011-11-25 07:45
精练:
time:16''
premise;human have cognitive faculties superior to other animals
premise:they cannot be happy by anything that doesn't involve gratification of these faculties.
即便不是cognitive方面的事物,人们仍然喜欢
A) Certain animals—dolphins and chimpanzees,
for example—appear to be capable of
rational communication.
----------------------和结论扯不上关系。。。
(B) Many people familiar both with intellectual
stimulation and with physical pleasures
enjoy the latter more.
----------------------correct。physical pleasure应该不是cognitive faculty的范畴,intellectual stimulation应该属于cognitive。但是人们还是喜欢后者,所以就weaken了
(C) Someone who never experienced classical
music as a child will usually prefer popular
music as an adult.
--------------------好像没啥关系吧。。。
(D) Many people who are serious athletes consider
themselves to be happy.
---------------------没关系。。。
(E) Many people who are serious athletes love
gourmet food.
---------------------没关系。。。
实在还是不能理解cr的精神世界,选项里面净是些八竿子把不着的东西,还是只有我这么觉得它们和题目毛关系都没有?实在看不出来某些选项的设计意义是啥。。。。
++++++++++++++++++++++++++++++++++++++++++++++++
Cognitive means “relating to the mental process of knowing, including reasoning and judgment.” In other words, cognitive faculties are thinking and analyzing, etc.
C:A preference for a certain type of music is likely a cognition-driven preference, and this preference is expressed by an adult who would certainly be aware of cognitive faculties. And, since no suggestion is made that individuals can be made happy without gratification of the cognitive faculties, this answer is incorrect.
原来对于music的preference是种cognitive faculty。。。真的是理解层面太浅薄了。。。
E: this answer may serve to slightly strengthen the argument because it shows that an individual with experience with the non-cognitive still retains a love of the cognitive.
虽然题目做对了,但对题目和选项并没真正理解
每个选项不是无端设计了。。。之前认识过于简单,所以有多时候做题总有种把握不住的感觉
作者: 南瓜0729 时间: 2011-11-25 08:25
占
作者: qiuhua01234567 时间: 2011-11-25 09:49
标题: 今天题目异常纠结啊!!!
占,感恩节快乐,虽然过了
1.20S weaken
Background:Human beings have cognitive faculties that are
superior to those of other animals
Ways: humans become aware of these
conclusion: they cannot be made happy by anything that does not involve gratification of
these faculties
(A)Certain animals—dolphins and chimpanzees,
for example—appear to be capable of
rational communication.
-----------------------------------------------------irrelevent
(B) Many people familiar both with intellectual
stimulation and with physical pleasures
enjoy the latter more.
------------------------------------------------------correct
(C) Someone who never experienced classical
music as a child will usually prefer popular
music as an adult.
----------------------------------------------------irrelevent
(D) Many people who are serious athletes consider
themselves to be happy.
----------------------------------------------irrelevent
(E) Many people who are serious athletes love
gourmet food.
----------------------------------------------irrelevent
2.31Ssupport.
Background: Traces of cultivated emmer wheat have been found among the earliest agricultural remains of many archaeological sites in Europe and Asia.
Premise: the oldest remains of cultivated emmer wheat yet found are from village sites in the same narrow strip
Conclusion: emmer wheat was first domesticated somewhere in that strip
(A)The presentday distribution of another wild wheat, einkorn, which was also domesticated early in the development of agriculture, covers a much larger area of southwest Asia.
----------------------------------------------------------------------weaken
(B) Modern experiments show that wild emmer wheat can easily be domesticated so as to yield nearly as well as traditionally domestic strains.
----------------------------------------------------------------------irrelevent
(C) At the time when emmer wheat was first cultivated, it was the most nutritious of all the varieties of grain that were then cultivated.
---------------------------------------------------------------------------irrelevent
(D) In the region containing the strip where wild emmer wheat has been found, climatic conditions have changed very little since before the development of agriculture.
---------------------------------------------------------------------------what is mean of since before?
(E) It is very difficult, without genetic testing, to differentiate the wild form of emmer wheat from a closely related wild wheat that also grows in southwest Asia.
-------------------------------------------------------------------------------------------------------weaken
3.23S.weaken
Background: In the late 1980s, the population of sea otters in the North Pacific began to decline.two plausible explation:predation,disease.
Premise: a concurrent sharp decline in populations of seals and sea lions is believed to have been caused by disease, and diseases that infect these creatures are likely to be able to infect sea otters also.
Conclusion: disease is the more likely.
(A)Killer whales in the North Pacific usually prey on seals and sea lions but will, when this food source is scarce, seek out other prey.
-------------------------------------------------irrelevent
(B) There is no indication that the sea otter population at any North Pacific location declined in the 1980s because of substantial numbers of sea otters migrating to other locations.
------------------------------------------------irrelevent
(C) Along the Pacific coast of North America in the 1980s, sea otters were absent from many locations where they had been relatively common in former times.
--------------------------------------------------irrelevent
(D) Following the decline in the population of the sea otters, there was an increase in the population of sea urchins, which are sea otters' main food source.
-----------------------------------------------------irrelevent
(E) The North Pacific populations of seals and sea lions cover a wider geographic area than does the population of sea otters.
------------------------------------------------------------correct
4.30S.paradox
background: From 1978 to 1988, beverage containers accounted for a steadily decreasing percentage of the total weight of household garbage in the United States.The increasingly widespread practice of recycling aluminum and glass was responsible for most of this decline.
ways: aluminum recycling was more widely practiced in this period than glass recycling
Founds: the weight of glass bottles in household garbage declined by a greater percentage than the weight of aluminum cans
(A)Glass bottles are significantly heavier than aluminum cans of comparable size.
-------------------------------------------------------------------------------correct
(B) Recycled aluminum cans were almost all beverage containers, but a significant fraction of the recycled glass bottles had contained products other than beverages.
------------------------------------------------------------------------irrelevent
(C) Manufacturers replaced many glass bottles, but few aluminum cans, with plastic containers.
-----------------------------------------------------------------------wrong,weaken the conclusion
(D) The total weight of glass bottles purchased by households increased at a slightly faster rate than the total weight of aluminum cans.
-----------------------------------------------------------------------irrelevent
(E) In many areas, glass bottles had to be sorted by color of the glass before being recycled, whereas aluminum cans required no sorting.
----------------------------------------------------------------------------------------------irrelevent
5.
Background: Tiger beetles are such fast runners that they can capture virtually any nonflying insect.However, when running toward an insect, the beetles intermittently stop, and then, a moment later, resume their attack
One explation: Perhaps they cannot maintain their pace and must pause for a moment's rest
Another explation: while running tiger beetles are unable to process the resulting rapidly changing visual information, and so quickly go blind and stop
A) When a prey insect is moved directly toward a beetle that has been chasing it, the beetle immediately turns and runs away without its usual intermittent stopping.
(B) In pursuing a moving insect, the beetles usually respond immediately to changes in the insect's direction, and pause equally frequently whether the chase is up or down an incline.
(C) The beetles maintain a fixed time interval between pauses, although when an insect that had been stationary begins to flee, the beetle increases its speed after its next pause.
(D) If, when a beetle pauses, it has not gained on the insect it is pursuing, the beetle generally ends its pursuit.
(E) When an obstacle is suddenly introduced just in front of running beetles, the beetles sometimes stop immediately, but they never respond by running around the barrier.
作者: 小意达de花儿 时间: 2011-11-25 10:20
1. Human’s cognitive faculties are superior to those of other animals
C:Human cannot be made happy by anything that does not involve G of these faculties
I GUESS: Others animals can be made happy by anything that does not involve G of these faculties.
(A) Certain animals—dolphins and chimpanzees, for example—appear to be capable of rational communication.
Irrelevant to the rational communication of D & C
(B) Many people familiar both with intellectual stimulation and with physical pleasures enjoy the latter more.
Correct, they also enjoy physical pleasures, which does not involve G of these faculties
(C) Someone who never experienced classical music as a child will usually prefer popular music as an adult.
Irrelevant to the music preference
(D) Many people who are serious athletes consider themselves to be happy.
Not as sufficient as B
(E) Many people who are serious athletes love gourmet food.
Irrelevant to food preference.
-------------------------------------------------------------------------------------------------
2.Traces of cultivated EW have been found in Europe and Asia→
Narrow strip of SA is the only place where wild form of EM has been found growing→
Because oldest remains of cultivated EM yet found are from village sites in the same narrow strip→
CONCLUSION: EM was first domesticated in that strip
(A) The present day distribution of another wild wheat, einkorn, which was also domesticated early in the development of agriculture, covers a much larger area of southwest Asia.
Irrelevant to einkorn
(B) Modern experiments show that wild emmer wheat can easily be domesticated so as to yield nearly as well as traditionally domestic strains.
Irrelevant to whether it is easy to domesticate EM
(C) At the time when emmer wheat was first cultivated, it was the most nutritious of all the varieties of grain that were then cultivated.
Irrelevant
(D) In the region containing the strip where wild emmer wheat has been found, climatic conditions have changed very little since before the development of agriculture.
Correct, the climatic conditions are preferable
(E) It is very difficult, without genetic testing, to differentiate the wild form of emmer wheat from a closely related wild wheat that also grows in southwest Asia.
Weaken
------------------------------------------------------------------------------------------------
3. The population of sea otters is declining
Two: possible reasons----predation & disease
Since Population of seals and sea lions is declining because of disease
Conclusion: The reason of declination of sea otters’ population is disease
(A) Killer whales in the North Pacific usually prey on seals and sea lions but will, when this food source is scarce, seek out other prey.
Correct
(B) There is no indication that the sea otter population at any North Pacific location declined in the 1980s because of substantial numbers of sea otters migrating to other locations.
Irrelevant
(C) Along the Pacific coast of North America in the 1980s, sea otters were absent from many locations where they had been relatively common in former times.
This is only a evidence of population of sea otters is declining
(D) Following the decline in the population of the sea otters, there was an increase in the population of sea urchins, which are sea otters' main food source.
Irrelevant
(E) The North Pacific populations of seals and sea lions cover a wider geographic area than does the population of sea otters.
Irrelevant comparision
-------------------------------------------------------------------------------------------------
4.Background: Beverage containers accounted for a decreasing percentage of household garbage in US. And practice of recycling aluminum and glass was responsible for most of this decline
Aluminum recycling was more widely practiced than glass recycling
But: Weight of glass bottles in household garbage declined more than the weight of aluminum cans
I guess: Glass Bottles are widely used
(A) Glass bottles are significantly heavier than aluminum cans of comparable size.
Insufficient, we do not know the total weight. 这题一开始错选了这个选项,没有考虑total weight AND single weight
(B) Recycled aluminum cans were almost all beverage containers, but a significant fraction of the recycled glass bottles had contained products other than beverages.
Insufficient, in the conclusion, it does not mention beverage containers
(C) Manufacturers replaced many glass bottles, but few aluminum cans, with plastic containers.
Correct, this account for the significantly declination of weight og glass bottles, even though the recycling of aluminum cans are more practiced.
(D) The total weight of glass bottles purchased by households increased at a slightly faster rate than the total weight of aluminum cans.
When we see the word ‘rate’, we should think about the relationship between rate and quantity
(E) In many areas, glass bottles had to be sorted by color of the glass before being recycled, whereas aluminum cans required no sorting.
Irrelevant
-------------------------------------------------------------------------------------------------
5. Background: Tiger beetles can capture any non-flying insect
But when running, tiger beetles stop intermittently
Two possible reasons: 1. they cannot maintain their pace and need a rest; 2. momentary blind
(A) When a prey insect is moved directly toward a beetle that has been chasing it, the beetle immediately turns and runs away without its usual intermittent stopping.
Overturn the hypothesis of momentary blind, but do not support the other one.
(B) In pursuing a moving insect, the beetles usually respond immediately to changes in the insect's direction, and pause equally frequently whether the chase is up or down an incline.
Overturn the momentary blind hypothesis.
(C) The beetles maintain a fixed time interval between pauses, although when an insect that had been stationary begins to flee, the beetle increases its speed after its next pause.
Correct, overturn the momentary blind, because the insects do not change direction in a fixed time interval. The beetles increase its speed after its next pause support the other hypothesis
(D) If, when a beetle pauses, it has not gained on the insect it is pursuing, the beetle generally ends its pursuit.
Irrelevant
(E) When an obstacle is suddenly introduced just in front of running beetles, the beetles sometimes stop immediately, but they never respond by running around the barrier.
Overture the momentary blind, but do not support the other hypothesis.
作者: zz42050524 时间: 2011-11-25 10:27
占位下午做 感觉自己做作业太慢了
作者: winghyy 时间: 2011-11-25 12:07
精炼题 35s
Premise:human beings have better cognitive than other animals.
Therefore,once humans become aware of these, they will not be happy unless with gratification of these faculties.
Prephase: 举反例?
选D
(A) Certain animals—dolphins and chimpanzees, for example—appear to be capable of rational communication.
结论中是讲humans的行为,与动物无关 irrelevant
(B) Many people familiar both with intellectual stimulation and with physical pleasures enjoy the latter more.
(C) Someone who never experienced classical music as a child will usually prefer popular music as an adult.
child vs adult的对比 无关
(D) Many people who are serious athletes consider themselves to be happy.
举了反例,
(E) Many people who are serious athletes love gourmet food.
对美食鉴赏也是一种认知能力,没有weaken
又错了。。。越“错”越勇,之前徘徊在B,D中。
作者: winghyy 时间: 2011-11-25 13:40
逻辑链
21. 50s
BG: Trace of cultivated emmer wheat have been found in Europe and Asia. The only place where the wild EW was found was in southwest Asia.
Premise: the oldest remains of cultivated emmer wheat yet found are from village sites in the same narrow strip.
Conclusion: it is clear that emmer wheat was first domesticated somewhere in that strip.
Prephase: There is no other evidence show that EM was first domesticated in Europe.
D
22. 45s-weaken
Premise:The pupulation of Sea Otter declined. The possible reasons: 1. predation 2.disease. Since the population of whales and sea lions declined because of disease.
Conclusion: The reason is disease.
Prephase: Offer new evidence that support "predation"
A
23. 70s-strengthen
Premise:Beverage container accounted for decreasing percentage of the total weight of household garbage, because of the recycling aluminum and glass.
findings: although AL recycling was more widely practiced than glass recycling, the weight of glass bottles in household garbage declined by a greater percentage than the weight of aluminum cans.
Prephase: there is another cause for the decline of glass.
C
24. 60s- strengthen one and weaken the other
BG: The tiger bettle can capture any nonflying insects. But during its chase, the tiger bettle will stop during the running and resume the attack a moment later.
Premise: There are two reasons: 1. run too fast so that it can not maintain the pace 2. rapidly changing visual information makes it go blind.
B
(这道OG上做过。。。好像还是错了)
作者: qiuhua01234567 时间: 2011-11-25 17:05
1.background information:Human beings have cognitive faculties that are superior to those of other animalspremise:once humans become aware of theseconclusion:they cannot be made happy by anything that does not involve gratification of these facultiesis there a link between happiness and cognitive faculties?(A) Certain animals—dolphins and chimpanzees, for example—appear to be capable of rational communication.--the premise and conclusion are all about humans--irrelevant
(B) Many people familiar both with intellectual stimulation and with physical pleasures enjoy the latter more.--weaken--right
(C) Someone who never experienced classical music as a child will usually prefer popular music as an adult.--irrelevant
(D) Many people who are serious athletes consider themselves to be happy.--support-
-if the athletes are happy because of the love of sports they play~right(E) Many people who are serious athletes love gourmet food.--???--slightly strengthen做对了~可是思路又错了!!!To weaken the argument we must show that individuals can be made happy without gratification of the cognitive faculties. Cognitive means “relating to the mental process of knowing, including reasoning and judgment.” B:Additionally, this answer has the benefit of addressing the phrase in the stimulus regarding awareness of cognitive faculties: “once humans become aware of these...” In this answer, unlike others, the individuals are known to be familiar with cognitive faculties. While we believe that recognition of cognitive faculties is inherent in adults (or some of the named types in other answers, such as serious athletes, who by definition would have to be teens or adults), this answer is stronger because it explicitly addresses the issue.这道题~还是需要多看几遍呀2.background information:Traces of cultivated emmer wheat have been found among the earliest agricultural remains of many archaeological sites in Europe and Asia.The only place where the wild form of emmer wheat has been found growing is a relatively narrow strip of southwest Asia.premise:the oldest remains of cultivated emmer wheat yet found are from village sites in the same narrow strip,conclusion:emmer wheat was first domesticated somewhere in that strip.is there a link between the location of the wild form of emmer and the place first domesticated it???what about other continents?(A) The present day distribution of another wild wheat, einkorn, which was also domesticated early in the development of agriculture, covers a much larger area of southwest Asia.--another wild wheat--irrelevant
(B) Modern experiments show that wild emmer wheat can easily be domesticated so as to yield nearly as well as traditionally domestic strains.--the difficulty --irrelevant
(C) At the time when emmer wheat was first cultivated, it was the most nutritious of all the varieties of grain that were then cultivated.--the nutrition--irrelevant
(D) In the region containing the strip where wild emmer wheat has been found, climatic conditions have changed very little since before the development of agriculture.--since before什么意思?--right?排除法
(E) It is very difficult, without genetic testing, to differentiate the wild form of emmer wheat from a closely related wild wheat that also grows in southwest Asia.
--weaken思路不对~选项不懂![](/static/legacy-emoticon/5.gif)
3.
background information:In the late 1980s, the population of sea otters in the North Pacific began to decline.There are two plausible explanations for the decline: predation, possibly by killer whales, or disease.premise:a concurrent sharp decline in populations of seals and sea lions is believed to have been caused by disease, and diseases that infect these creatures are likely to be able to infect sea otters also.conclusion:Of these two, disease is the more likelywhat about killer whales? it doesn't mention it~~~"are likely to be able to"~and is the disease fatal to sea otters?(A) Killer whales in the North Pacific usually prey on seals and sea lions but will, when this food source is scarce, seek out other prey.
--seek sea otters--right
(B) There is no indication that the sea otter population at any North Pacific location declined in the 1980s because of substantial numbers of sea otters migrating to other locations.
--slightly support or irrelevant
(C) Along the Pacific coast of North America in the 1980s, sea otters were absent from many locations where they had been relatively common in former times.
--it isn't an explanation--irrelevant
(D) Following the decline in the population of the sea otters, there was an increase in the population of sea urchins, which are sea otters' main food source.
--it must happened--irrelevant
(E) The North Pacific populations of seals and sea lions cover a wider geographic area than does the population of sea otters.
--irrelevant
4.
background information:From 1978 to 1988, beverage containers accounted for a steadily decreasing percentage of the total weight of household garbage in the United States.The increasingly widespread practice of recycling aluminum and glass was responsible for most of this decline.premise:aluminum recycling was more widely practiced in this period than glass recyclingconclusion:it was found that the weight of glass bottles in household garbage declined by a greater percentage than the weight of aluminum cans.the container of glass is heavier than that of aluminum~(A) Glass bottles are significantly heavier than aluminum cans of comparable size.--right
(B) Recycled aluminum cans were almost all beverage containers, but a significant fraction of the recycled glass bottles had contained products other than beverages.--the products are irrelevant--it's only about containers
(C) Manufacturers replaced many glass bottles, but few aluminum cans, with plastic containers.--?--right
(D) The total weight of glass bottles purchased by households increased at a slightly faster rate than the total weight of aluminum cans.--a slightly faster rate & a greater percentage--no effect
(E) In many areas, glass bottles had to be sorted by color of the glass before being recycled, whereas aluminum cans required no sorting.--irrelevantA缺少条件吧~5.Which of the following, if discovered in experiments using artificially moved prey insects, would support one of the two hypotheses and undermine the other?
background information:Tiger beetles are such fast runners that they can capture virtually any nonflying insect
premise:Perhaps they cannot maintain their pace and must pause for a moment's rest
an alternative hypothesis: while running tiger beetles are unable to process the resulting rapidly changing visual information, and so quickly go blind and stop.
conclusion:when running toward an insect, the beetles intermittently stop, and then, a moment later, resume their attack.
猜不出来~~~
(A) When a prey insect is moved directly toward a beetle that has been chasing it, the beetle immediately turns and runs away without its usual intermittent stopping.
--support neither--no blind and no rest
(B) In pursuing a moving insect, the beetles usually respond immediately to changes in the insect's direction, and pause equally frequently whether the chase is up or down an incline.
--support the first, weaken the second--right
(C) The beetles maintain a fixed time interval between pauses, although when an insect that had been stationary begins to flee, the beetle increases its speed after its next pause.
--pause for a rest and attack-no information about whether it goes blind--right
(D) If, when a beetle pauses, it has not gained on the insect it is pursuing, the beetle generally ends its pursuit.
--irrelevant
(E) When an obstacle is suddenly introduced just in front of running beetles, the beetles sometimes stop immediately, but they never respond by running around the barrier.
--blind, but not support the first one呃...为什么B错呢?-- by 会员 xeyyxzty (2011/11/24 22:42:05)
能把你做的正确的思路再写下我嘛?今天题目做得我异常纠结啊、谢谢啊
作者: balapupu 时间: 2011-11-25 21:40
欢迎小花~~1.【40s】【看了两遍原来最后一句是双重否定】
P:people have cognitive ability better than animals-->C
nce that realize this-->they will not happy by anything,with out this cognitive ability.
weaken:animals also have feelings of happiness.
A:
(A) Certain animals—dolphins and chimpanzees,
for example—appear to be capable of
rational communication.-->does not mean they have cognitive ability better than people.
(B) Many people familiar both with intellectual
stimulation and with physical pleasures
enjoy the latter more.-->does not indicate without cognitive ability
(C) Someone who never experienced classical
music as a child will usually prefer popular
music as an adult.-->R-->means not know about the popular but still like the popular music.
(D) Many people who are serious athletes consider
themselves to be happy.-->serious does not mean no cognitive ability.
(E) Many people who are serious athletes love
gourmet food.-->serious does not mean no cognitive ability.
[又错了~~~每次做的很伤有木有。。。。]
作者: qiuhua01234567 时间: 2011-11-25 21:54
欢迎小花~~
-- by 会员 balapupu (2011/11/25 21:40:55)
是我嘛?自恋下可不可以![](/static/legacy-emoticon/2.gif)
作者: fox0923 时间: 2011-11-25 23:37
欢迎小花~~1.【40s】【看了两遍原来最后一句是双重否定】
P:people have cognitive ability better than animals-->C
![](static/image/smiley/default/shocked.gif)
nce that realize this-->they will not happy by anything,with out this cognitive ability.
weaken:animals also have feelings of happiness.
A:
(A) Certain animals—dolphins and chimpanzees,for example—appear to be capable ofrational communication.-->does not mean they have cognitive ability better than people.(B) Many people familiar both with intellectualstimulation and with physical pleasuresenjoy the latter more.-->does not indicate without cognitive ability(C) Someone who never experienced classicalmusic as a child will usually prefer popularmusic as an adult.-->R-->means not know about the popular but still like the popular music.(D) Many people who are serious athletes considerthemselves to be happy.-->serious does not mean no cognitive ability.(E) Many people who are serious athletes lovegourmet food.-->serious does not mean no cognitive ability.[又错了~~~每次做的很伤有木有。。。。]-- by 会员 balapupu (2011/11/25 21:40:55)
有,已经麻木了~所以再被伤下去就不会感觉痛了。
作者: zz42050524 时间: 2011-11-25 23:58
天亮来补,我现在就是一个打作业补丁的~~~~~
作者: UlysessHope 时间: 2011-11-26 09:21
啊啊啊…昨天是我值班 记成今天了 囧! 该打啊该打 我赶快发上!
作者: 小意达de花儿 时间: 2011-11-26 10:12
欢迎小花~~
-- by 会员 balapupu (2011/11/25 21:40:55)
是我嘛?自恋下可不可以
![](/static/legacy-emoticon/2.gif)
-- by 会员 qiuhua01234567 (2011/11/25 21:54:18)
那我是不是也可以自恋一下
作者: zz42050524 时间: 2011-11-26 10:36
精炼 weaken 23s
P: Human beings have the cognitive faculties that are superior to animals.
C: Humans do something without cognitive thinking would be happy.
Prephrase: Something without cognitive thinking can also makes human happy.
(A) Certain animals—dolphins and chimpanzees, 这是无关项,结论重点是human happy
for example—appear to be capable of
rational communication.
(B) Many people familiar both with intellectual 这事答案,物理运动不需要cognitive
stimulation and with physical pleasures thinking,依然能让你happy
enjoy the latter more.
(C) Someone who never experienced classical 这是无关项 和cognitive 不联系
music as a child will usually prefer popular
music as an adult.
(D) Many people who are serious athletes consider 这是shell game,运动员认为自己快乐,
themselves to be happy. 但是没有说快乐的原因是否需要cognitive
thinking
(E) Many people who are serious athletes love 无关项
gourmet food.
逻辑链: 2. Strengthen 40s
P: Archaeologists trace the origin of the wheat. The only place of wild wheat and the oldest place of cultivated wheat are the same.
C: That place is the origin of the wheat.
Prephrase: wheat 其他种植的地方都比这个地方种的时间晚
D : 推测又错,气候的没有变化说明这里适宜种植wheat,增强结论。
3. Weaken 32s
P: The number of sea otters has declined, two possible reasons: predation and disease. The disease that caused sea lions decline is likely to affect other animals.
C; disease is the reason.
Some evidence about predation was the reason or may other reasons.
B other reasons
4. 47s
P: Because of aluminum and glass bottle, containers account for decreasing percentage of garbage.
C: G decreases significantly than A.
A 好用,更受欢迎
C 思路又没对
5: 55s
P: TB can capture any non-flying insect, before attack, it will stop intermittently.
C; It runs so fast and can not maintain it’s pace, stop to rest
C2: When Tb running it can not capture the sight.
…预测不出来。。
B 不知道正确与否 感觉关键在于去考虑 一个人weaken 另一个还要是support ,仅仅满足一个的都是错的
作者: 风无衣 时间: 2011-11-26 16:07
【精炼1-6】
weaken_35s
premise:human have cognitive faculties that superior to those of other animals
conclusion:human will not feel happy without gratification of these faculties.
(A) Certain animals—dolphins and chimpanzees,
for example—appear to be capable of
rational communication.——irrelevant
(B) Many people familiar both with intellectual
stimulation and with physical pleasures
enjoy the latter more.
(C) Someone who never experienced classical
music as a child will usually prefer popular
music as an adult.
(D) Many people who are serious athletes consider
themselves to be happy.——irrelevant
(E) Many people who are serious athletes love
gourmet food.——irrelevant
B、C之间还是选错了~我已经不知道该怎么分析题目~啊~~~
Answer choice (B): This is the correct answer, and a somewhat risqué one at that. By showing that many people enjoy the physical more than the cognitive, the answer shows that people can be made happy by gratification of something other than cognitive faculties. Cognitive faculties, being mental in nature, are of course distinct from physical pleasures.
Additionally, this answer has the benefit of addressing the phrase in the stimulus regarding awareness of
cognitive faculties: “once humans become aware of these...” In this answer, unlike others, the individuals are known to be familiar with cognitive faculties. While we believe that recognition of cognitive faculties is inherent in adults (or some of the named types in other answers, such as serious athletes, who by definition would have to be teens or adults), this answer is stronger because it explicitly addresses the issue.
Answer choice (C): A preference for a certain type of music is likely a cognition-driven preference, and
this preference is expressed by an adult who would certainly be aware of cognitive faculties. And, since no suggestion is made that individuals can be made happy without gratification of the cognitive faculties, this answer is incorrect.
作者: balapupu 时间: 2011-11-26 20:54
欢迎小花~~
-- by 会员 balapupu (2011/11/25 21:40:55)
是我嘛?自恋下可不可以
![](/static/legacy-emoticon/2.gif)
-- by 会员 qiuhua01234567 (2011/11/25 21:54:18)
那我是不是也可以自恋一下
-- by 会员 小意达de花儿 (2011/11/26 10:12:09)
各位太欢乐了~~原来我们小分队里有这么多花~~哈哈~~那就那就欢迎大花~二花~~和小花~~![](/static/legacy-emoticon/20.gif)
作者: trackofheart 时间: 2011-11-26 21:34
Human beings have cognitive faculties that are superior to those of other animals(前提),and once humans become aware of these, they cannot be made happy by anything that does not involve gratification of
these faculties.(结论)
Which one of the following statements, if true, most calls into question the view above?
———Bible Weaken Question Problem Set NO.1
(A) Certain animals—dolphins and chimpanzees, for example appear to be capable of rational communication.
(B) Many people familiar both with intellectual stimulation and with physical pleasures
enjoy the latter more.
(C) Someone who never experienced classical music as a child will usually prefer popular
music as an adult.
(D) Many people who are serious athletes consider themselves to be happy.
(E) Many people who are serious athletes love gourmet food.
横竖都没有看出来选项与文章的关系。。。
额。。没有明白faculty有能力,天赋的意思。。。
作者: chensong211 时间: 2011-11-26 23:09
同学们加油,最近工作超级忙。掉队了。给大家打气。
作者: ugly5552000 时间: 2011-11-27 13:49
1/background: human beings cognitive faculties more advanced than those of animals, when they are aware of the fact, they cannot be happy by anything that does not show such advantages of faculties.
Premise: most of human beings’ happiness do not come from the superior cognitive faculties.
Prephrase: because of the cognitive faculties, human beings’ thoughts will be more complex, sometimes they are worried about or get troubled by these faculties, it is more difficult for them to be happy.
B
A: irrelevant, the argument is about human beings and being happy.
B: the answer means that it is the physical pleasures that people enjoy more, it attacks the argument, makes the cognitive faculties a insufficient condition in acquiring happiness.
C: the answer in some extent support the argument.
D: it does not mention it is cognitive faculties make him happy.
E: irrelevant.
2/Background: among the earliest agricultural remains of many agricultural site in Europe and Asia, the cultivated emmer wheat have been found. The wild emmer wheat was only found in a strip of southwest Asia, where the oldest cultivated emmer wheat was also found.
Premise: the emmer wheat was first cultivated in the strip.
Prephrase: there is no cultivated emmer wheat older than that in the strip have been found in other places.
D
A: einkon is irrelevant.
B: the production of emmer wheat is irrelevant.
C: irrelevant to why the strip is the first place where emmer wheat is cultivated.
D: the climate condition is preferable to the emmer wheat, support.
E: differentiation is irrelevant
3/Background: in the late 1980s, the population of sea otters in the North Pacific began to decline, which is more likely to be caused be disease than by killer whales.
Premise: the disease concurrently caused a sharp decline in populations of seals and sea lions might infect sea otters.
Prephrase: the sea otters are naturally immune to the disease, the living areas of sea otters and seals and sea lions are far away from each other.
A
A: with the sharp decline in population of seals and sea lions, the killer whales has less food, they switch to prey sea otters as their food, this make the decline of the population of sea otter, not the disease. Weaken.
B: irrelevant.
C: this is another explanation, but irrelevant to de decline of the population of sea otters.
D: the of outcome of decline, irrelevant.
E: no connection with the geographic areas, irrelevant.
4/Background: because of the widespread practice of recycling aluminum and glass, the beverage containers accounted for a decreasing proportion of the total from 1978 to 1988.
Premise: although aluminum recycling was more widely practiced than glass, it was found that the decline of the weight of glass bottles garbage declined greater than that of aluminum cans.
Prephrase: the recycle procedure of glass bottles is much simple than that of aluminum cans. Some aluminum cans is unrecyclable. Concerning with the lighter aluminum, beverage producers change their glass bottles with aluminum cans, in order to reduce the cost of transportation.
C
A: it omits the number of aluminum cans, what if the number of aluminum cans is significantly more than that of glass bottles?
B: whether the glass bottles contain beverage or not, they will discarded as garbage.
C: there are much less glass bottles than aluminum cans, support.
D: weaken
E: irrelevant
5/Background: it is found that the beetles intermittently stop and then resume the attack while chasing an insect.
Premise: the beetles might not maintain its pace while running fast, or they are unable to adjust to the rapid changes of visual information and quickly go blind.
C
A: only weakens the second assumption
B: weakens both assumptions
C: it weakens the first assumption and support the second one
D: irrelevant
E:new situation, irrelevant
作者: balapupu 时间: 2011-11-28 23:20
【28s】
P:the cultivated emmer were found in Europe and Asia?the only place have wild emmer is Asia?the oldest remains of emmer in the same place?emmer was first domestic in that place.
Support: the domestic place is more likely to have remains than other places.
A:
(A) The presentday distribution of another wild wheat, einkorn, which was also domesticated early in the development of agriculture, covers a much larger area of southwest Asia.-->irrelevant with other food.
(B) Modern experiments show that wild emmer wheat can easily be domesticated so as to yield nearly as well as traditionally domestic strains.-->R
(C) At the time when emmer wheat was first cultivated, it was the most nutritious of all the varieties of grain that were then cultivated.-->irrelevant with comparison of the nutritious
(D) In the region containing the strip where wild emmer wheat has been found, climatic conditions have changed very little since before the development of agriculture.-->irrelevant with the climate.
(E) It is very difficult, without genetic testing, to differentiate the wild form of emmer wheat from a closely related wild wheat that also grows in southwest Asia.-->irrelevant comparison.
[对于选项不是很理解,这个和开始发展农业前的气候变化有什么关系???]
2.【24s】
P:The population of sea otters is decline?1)predation2)disease?other sea animal’s population also decline because of the disease.-->they will all infect.
Weaken: there are some other sea animals that likely to get the disease but their number increase.
A:
(A) Killer whales in the North Pacific usually prey on seals and sea lions but will, when this food source is scarce, seek out other prey.-->R. provide other reason
(B) There is no indication that the sea otter population at any North Pacific location declined in the 1980s because of substantial numbers of sea otters migrating to other locations.-->refute the statement in the passage.
(C) Along the Pacific coast of North America in the 1980s, sea otters were absent from many locations where they had been relatively common in former times.->repeat the statement.
(D) Following the decline in the population of the sea otters, there was an increase in the population of sea urchins, which are sea otters' main food source.-->irrelevant with the following consequences.
(E) The North Pacific populations of seals and sea lions cover a wider geographic area than does the population of sea otters.-->irrelevant with the area.
3.[24s]
P: aluminum and glass containers are widely used today?the glass containers are more widely used than the aluminum containers? however, the decreased percent of glass is greater the aluminum.
Explain: the glass containers are more collected by people for recycle
A:
(A) Glass bottles are significantly heavier than aluminum cans of comparable size.—>irrelevant the weight
(B) Recycled aluminum cans were almost all beverage containers, but a significant fraction of the recycled glass bottles had contained products other than beverages.-->narrow the scope to the bverage.
(C) Manufacturers replaced many glass bottles, but few aluminum cans, with plastic containers.-->R
(B) (D) The total weight of glass bottles purchased by households increased at a slightly faster rate than the total weight of aluminum cans.-->deepen the paradox
(E) In many areas, glass bottles had to be sorted by color of the glass before being recycled, whereas aluminum cans required no sorting.-->irrelevant with the sort way.
4.[28s]
P:tiger beetles stop and then move when they cathing food?1)they can not maintain their pace and need rest 2) they will become blind because the changing visual.
Support one and weaken one:
When placed the tiger beetles in a box keep changing the color, they can not move.
A:
(A) When a prey insect is moved directly toward a beetle that has been chasing it, the beetle immediately turns and runs away without its usual intermittent stopping.
(B) In pursuing a moving insect, the beetles usually respond immediately to changes in the insect's direction, and pause equally frequently whether the chase is up or down an incline.
(C) The beetles maintain a fixed time interval between pauses, although when an insect that had been stationary begins to flee, the beetle increases its speed after its next pause.
(D) If, when a beetle pauses, it has not gained on the insect it is pursuing, the beetle generally ends its pursuit.
(E) When an obstacle is suddenly introduced just in front of running beetles, the beetles sometimes stop immediately, but they never respond by running around the barrier.
[额。。。B和C的区别在哪里啊。。。]
作者: doraliu 时间: 2011-12-12 09:15
求加入~~~求群号~~~
作者: daisyの小夢想 时间: 2011-12-13 10:49
求加入~~~求群号~~~
-- by 会员 doraliu (2011/12/12 9:15:45)
我们的群规是:要做满20天的练习才有权加入。呵呵。好好加油!
作者: lan0604 时间: 2012-2-17 00:08
精炼:
00:35
weaken
involve gratification--->make people happy
(A) Certain animals—dolphins and chimpanzees,
for example—appear to be capable of
rational communication.
------------------------------------irrevelent
(B) Many people familiar both with intellectual
stimulation and with physical pleasures
enjoy the latter more.
------------------------------------怪怪的
错误了,B是正确的
(C) Someone who never experienced classical
music as a child will usually prefer popular
music as an adult.
-------------------------------------irrevelent
(D) Many people who are serious athletes consider
themselves to be happy.
-------------------------------------correct
(E) Many people who are serious athletes love
gourmet food.
-------------------------------------irrevelent
21
00:51
support
wheat was first domesticated somewhere in the strip
D
22
00:30
weaken
seals and sea lions caused by diease & can be infected--->disease
A
23
00:33
support
glass bottles in household declined more
C
25
00:43
weak one, support another
H1:cannot maintain their pace
H2:go blind
C么?介个不会啊,好绕
(A) When a prey insect is moved directly toward a beetle that has been chasing it, the beetle immediately turns and runs away without its usual intermittent stopping.
啥都没说清楚
(B) In pursuing a moving insect, the beetles usually respond immediately to changes in the insect's direction, and pause equally frequently whether the chase is up or down an incline.
没有blind,undermine H2,与速度无关,undermine H1,不知道这样解释对不对捏
(C) The beetles maintain a fixed time interval between pauses, although when an insect that had been stationary begins to flee, the beetle increases its speed after its next pause.
为保持速度而停,SUPPORT H1;看到prey动,undermine H2
(D) If, when a beetle pauses, it has not gained on the insect it is pursuing, the beetle generally ends its pursuit.
啥都没说清楚
(E) When an obstacle is suddenly introduced just in front of running beetles, the beetles sometimes stop immediately, but they never respond by running around the barrier.
啥
作者: Rena张 时间: 2012-4-9 14:01
昨天调整复习计划去了,没按时完成作业,现在马上就补上~~![](/static/legacy-emoticon/30.gif)
-----------------
今天又错了好多……精练是猜对的,逻辑链错了三~~~![](/static/legacy-emoticon/43.gif)
![](/static/legacy-emoticon/43.gif)
又不好意思贴出来了~~~得研究下~~~
作者: maestri 时间: 2012-4-9 14:45
先写精炼题:
1) 计时:45s
2) 逻辑链Situation:
Background Information:
Human beings have cognitive faculties that are superior to those of other animals
Premise:
once humans become aware of these
Conclusion:
they cannot be made happy by anything that does not involve gratification of these faculties.
3) 推测:whether people will be happy because of other things apart from cognitive faculties?
4) 选项分析:选B
(A) Certain animals—dolphins and chimpanzees, or example—appear to be capable of
rational communication.---has nothing to do with the conclusion
(B) Many people familiar both with intellectual stimulation and with physical pleasures
enjoy the latter more.—Correct: People will also be happy for physical pleasures
(C) Someone who never experienced classical music as a child will usually prefer popular music as an adult.--- has nothing to do with the conclusion
(D) Many people who are serious athletes consider themselves to be happy.—Don’t mention why athletes will be happy. The conclusion talks about the reason of happiness
(E) Many people who are serious athletes love gourmet food.—has nothing to do with the conclusion
作者: corneliaflower 时间: 2012-4-30 17:43
精炼
人们在认知能力方面超过其他动物。所以人们一旦意识到,就很难对于那些没有这种认知能力的事物感到开心。
weaken:不是认知能力的事物也可以开心
~B
此题好难读懂啊T.T LAST太绕了
作者: dwindwin1106 时间: 2012-5-4 05:53
(1)
P:human beings have cognitive faculties that are superior than those of other animals
C
nce they become aware of these, they cannot be happy by anything that does not include gratification of these faculties
Weaken: there are other things that can make people happy except things that involve gratification of cognitive faculties.
Answer: B
A.Irrelevant ------ the topic is about human’s cognitive faculties not animals
B.Correct ------ it shows that physical factors not cognitive factors can make people happy, a statement that is opposite to the argument.
C.Irrelevant ------ it shows that cognitive faculties have some influence on people’s life to some extent.
D.Irrelevant ------ it does not explain whether the happiness is from cognitive aspect or not.
E.Irrelevant ------ it relates to cognitive faculties to some extent.
(2)
P:the cultivated EW has been found in Eu and Asia.
P:the only place where wild EW has been found is a relatively narrow strip of southwest Asia.
C: since the oldest cultivated EW was found from village sites in the same narrow strip, EW was fist domesticated somewhere in that strip.
Strengthen:
修正思路:排除其他原因,如气候,D选项表示那里的气候基本不变,因此适合人们耕种
Answer: D
A.irrelevant
B.irrelevant
C.irrelevant
D.correct
E.weaken
(3)
P:in the late 1980s, sea otter in the NP declined
2 reasons: predation or disease
P:since a concurrent sharp decline in population of seals and sea lions is cause by disease and the this kind of disease are likely to be able to infect sea otters, too
C:sea otter is more likely to be killed by disease.
Weaken: there is few possibility for sea otter to be infected
修正思路:给出了2种原因,要否定其中一种,指出另一种原因的可能性更大
Answer: A
A.Correct seals和sea lions数量下降使得kill whales吃sea otter的可能性增大-->sea otter因为predation而下降的可能性增大
B.Irrelevant
C.Irrelevant
D.Irrelevant
E.Irrelevant
(4)
F:increasingly practice of recycling al and gl ? decreasing percentage of the total weight of HG. In US
F:al recycling > gl recycling
F:the weight of gl in HG declined by a greater percentage than the weight of al.
Resolve: other reasons lead to the situation
Answer: C
A.irrelevant
B.irrelevant
C.correct 由于被替换所以gl少了,所以下降比例大,而且回收的量也不用增大
D.irrelevant
E.irrelevant
(5)
P:TB intermittently stop and then resume the attack when running towards an insect.
Reason 1: TB cannot maintain the pace and must pause for a rest.
Reason 2: TB is unable to process the rapidly changing visual information and go blind, then stop.
Support one weaken another
Answer:C
A.Irrelevant
B.前半句证明不是blind,后半句证明不是为了rest
C.Correct 前半句削弱blind的原因,后半句证明是为了rest
D.Irrelevant
E.只是证明了不是blind,没支持是为了rest
作者: FB小贝 时间: 2012-5-4 18:54
6.【精练】
P:human beings have cognitive faculties
C : cannot be happy if not involve gratification of these faculties
W:can be happy when not.
1、无关 2、correct 3、无关 4、无关 5、无关
【逻辑链】
1.P:cultivated emmer found in E&A, wild emmer found in sA, cultivated emmer found are in the same narrow strip
C:emmer wheat was first domesticated in that strip
Strengthen:not domesticated in E
1、无关 2、正确,增加可能性 3、无关 4、correct 5、无关
2、P:concurrent sharp decline in populations of seals and sea lions is believed to have been caused by diseases that are likely to be able to infect sea otters also
C: the decline in 1980s in NP is caused by disease rather than predation
Weaken: the diseases would not infect sea otters / is caused by predation
1、正确 2、无关 3、无关 4、无关 5、无关
3、C: aluminum recycling more widely practiced than glass recycling, the weight of glass bottles in hg declined by a greater percentage
Account for : the weight of glass bottles is relatively easier to be reduced
1、不充分 2、无关 3、正确 4、不充分 5、无关
4、P: TB intermittently stop when running toward an insect
a. pause for a rest
b. go blind and stop
1、Not blind , not rest
2、Not blind, rest
3、irrelevant correct
4、 irrelevant 5、irrelevant
作者: iaoinging 时间: 2012-5-9 15:34
第一篇:1, 28s
2, B: HB have cognitive faculties that are superior to those of other animals
C: HB cannot be made happy by anything that does not involve gratification of these faculties once HB become aware of these.
3, How HB become aware of these?
4, B
A, 因为是讲人,我觉得更动物没有关系。
B,只有这个通过说明认知感与喜好无关系。
C, 这也应该是支持文章的
D, 这也应该是支持文章的
E,这应该是支持文章的。
第二篇:1, 39s
2, B: EW 在EUROPE 和ASIA都有,但是THE WILD FORM OF EW 的唯一发现地点是 Southwest Asia的一些小部落。
C: emmer wheat was first domesticated somewhere in that strip.
3, 支持文中论据的话,也就是要证明EW这种东西是从这些小部落里传出去的,或者是别的人进入了这些小部落学了他们的方法。也就是要说明这些小部落是适合种植EW的。
4,D
A,B,C都是无关的
D应该是说明了气候适宜
E, 能不能区别应该也是没有关系的吧。
第三篇:1, 37s
2, B:late 1980s, the population of sea otters in the North Pacific began to decline. 2 possible reasons for this situation: predation or disease.
 
: concurrent sharp decline in populations of seals and sea lions is because the disease, and diseases are likely to be able to infect sea otters
C: its decline may be caused by disease.
3, if it is really caused by disease, then the Killer whales will also had decrease.
4, A
A: 成功解释了SO减少的可能的原因
B:支持文章
c, D, E: 无关选项
第四篇:1, 47s
2, B: 1978-1988, beverage containers 下降 of the total weight of household garbage in the U.S. 大部分是因为recycling aluminum 和 glass的大量使用 .RA用的比RG更广泛. 但是, the weight of glass bottles in household garbage declined by a greater percentage than the weight of aluminum cans.
3, 出现这样的原因应该就是RG比RA要重吧
4, A
BCDE: 都对题目没影响 不懂!!!
第五篇: 1, 46s
2, TB 速度很快,基本可以抓住任何一个NONflying insect. 但他们抓insect途中,会停下来之后再攻击. 有两个解释: 第一个,TB需要休息,第二个: TB在快速奔跑时看不见东西,必须停下来确定INSECT的方位.
3, 题目问支持一个,削弱一个的话, 就找两个的不同点. 如果是休息的话, 那么证明TB是没有看不见insect的 .如果是因为 看不见的话, 那么TB是不需要休息的.
4, B
A: 我觉得A 貌似也是滴的.但 还是B比较像. 他是支持了一个,但没有undermine. E也是这样的
B: 就支持第一,然后undermine第二个咯.
CD:无关
作者: spencerX 时间: 2012-5-15 11:11
精练(50s)
Premise:
Human beings have cognitive faculties that are superior to those of other animals
Conclusion:
Once humans realize that, they cannot be happy by anything doesn’t involve gratification.
Pre:
There are some activities don’t involve cognitive faculties can make humans happy.
My answer: B
A. The object of the argument is human, not animals, so this choice is irrelevant.
B. If they enjoy the physical activities, then the argument will be weakened.
C. Irrelevant
D.?
E. Irrelevant.
1. Cognitive means “relating to the mental process of knowing, including reasoning and judgment.”
2. Answer choice (D): 选项D错误的原因是因为它建立在了一个隐含的前提:这些athletes在运动的过程中没有用到cognitive faculties。(在选项B中,清晰地写出了“familiar with”),而结论的范围是“gratification of these faculties”。所以选项D是错误的。
要注意结论的特殊性,以及选项中是否有隐含的前提,是不是直接和argument相关。
逻辑链:
1.
Background:
One kind of wheat has been found in certain place.
Premise:
The oldest remains of cultivated emmer wheat yet found are from village sites in the same narrow strip.
Conclusion:
Emmer wheat was first domesticated somewhere in that strip.
Pre:
We cannot find any other wheat older than we find that narrow strip.
My answer: D
A.irrelevant
B.irrelevant
C.irrelevant
D.contender
E.support
用排除法做的,对于D不是很清楚,准备再想一下,然后看一下NN的讨论…
2.
Background:
population of sea otters↓, there are two reasons may account for that:1)predation;2)disease
Premise:
sharp decline of seals and sea lions—disease—this disease can infect sea otter as well.
Conclusion:
Disease--- more plausible
Pre:
The analogy between sea otters and seals or sea lions cannot be justified.
My answer: E
A.irrelevant
B.support the argument
C.irrelevant
D.irrelevant
E.?
这道题我选A是因为我觉得前面都不对啊,虽然说whale会吃其他的猎物,但是也没有会说是sea otters啊,而且说seals和sea lions下降的原因是因为disease,whale是不是吃它们有什么关系呢?
3.
Fact: aluminum weight percentage < glass weight percentage; BUT: aluminum recycle> glass recycle
pre:
There is a difference between total number and percentage.
My Answer: A (又错了,今天是肿么了…哭)
4.
Background:
tiger beetles—run fast
When running toward an insect, the beetles intermittently stop, and then, a moment later, resume their attack.
possible hypothesis:
1) cannot maintain their pace and must pause for a moment's rest
2) unable to process the resulting rapidly changing visual information, and so quickly go blind and stop
Conclusion:
One hypothesis is right, while another is wrong.
My answer:C
A.irrelevant
B.this answer actually support both the first hypothesis and the second one.
C.?
D.irrelevant
E.this answer concerns with the second hypothesis, but is irrelevant to the first one, so it cannot support one and , at the same time, weaken the first one.
⊙﹏⊙b汗,还是用排除法作出来的…这次的练习要好好研究一下
作者: 卖红薯a 时间: 2012-5-18 20:10
1.
1)计时:20s
2)逻辑链Situation
Background information: Humans have the best cognitive faculties.
Premise: once humans know these
Conclusion: They can only be made happy by things that involve gratification of these faculties.
3)推测: People need to do things after thinking.
4)选项分析:选B
A. Opposite.
B. Right.
C. Irrelevant. not mention the non-gratification of cognitive faculty
D. Irrelevant.
E. Irrelevant.
21.
1)计时:70s
2)逻辑链Situation
Background information: Cultivated EW has been found among the earliest agricultural remains in Europe and Asia. Wild EW has only been found in a narrow strip of southwest Asia.
Premise: the oldest remains cultivated EW is from the same narrow strip
Conclusion: so EW was first domesticated in that strip
3)推测: since the wild EW has only been found in that strip and the oldest cultivated EW is from that strip also, people tried to grow EW in the same area
4)选项分析:选B
A. Irrelevant. about another wild wheat: einkorn
B. Right.
C. Irrelevant. about EW’s nutrition
D. Irrelevant.
E. Irrelevant. distinguish wild EW and a closely related wild wheat
22.
1)计时:47s
2)逻辑链Situation
Background information: the population of sea otters is declining↓, 2 plausible reasons: 1) predators 2) disease
Premise: disease is the most possible reason: seals and sea lions’ population is declining because of diseases
Conclusion: sea otters can be infected by those diseases too
3)推测: predator can be infected too, but their population didn’t decline;
maybe other reasons like: low natality, etc
4)选项分析:选B
A. Irrelevant.
B. Right.
C. Support.
D. Irrelevant.
E. Irrelevant.
23.
1)计时:70s
2)逻辑链Situation
Background information: beverage container’s weight↓---? garbage’s total weight↓
Premise: widespread practice of recycling A and G---? ↓most of the decine
Conclusion: though the A recycling was wider than G recycling, G weight has a greater percentage of decrease than A weight
3)推测: G bottle is heavier than A container in same size, so the conclusion is fair.
4)选项分析:选A
A. Right.
B. Irrelevant. we are talking about the G weight, not the products in G bottles
C. Irrelevant.
D. Irrelevant. about purchasing speed
E. Irrelevant. about sorting
24.
1)计时:50s
2)逻辑链Situation
Background information: TB run fast, and it can catch any nonflying insect.
Premise: TB needs a moment stop before attack their food.
Conclusion: assumptions: 1) TB can’t maintain the pace and need a moment’s rest 2) TB cant’ process rapid visual changes, and so go blind and stop.
3)推测: Yes. The two assumptions are indicating two alternative theses: maintaining and visual ability.
4)选项分析:选A
A. Right.
B. Irrelevant. not moving insect, is moving prey insect
C. Irrelevant.
D. Irrelevant. ?
E. Irrelevant. ?
作者: baseboss 时间: 2012-5-19 21:52
1.B:Human being has the cognitive faculties better than other animals.
P:When human being know this.
C:They will not be happy by anything that not involve the satisfaction of the faculties.
presume:
1.Even human know that they are better than other animals,they will also be happy.
选D B
2 In euro and asia can found the remains of cultivated emmer wheat.
The wild form was found in the special place,where also found the remains of cultivated emmer wheat from village site.So domesticated wheat is from that special place.
presume:
1.From DNA or anyother ways has improved the oldest cultivated emmer wheat has the relationship with the wild wheat.
c D
3.B Root cause of the population of sea otter decline
P: Concurrent the population of sea lion and seal is due to disease.
C
isease will also effect sea otter,which will be the root cause.
presume:
1.Popuation of seal and sea lion decline,the natual enemy for both three will eat more sea otter than before.
A
4.Al and glass were decline in the household garbage.Although recycling AL is more widely the recycling glass,the weight of glass decline more than that of AL cans.
presume:
glass bottle can be cleaned to use again,al cans can use only once.
beverage company will allow people use glass bottle to exchange for the new beverage.
c
5 TB is fastest runner to capture any nonflying insects.
TB need have a rest instead of continuous running.
1.TB can not maintain the pace
2.TB will be bind under the rapid speed.
presume
To define the longest distance that TB can continuos running.Then put the insect in that distance to check the result.
B C
作者: Threesu 时间: 2012-5-25 09:34
B:Human beings have a superior cognitive system than other animals.
P:If Human aware of these
C:They would not be happy about anything that not involve gratification of these faculties
(A) Certain animals—dolphins and chimpanzees,
for example—appear to be capable of
rational communication.---Irrelevant
(B) Many people familiar both with intellectual
stimulation and with physical pleasures
enjoy the latter more.---support
(C) Someone who never experienced classical
music as a child will usually prefer popular
music as an adult.------support
(D) Many people who are serious athletes consider
themselves to be happy.------Weaken
(E) Many people who are serious athletes love
gourmet food.------Irrelevant
做错了,是B
B里面对比的是cognitive和physical之间的问题,而在D里面对比的是cognitive 和cognitive
B:traces of emmer wheat have be found in ancient Asia and Europe
P:Emmer wheat are found in narrow strips both in Asia and Europe
C:Emmer wheat cultivated in strips.
(A) The presentday distribution of another wild wheat, einkorn, which was also domesticated early in the development of agriculture, covers a much larger area of southwest Asia.---irrelevant
(B) Modern experiments show that wild emmer wheat can easily be domesticated so as to yield nearly as well as traditionally domestic strains.---irrelevant
(C) At the time when emmer wheat was first cultivated, it was the most nutritious of all the varieties of grain that were then cultivated.---irrelevant
(D) In the region containing the strip where wild emmer wheat has been found, climatic conditions have changed very little since before the development of agriculture.---support
(E) It is very difficult, without genetic testing, to differentiate the wild form of emmer wheat from a closely related wild wheat that also grows in southwest Asia.---weaken
B:The decrease of sea otters may because either predator or disease
P:the decrease of sea lions and seals is because of disease, which is likely to infect sea otters
C:The reason why the number of sea otters decreases is disease
(A) Killer whales in the North Pacific usually prey on seals and sea lions but will, when this food source is scarce, seek out other prey.---weaken
(B) There is no indication that the sea otter population at any North Pacific location declined in the 1980s because of substantial numbers of sea otters migrating to other locations.---support
(C) Along the Pacific coast of North America in the 1980s, sea otters were absent from many locations where they had been relatively common in former times.---irrelevant
(D) Following the decline in the population of the sea otters, there was an increase in the population of sea urchins, which are sea otters' main food source.---irrelevant
(E) The North Pacific populations of seals and sea lions cover a wider geographic area than does the population of sea otters.---irrelevant
B:The beverage container producers are change the material of it to recycling aluminum, and this action accounts for a sharply decrease of total weight of garbage in households.
S:The weight of glass is decrease even more quickly than the aluminum ones.
(A) Glass bottles are significantly heavier than aluminum cans of comparable size.---
(B) Recycled aluminum cans were almost all beverage containers, but a significant fraction of the recycled glass bottles had contained products other than beverages.
(C) Manufacturers replaced many glass bottles, but few aluminum cans, with plastic containers.
(D) The total weight of glass bottles purchased by households increased at a slightly faster rate than the total weight of aluminum cans.
(E) In many areas, glass bottles had to be sorted by color of the glass before being recycled, whereas aluminum cans required no sorting.---Irrelevant
B:tiger beetles are fast enough to catch nonflying insects. However, when they are predating they have to stop a little while before catching insects.
P: The reason is either to keep pace or flash-blind
(A) When a prey insect is moved directly toward a beetle that has been chasing it, the beetle immediately turns and runs away without its usual intermittent stopping.---irrelevant
(B) In pursuing a moving insect, the beetles usually respond immediately to changes in the insect's direction, and pause equally frequently whether the chase is up or down an incline.---contradiction
(C) The beetles maintain a fixed time interval between pauses, although when an insect that had been stationary begins to flee, the beetle increases its speed after its next pause.---
(D) If, when a beetle pauses, it has not gained on the insect it is pursuing, the beetle generally ends its pursuit.---
(E) When an obstacle is suddenly introduced just in front of running beetles, the beetles sometimes stop immediately, but they never respond by running around the barrier.
作者: 二楼往下掉 时间: 2012-6-1 00:38
1.
(1)45s weaken
(2)逻辑链
Primes:
Human beings have cognitive faculties that are superior to those of other animals
Conclusion:
Once humans become aware of these, they cannot be made happy by anything that does not involve gratification of these faculties.
(3)example:human beings can be made happy by something that does not involve cognitive
(4)A:意图攻击primes “ Human beings have cognitive faculties that are superior to those of other animals” , 但是根据选项或尝试都可以判断,攻击不可能实现证明primes是错的,所以loser。
B:很好的证明了human beings can be made happy by something that does not involve cognitive,contender
C:
D:是想说athletes平常做的都是lack of cognitive的事,所以想由此weaken原argument?(是这个意图么?)P.S.这个选项感觉没头没脑的就来这么一个。。。
E:同D吧
所以,B正确
2.
(1)45s strengthen
(2)逻辑链
Primes:
<1>traces of cultivated EW have been found in E and A
<2>wild EW has been found in narrow strip of SA
Conclusion:
Cultivated & wild EW both found in the strip—>EW first domesticated in the strip
(3)在strip发现了种植的方法记载,别的地方的种法都是跟strip相似的
(4)A:类比加强,但是感觉没啥效果。。。
B:正确选项,说明EW easily be domesticated
C:无关
D:无关
E:两种wheat很难分辨,也就是说在SA发现的可能不是EW?从而削弱了。。。?
3.
(1)45 weaken
(2)逻辑链
Background:
<1>In the late 1980s, the population of sea otters in the North Pacific began to decline.
<2>There are two plausible explanations for the decline: predation, possibly by killer whales, or disease.
Primes:
<1>A concurrent sharp decline in populations of seals and sea lions is believed to have been <2>Diseases that infect these creatures are likely to be able to infect sea otters
Conclusion:
Of these two, disease is the more likely
(3)做出一番新的解释
<1>该时期发生了很不利于otter的气候变化,因而不一定是disease导致的decline
<2>Otter体内有抵抗这种disease的成分
(4)A:对otter的数量下降做了一番新的解释,contender
B:减掉一种解释,是对原解释的加强!loser!
C:列举了一个能证明otter确实减少了现象,但是对于argument没有影响,loser
D:阐述了otter数量减少后会带来的效果,对argument没影响,loser
E:将NP这个地方的seal & sea lions的数量与otter的数量进行对比,无关对比,不影响argument,loser
所以,A正确
4.
(1)45s explain
(2)逻辑链
Background:
1978~1988,beverage containers accounted for a steadily decreasing percentage of the total weight of household garbage in the United States在美国饮料包装占总家庭垃圾重量的比例下降了
Conclusion:
Widespread practice of recycling aluminum and glass was responsible for most of this decline
Further:
Recycling:A>glass
Percentage of weigh decline: A>glass
Why?
(3)glass还没等扔就摔坏了
(4)A:相同大小的glass>A in weigh, 以此说明尽管glass回收的少,但是在重量占比上回有很大变化,contender
B:是不是想说A回收的更彻底?loser
C:用塑料代替glass的比代替A的多,所以glass的重量占比下降很快,contender
D:既然买的多,为啥会扔的占比少?loser
E:说明了glass回收相对麻烦一点,但是不能解释为什么glass占比减的比A还快
综上,A只能说明glass会减少,但是不能说明为什么glass的占比减少的比A快
所以,选C
5.
(1)45s
(2)逻辑链
Primes:
<1>Tiger beetles are such fast runners that they can capture virtually any non-flying insect
<2>when running toward an insect, the beetles intermittently stop, and then, a moment later, resume their attack
Hypothesis:
<1> they cannot maintain their pace and must pause for a moment's rest
<2> while running tiger beetles are unable to process the resulting rapidly changing visual information, and so quickly go blind and stop
A:说明B还是能看见的,但是不能support<1>,loser
B:同A,loser
C:B的休息是固定的,而且在看见猎物要逃跑时会加速,contender
D:歇的时候如果没有收获,就放弃,好像没啥关系,loser
E:B能看见,loser
所以,C是对的
作者: emmasy 时间: 2012-6-5 09:11
1.
premise:people have cognitive faculties that are superior to those of other animals.
conclusion:people cannot be happy without gratification of these faculties once people aware of them.
a.irrevelant
b.irrevelant
c.irrevelant
d.support
e.weaken
2.
background:E has been found in Europe and Asia. The only place where wild E has been found is southwest Asia.
premise:the oldest E has been found the same place as the wild E
conclusion:E was first domesticated there
a.irrevelant
b.irrevelant
c.irrevelant
d.support
e.weaken
3.
background: the number of SO declined. And there are two explanations: disease or predation.
premise:S and SL were killed by disease which could also infect SO.
conclusion:the decline was caused by disease.
a,weaken
b.irrevelant
c.irrevelant
d.irrevelant
e.irrevelant
4.
background: because the widespread practice of recycling A and G, the percentage of BC in the household garbage declined.
premise:A recycling was practiced more widely than G recycling
result:the weight of G decline is greater than that of A decline.
a. explain
b.irrevelant
c.weaken
d.irrevelant
e.irrevelant
5.
background: TB runs so fast that it can catch a lot of nonflying insect. When it runs to a insect, it stops and then resume attack.
explanation:1.it cannot maintain its pace and need a rest;2.it is unable to process rapidly changing visual information, so it goes blind and stop.
a.support 2
b.irrevelant
c.irrevelant
d.irrevelant
e.irrevelant
作者: bet 时间: 2012-6-9 00:28
1 29”
P:Human beings have cognitive faculties that are superior to those of other animals.
C
nce humans become aware of these, they cannot be made happy by anything that does not involve gratification of these faculties.
预测:人类因为其他的事情也可以happy
选D 其他选项和人类是否happy无关
2 35”
P: both the wild form of emmer wheat and the oldest remains of cultivated emmer wheat yet found are from village sites in the same narrow strip.
C: emmer wheat was first domesticated somewhere in that strip.
选B
还没做完,没时间了,明天补全~~
作者: jetyxo 时间: 2012-6-11 19:16
1 not gratification with cognitive faculties---> not happy
A C Eirrelevant; D athletes consider themselives may be for other reasons,which contain cognitive progresses
So my answer is B
2 premise : cultivated emmer wheat found----> agricultural remains of many archaeological sites in Europe and Asia
wild emmer wheat found---> a narrow strip of southwest Asia
the oldest cultivated emmer wheat found---> in the same narrow strip
conclusion: emmer wheat was first domesticated somewhere in that strip
support? A B C E irrelevant the answer is D,
3 premise: 1population of sea otter declined----two reasons: predation or disease
2 seals and sea lions also declined because of disease
conclusion: the reason of sea otter decliance is disease bacause affection
weaken? A---> the predation is the reason
4 household garbage declined<--- increasingly widespread practice of recycling aluminum and glass
aluminum recycling was more widely than glass recyciling---- the glass garbage declined more than aluminum garbge
reason? 基数大---->比列小 C replace more glass---> less glass container were thrown
5 A D E irrelevant;B suppots the reason have a rest, but didn't weaken the other reason ---》C
作者: penguinsdz 时间: 2012-6-14 09:33
标题: 6.13+【1-6】
前两天一直在做Verbal review 2nd edition 做了大概80道逻辑题 所以停顿了逻辑小队作业~
6.13+【1-6】
1. Human beings have cognitive faculties that are superior to those of other animals, and once humans become aware of these, they cannot be made happy by anything that does not involve gratification of these faculties.
Which one of the following statements, if true, most calls into question the view above?
(A) Certain animals—dolphins and chimpanzees, for example—appear to be capable of rational communication.
(B) Many people familiar both with intellectual stimulation and with physical pleasures
enjoy the latter more.
(C) Someone who never experienced classical music as a child will usually prefer popular music as an adult.
(D) Many people who are serious athletes consider themselves to be happy.
(E) Many people who are serious athletes love gourmet food.
Background Information:
Human beings have cognitive faculties that are superior to those of other animals.
Premise:
Humans become aware of these.
Conclusion:
They can not be made happy by anything that does not involve gratification of these faculties.
推测(Prephrase: mentally formulate your answer to the question stem):
They can be made happy by something that does not involve gratification of these faculties.
选项分析:选B
2. Traces of cultivated emmer wheat have been found among the earliest agricultural remains of many archaeological sites in Europe and Asia. The only place where the wild form of emmer wheat has been found growing is a relatively narrow strip of southwest Asia. Since the oldest remains of cultivated emmer wheat yet found are from village sites in the same narrow strip, it is clear that emmer wheat was first domesticated somewhere in that strip.
Which of the following, if true, most strengthens the argument?
(A) The present day distribution of another wild wheat, einkorn, which was also domesticated early in the development of agriculture, covers a much larger area of southwest Asia.
(B) Modern experiments show that wild emmer wheat can easily be domesticated so as to yield nearly as well as traditionally domestic strains.
(C) At the time when emmer wheat was first cultivated, it was the most nutritious of all the varieties of grain that were then cultivated.
(D) In the region containing the strip where wild emmer wheat has been found, climatic conditions have changed very little since before the development of agriculture.
(E) It is very difficult, without genetic testing, to differentiate the wild form of emmer wheat from a closely related wild wheat that also grows in southwest Asia.
Background Information:
Traces of cultivated emmer wheat have been found among the earliest agricultural remain s of many archaeological sites in Europe and Asia.
Premise:
The oldest remains of cultivated emmer wheat yet found are from village sites in the same narrow strip.
Conclusion:
Emmer wheat was first domesticated somewhere in that strip.
选项分析:选D
3. In the late 1980s, the population of sea otters in the North Pacific began to decline. There are two plausible explanations for the decline: predation, possibly by killer whales, or disease. Of these two, disease is the more likely, since a concurrent sharp decline in populations of seals and sea lions is believed to have been caused by disease, and diseases that infect these creatures are likely to be able to infect sea otters also.
Which of the following, if true, most seriously weakens the reasoning?
(A) Killer whales in the North Pacific usually prey on seals and sea lions but will, when this food source is scarce, seek out other prey.
(B) There is no indication that the sea otter population at any North Pacific location declined in the 1980s because of substantial numbers of sea otters migrating to other locations.
(C) Along the Pacific coast of North America in the 1980s, sea otters were absent from many locations where they had been relatively common in former times.
(D) Following the decline in the population of the sea otters, there was an increase in the population of sea urchins, which are sea otters' main food source.
(E) The North Pacific populations of seals and sea lions cover a wider geographic area than does the population of sea otters.
Background Information:
The population of sea otters in the North Pacific began to decline.
Premise:
A concurrent sharp decline in population of seals and sea lions is believed to have been caused by disease.
Conclusion:
Diseases that infect seals and sea lions are likely to be able to infect sea otters also.
推测(Prephrase: mentally formulate your answer to the question stem):
Killer whales maybe the main reason for the decline.
选项分析:选A
4. From 1978 to 1988, beverage containers accounted for a steadily decreasing percentage of the total weight of household garbage in the United States. The increasingly widespread practice of recycling aluminum and glass was responsible for most of this decline. However, although aluminum recycling was more widely practiced in this period than glass recycling, it was found that the weight of glass bottles in household garbage declined by a greater percentage than the weight of aluminum cans.
Which of the following, if true of the United States in the period 1978 to 1988, most helps to account for the finding?
(A) Glass bottles are significantly heavier than aluminum cans of comparable size.
(B) Recycled aluminum cans were almost all beverage containers, but a significant fraction of the recycled glass bottles had contained products other than beverages.
(C) Manufacturers replaced many glass bottles, but few aluminum cans, with plastic containers.
(D) The total weight of glass bottles purchased by households increased at a slightly faster rate than the total weight of aluminum cans.
(E) In many areas, glass bottles had to be sorted by color of the glass before being recycled, whereas aluminum cans required no sorting.
Background Information:
Beverage containers account for a steadily decreasing percentage of the total weight of household garbage in the US.
Premise:
Aluminum recycling was more widely practiced than glass recycling.
Conclusion:
The weight of glass bottles in household garbage declined by a greater percentage than the weight of aluminum cans.
选项分析:选C
5. Tiger beetles are such fast runners that they can capture virtually any non flying insect. However, when running toward an insect, the beetles intermittently stop, and then, a moment later, resume their attack. Perhaps they cannot maintain their pace and must pause for a moment's rest; but an alternative hypothesis is that while running tiger beetles are unable to process the resulting rapidly changing visual information, and so quickly go blind and stop.
Which of the following, if discovered in experiments using artificially moved prey insects, would support one of the two hypotheses and undermine the other?
(A) When a prey insect is moved directly toward a beetle that has been chasing it, the beetle immediately turns and runs away without its usual intermittent stopping.
(B) In pursuing a moving insect, the beetles usually respond immediately to changes in the insect's direction, and pause equally frequently whether the chase is up or down an incline.
(C) The beetles maintain a fixed time interval between pauses, although when an insect that had been stationary begins to flee, the beetle increases its speed after its next pause.
(D) If, when a beetle pauses, it has not gained on the insect it is pursuing, the beetle generally ends its pursuit.
(E) When an obstacle is suddenly introduced just in front of running beetles, the beetles sometimes stop immediately, but they never respond by running around the barrier.
Background Information:
Tiger beetles are such fast runners that they can capture any non flying insect.
Premise:
The beetles intermittently stop, and then, resume their attack.
Conclusion:
1.They can not maintain their pace and must pause for a rest.
2. They are unable to process the resulting rapidly changing visual information while running.
选项分析:选C
作者: LuckyYolandaLi 时间: 2012-6-20 22:15
1
1)计时:29s
2)逻辑链:
Background information:
Premise: Humans become aware that human beings have cognitive faculties that are superior to those of other animals.
Conclusion: Humans cannot be made happy by anything that does not involve gratification of these faculties.
3)推测:就是觉得这个推理没什么因果关系。。。
4)选项分析:选B
A: The content of this option conflicts with the premise.
B: Correct.
C: Irrelevant.
D: This option doesn’t point out what make them happy.
E: Irrelevant.
2
1)计时:45s
2)逻辑链:
Background information: Traces of cultivated emmer wheat have been found among the earliest agricultural remains of many archaeological sites in Europe and Asia.
Premise: The only place where the wild form of emmer wheat has been found growing and the oldest remains of cultivated emmer wheat found are both from village sites in a relatively narrow strip of southwest Asia.
Conclusion: Emmer wheat was first domesticated somewhere in the narrow strip.
3)推测:The first domesticated emmer wheat has some remains.
4)选项分析:选D
A: 也有点weaken的感觉,不是太确定,犹豫了,刚开始想选这个。。。
B: Irrelevant.
C: Irrelevant.
D:Correct.不确定,climate conditions不知道有关无关。。。
E: Weaken.
3
1)计时:41s
2)逻辑链:
Background information: In the late 1980s, the population of sea otters in the North Pacific began to decline.
Premise: A concurrent sharp decline in populations of seals and sea lions is believed to have been caused by disease, and diseases that infect these creatures are likely to be able to infect sea otters also.
Conclusion: Disease is the more likely explanations than predation for the decline of sea otters’ population in the North Pacific.
3)推测:There are no predators of seals or sea lions there, or perhaps that sea otters are not as inclined to get infected as seals and sea lions.
4)选项分析:选A
A: Correct.
B: This option negates the premise.
C: This option just restates the background information.
D: Irrelevant. Whether the conclusion can be drawn or not, the statement in this option can be set up.
E: Irrelevant.
4
1)计时:51s
2)逻辑链:
Background information: From 1978 to 1988, beverage containers accounted for a steadily decreasing percentage of the total weight of household garbage in the United States.The increasingly widespread practice of recycling aluminum and glass was responsible for most of this decline.
Premise: Aluminum recycling was more widely practiced in this period than glass recycling.
Conclusion: The weight of glass bottles in household garbage declined by a greater percentage than the weight of aluminum cans.
3)推测:People use much more aluminum cans than glass bottles.
4)选项分析:选C
5
1)计时:77s
2)逻辑链:
Background information: Tiger beetles are such fast runners that they can capture virtually any nonflying insect.
Premise: when running toward an insect, the beetles intermittently stop, and then, a moment later, resume their attack
Conclusion: There are two hypotheses of the phenomenon.
3)推测:
4)选项分析:选D(正确答案C)
A: Undermine both.
B: Undermine both.
C: Undermine the latter.(看完答案,大概明白Support the former, 因为beetles间隔固定时间就要休息)
D: Correct. Undermine the former and support the latter.(这个不太明白。。。)
E: Support the latter. 但后半句没明白。。。
作者: angelafeng 时间: 2012-6-23 12:03
Premise:Human beings have cognitive faculties that are superior to those of other animals,once humans become aware of these.
Conclusion:they cannot be made happy by anything that does not involve gratification of these faculties.
Prephrase:(weaken)they can be made happy by something that does not involve gratification of these faculyies.
(A) Certain animals—dolphins and chimpanzees,
for example—appear to be capable of
rational communication.--------------------the argument do not mention certain animals
(B) Many people familiar both with intellectual
stimulation and with physical pleasures
enjoy the latter more.----------------------更喜欢不能说明就不喜欢前者,因此不能削弱----------correct,intellectual stimulation=cognitive faculties,physical pleasures≠cognitive faculties,同时具有这两个能力且更喜欢PP,说明人们可以获得快乐不通过CF而通过PP
(C) Someone who never experienced classical
music as a child will usually prefer popular
music as an adult.---------------------------从未接触过classical music说明这人不认知music,但其长大后你仍然喜欢音乐就削弱了the conclusion----------------correct--------------since no suggestion is made that individuals can be made happy without gratification of the cognitive faculties, this
answer is incorrect.
(D) Many people who are serious athletes consider
themselves to be happy.-------------------support the conclusion
(E) Many people who are serious athletes love
gourmet food.-------------------------------喜欢另外一种事物不能说明就不喜欢具有很强认知能力的事情
没有理解什么是cognitive faculties!!!
21.(28551-!-item-!-188;#058&003690)
Background information: Traces of cultivated emmer wheat have been found among the earliest agricultural remains of many archaeological sites in Europe and Asia.The only place where the wild form of emmer wheat has been found growing is a relatively narrow strip of southwest Asia.
Premise: Since the oldest remains of cultivated emmer wheat yet found are from village sites in the same narrow strip,
Conclusion: it is clear that emmer wheat was first domesticated somewhere in that strip.
Prephrase: (strengthen) 野生的EW可以在那个时期家养。
Which of the following, if true, most strengthens the argument?
(A) The presentday distribution of another wild wheat, einkorn, which was also domesticated early in the development of agriculture, covers a much larger area of southwest Asia.
(B) Modern experiments show that wild emmer wheat can easily be domesticated so as to yield nearly as well as traditionally domestic strains.
(C) At the time when emmer wheat was first cultivated, it was the most nutritious of all the varieties of grain that were then cultivated.
(D) In the region containing the strip where wild emmer wheat has been found, climatic conditions have changed very little since before the development of agriculture.
(E) It is very difficult, without genetic testing, to differentiate the wild form of emmer wheat from a closely related wild wheat that also grows in southwest Asia.
22.(28829-!-item-!-188;#058&003748)
27s
Background information:In the late 1980s, the population of sea otters in the North Pacific began to decline.There are two plausible explanations for the decline:predation, possibly by killer whales, or disease.
Premise:since a concurrent sharp decline in populations of seals and sea lions is believed to have been caused by disease, and diseases that infect these creatures are likely to be able to infect sea otters also.
Conclusion:Of these two, disease is the more likely
prephrase:(weaken)populations of seals and sea lions decline,then the predator of them have little food to eat,so they will have to eat sea otters.
23.(29107-!-item-!-188;#058&003854)
32s
background information: From 1978 to 1988, beverage containers accounted for a steadily decreasing percentage of the total weight of household garbage in the United States. The increasingly widespread practice of recycling aluminum and glass was responsible for most of this decline.
Premise: although aluminum recycling was more widely practiced in this period than glass recycling
Conclusion: it was found that the weight of glass bottles in household garbage declined by a greater percentage than the weight of aluminum cans.
prephrase: (explain) 因为玻璃瓶子的单位重量比铝的易拉罐重
24.(29155-!-item-!-188;#058&003857)
46S
Background information:Tiger beetles are such fast runners that they can capture virtually any nonflying insect.However, when running toward an insect, the beetles intermittently stop, and then, a moment later, resume their attack.
Premise1:Perhaps they cannot maintain their pace and must pause for a moment's rest;
Premise2:but an alternative hypothesis is that while running tiger beetles are unable to process the resulting rapidly changing visual information, and so quickly go blind and stop.
prephrase: (support)
(A) When a prey insect is moved directly toward a beetle that has been chasing it, the beetle immediately turns and runs away without its usual intermittent stopping.
(B) In pursuing a moving insect, the beetles usually respond immediately to changes in the insect's direction, and pause equally frequently whether the chase is up or down an incline.
(C) The beetles maintain a fixed time interval between pauses, although when an insect that had been stationary begins to flee, the beetle increases its speed after its next pause.
(D) If, when a beetle pauses, it has not gained on the insect it is pursuing, the beetle generally ends its pursuit.
(E) When an obstacle is suddenly introduced just in front of running beetles, the beetles sometimes stop immediately, but they never respond by running around the barrier.
C中说beetles的停顿有固定的时间间隔且在停顿后加速,说明beetles不是因为速度不够要休息。反驳了premise1,肯定了premise2.
B只肯定了premise2,但没有说premise1问什么不对。
作者: jiajiajudy 时间: 2012-6-24 03:39
今日总结:读题太重要了,往往一个单词之差,万万不可急躁
1.
BG: 人have cognitive faculties > animals
Conclusion: 人aware of these? not happy by anything that does not involve gratification of these faculties
(A) Irrelevant
(B) Many people familiar both with intellectual stimulation and with physical pleasures enjoy the latter more.
(C) Irrelevant
(D) Irrelevant
(E) Irrelevant
2.
BG: EW found in many archaeological sites in EU and asia
Premise: only place to fine wild form relatively narrow strip of southwest Aisa
Conclusion: The oldest remains EW are found from village sites in the same narrow strip? first domesticated in that strip
(A) irrelevant
(B) irrelevant
(C) At the time when emmer wheat was first cultivated, it was the most nutritious of all the varieties of grain that were then cultivated.
(D) irrelevant
(E) weaken
3.
Predation decline of sea otters
Infection
(A) irrelevant
(B) There is no indication that the sea otter population at any North Pacific location declined in the 1980s because of substantial numbers of sea otters migrating to other locations.
(C) strength
(D) irrelevant
(E) irrelevant
4.
BG: beverage containers decreasing percentage of the total weight of garbage
Premise: al and glass was responsible for this decline
Counter premise: al widely practiced than glass recycling
Conclusion: weight of glass 下降 percentage> aluminum
(A) irrelevant
(B) Recycled aluminum cans were almost all beverage containers, but a significant fraction of the recycled glass bottles had contained products other than beverages.
(C) irrelevant
(D) irrelevant
(E) irrelevant
5.
BG: TB faster, capture any nonflying insects
Counter premise: toward, pause, the resume
Can’t maintain their pace
Can’t changing visual imformation? blind and stop
(C) The beetles maintain a fixed time interval between pauses, although when an insect that had been stationary begins to flee, the beetle increases its speed after its next pause.
作者: 呵呵牙 时间: 2012-6-24 23:11
1. 28s
Premise: Humans have cognitive faculties that are superior to those of other animals.
Conclusion: Once humans become aware of faculties, they cannot be made happy by anything that does not involve gratification of these faculties.
Prephrase: Humans can be happy because they are superior to other animals
选D ———— it talks about individual, but the view above talks the humans.
A (certain animals)——irrelevant
B (enjoy physical pleasures)——irrelevant ————————correct
C (child & adult)——irrelevant
E (gourmet food)——irrelevant
2. 36s
Background Information: Cultivated emmer wheat have been found in Europe and Asia.
Premise: The only place where the wild form of emmer wheat has been found is a narrow strip of southwest Asia. The oldest remains of cultivated emmer wheat yet found are from village sites in the same narrow strip
Conclusion: The emmer wheat was first domesticated somewhere in that strip.
选D
3. 36s
Background Information: In the late 1980s, the population of sea otters in the North Pacific began to decline.
Premise: There are two explanations for the decline: killer whales or disease.
Conclusion: Diseases is more likely because the decline of seals and sea lions are caused by disease.
Prephrase: Killer whales will attack sea otters.
选A
4. 42s
Background Information: From 1978 to 1988, beverage containers accounted for a steadily decreasing percentage of the total weight of family garbage in the US. The practice of recycling aluminum and glass was responsible for most of this decline.
Premise: Aluminum recycling was more widely practiced in this period than glass recycling.
Conclusion: The weight of glass bottles in household garbage declined by a greater percentage than the weight of aluminum cans.
Prephrase: Glass bottles can bigger volume than Aluminum cans
选C
5. 40s
Background Information: Tigers beetles are such fast runners that they can capture insects.
Premise: The beetles stop and then, a moment later, resume their attack.
Conclusion: Maybe they cannot maintain their pace and must pause for a moment’s rest; running tiger beetles are unable to process the resulting rapidly changing visual information, and so quickly go blind and stop.
选C
作者: cleotina 时间: 2012-6-29 07:16
4''
weaken
B: people have higher cognitive feculties than any other animals.
P: once people aware of this, they cannot be happy by anything that does not involve gratificate of these faculties
guess: If other things except this will forcefully fight against this argument
OA: B physical pleasure is not belong to feculties
others: I
44''
B: trace the cultivation of emmer wheat
p: the early wild wheat found in a narrow strip of Asia, the early cultivation of this kind wheat also found in this place
C: the wheat native to that place
Guess: any information similar to the arguments will strenghen
OA: D ? not sure
others:I
41''
weaken
B: In one place the population of sea otter decline, shark , disaese?
P: it should be desease, since the populations of seals and lions have declined. and this desease also infect these creatures are likly be to otters
Guess:
1 this disease is not fatal for otters that infected
2 as the decrease of other animals, the shark have to hunt more otters for food
OA: A similar to the term 2 above
B: strong
others: Irrelevant
46''
supporting
B: beverage container accounted for the total mount of gabage in American family declined. American pratice of recycly glass and aluminum.
question: aluminum recycle was more widely than glass, however, the weight
of glass decline by a greater percentage in the garbage than the weight on
the aluminum cans
guess:
1 since recycle glass is laborable than alumium, people use less glass cans than before
2 per glass can is weight more than aluminum
OA: A
B: irrelevant
C: support, but not as good as A
D: weaken
E: Irrelevant
(细想一下应该是C更优)
1'03
support one and weaken another
B: Beetles run so fast that they can pry any nonflying insect, an specific phenominon is that they have to pause a while to resume their attack
P:1 they have to pause to maintain their pace
2 they are blind when run quickly. have to stop recover
Guess:
1 if they can maintain their pace, they will not blind even run fastly
2 they will become blind before they ran too fast to lose pace
OA : C
作者: Donts 时间: 2012-7-14 11:09
1.计时36''
premise:human has cognitive faculties that animals do not.
conclusion: once humans realize this, they can not be made happy by anything without the gratification of those faculties.
hidden assumption: humans will be happy if they have cognitive faculties that animals do not have.
推测:削弱就是去攻击这个assumption,比如他们还有别的东西可以令他们高兴
(A) Certain animals—dolphins and chimpanzees,
for example—appear to be capable of
rational communication.
(B) Many people familiar both with intellectual
stimulation and with physical pleasures
enjoy the latter more.
(C) Someone who never experienced classical
music as a child will usually prefer popular
music as an adult.
(D) Many people who are serious athletes consider
themselves to be happy.
(E) Many people who are serious athletes love
gourmet food.
选择B
总结:做conclusion还要更准确更到位。Answer choice (D): This can be an attractive answer at first, but it depends on the assumption that the serious athletes are happy due to their athletic endeavors. However, that connection is not explicitly stated,
and it could be that the serious athletes are happy because of some gratification of their cognitive faculties,
in their respective sport or otherwise.
作者: showysimple 时间: 2012-7-25 15:39
1.
[Premise] - human are superior to other animals because of better cognitive faculties.
[Conclusion] - human won't feel happy if CF wasn't satisfied
求weaken
A - irrelevant - challenge premise but not conclusion
B - weaken, right answer
C - incorrect - support the premise but nothing to do with the conclusion. 虽然preference of music与CF有关,但选项中未提及不满足CF也能带来快乐。
D - not direct - 未直接阐明快乐的来源是否与CF有关
E - strengthen - loving gourmet food与CF有关
21. 做错,错误选项B
[Premise] Traces of CULTIVATED E wheat was found in Eur&Asia. The oldest remains of CULTIVATED E wheat is in a strip of SW Asia where WILD E wheat is still growing.
[Conclusion] - E wheat was first domesticated in that strip.
求 strengthen
A - irrelevant - 'another wild wheat' 与论点无关
B - weaken/wrong - so emmer wheat can be grown easily in many OTHER areas, this is to weaken the statement, wrong C - irrelevant
D - strengthen, right answer
E - weaken - challenge to the premise that wild form of E wheat is still growing in SW Asia.
22. 做错,错误选项B
[Premise] - 2 reasons for declining number of sea otters - by killer whales or disease.
[Conclusion] - disease is more likely because other sea mammals' population dropped by disease which can infect sea otters.
求weaken
A - weaken/right answer - 鲸鱼吃海豹海狮,但海豹海狮生病死了,鲸鱼没东西吃就要吃海濑
B -
C - irrelevant - seems support to premise that otters' population declined but didn't explain it declined.
D - irrelevant
E - irrelevant
23. 做错,错误选项B
[Premise] - beverage containers dropped in the total weight in household garbage because of recycle program. aluminum recycling was wider practiced.
[Conclusion] - glass bottles declined greater than aluminum can
求 strengthen
A - strengthen/right answer
B - weaken/in-direct
C - 'plastic container' is irrelevant
D - 'fast rate' is irrelevant,混淆概念
E - irrelevant
24
[Premise] - TB is faster runner. BUT it stops and attacks later when running toward an insect.
[Conclusion] - two reasons, a) for a moment's rest or b) processing changing visual information
求 one reason support one but undermine another
A - support b)
B - support both
C - right answer
D - irrelevant
E - irrelevant
作者: TICKCOCK 时间: 2012-8-5 18:40
逻辑小分队8.5+【1-6】
1、 Background: human beings have cognitive faculties that are superior to those of other animals.
Premise: once humans find this fact
Conclusion: They can not be happy by things does not involve gratification of these faculties.
Prophase: they can be happy by such things.
Answer: B
2、 Background: the cultivated emmer wheat have been found among the earliest of agricultural remains in Eu and ASIA. The only wild form has been found is from Southwest Asia.
Premise: the oldest remains of cultivated emmer wheat yet found are from village sites there.
Conclusion: emmer wheat was first domesticated in that strip.
Answer: D
3、 Background: the population of sea otters in the NP began to decline. And there are two alternatives: Predation and disease.
Premise: since population of seals and sea lions is believed to have been caused by disease.
Conclusion: Disease is the more probable factor that cause otter’s population decline.
Answer: E
4、 from 1978 to 1988, beverage containers accounted for a steadily decrease percentage of total weight of household garbage in the US.
However, although aluminum recycling was more widely practiced in this practice, the weight of glass bottles in the household garbage declined by a comparable greater percentage.
Answer: C
5、 Tigers beetles may stop intermittently when they catch nonflying insects. There are two hypothesis for this. First, they can’t maintain their pace and must rest. Second, they can’t process the changing visual information and go blind and stop.
Answer: E
作者: wanggang0411 时间: 2012-8-13 17:26
1 29
Conclusion: Once people aware faculties, people cannot feel happy by anything that does not involve gratification of these faculties.
To weaken this conclusion: Once people aware faculties, people can feel happy without gratification.
2 34
Premise: WE found in Europe and Asia.
Premise: The only place where the wild WE found in the narrow strip of SW Asia.
Conclusion: Since the oldest cultivated EW found in the same strip, so EW is first domesticated in that Strip.
Strengthen: Stronger and Defender
Since the climatic changed little during the development of agriculture, option D strengthens the conclusion weakly.
3 36
Premise: sea outer began to decline.
Premise: explanation A is predation.
Premise: explanation B is disease.
Conclusion: Since other marine creatures caused by these disease, sea outer also impacted by these disease.
Weaken: 1) There is no such evidence that disease impact the population of sea outer sharply.
2) Any possibilities break the connection between the disease which impacts other marine creatures and the population decline of sea outer would be the correct answer.
4 55
Fact set: Aluminum recycling was more practiced than glass recycling.
Fact set: The weight of glass bottles declined in a greater percentage than that of aluminum.
Paradox resolving: An option explains both sides of the argument would be a correct answer.
There is a very important word, “Percentage”.
5 41
Premise: hypotheses one, TB cannot keep the pace, so stop.
Premise: hypotheses two, TB cannot process visual information rapidly, so stop.
Stem: An option which weakens one hypothesis, and strengthens the other one.
(B) In pursuing a moving insect, the beetles usually respond immediately to changes in the insect's direction, (Weaken Visual) and pause equally frequently whether the chase is up or down an incline. (Weaken Pace)
(C) The beetles maintain a fixed time interval between pauses, (Weaken Visual) although when an insect that had been stationary begins to flee, the beetle increases its speed after its next pause.(Support Pace)
作者: wanggang0411 时间: 2012-8-13 17:28
You know what, my performance is quite fluctuating.
Wish it can become more stable in the closed future.
作者: chengzaaaa 时间: 2012-8-23 09:39
1.
1)14”weaken
2)BI:human beings. Cognitive faculties. Superior to other animals.
P:humans become aware of BI
C:no gratification of these faculties, then no happy.
3)other things that have nothing to do with cognitive faculties could make humans happy.
4)B
5)A. something about animal. Nothing to do with humans.
B. Correct. Physical things bring more joy than intellectual thins do.
C. irrelevant.
D. atheletes don’t necessarily mean not intellectual.
E. nothing to do with food.
21.
1)31”support
2)BI: found emmer in Europe and Asia
P: wild form emmer growing, found only in a narrow strip of southwest Asia. oldest remains of cultivated emmer are from near the strip
C: emmer wheat first domesticated around that strip
3)the emmer found in Europe are from Asia
D
22.
1)34”weaken
2)BI: sea otters declined, 2 possible explanations. Predation or disease
P: disease caused seals and sea lions to decline, and this disease is likely to infect sea otters
C: disease is more likely
3)sea otters have immune system for the disease and other two animals don’t
A
23.
1)27”evaluate
2)BI: beverage containers decreasing percentage of total weight of garbage.
P: recycling of al and glass. Recy of al more practiced than glass
C: the decline in glass is more than the decline in al
3)during this period, manufactures replace glass containers with al containers
C
24.
1)38”evaluate
2)BI: TB are fast. Run, stop, resume.
P:
C: 1. Can’t pick up pace and must pause for a rest. 2.changing visual, go blind for a sec.
3)…..?
B C
谢谢lz!!!![](/static/legacy-emoticon/92.gif)
作者: 我心匪席 时间: 2012-8-31 17:35
Time: 18”
Background Information: Humans have superior cognitive faculties.
Premise: Human are aware of this point, and be happy
Conclusion: Only from gratification of these faculties
Prephrase: Attacking necessary condition
A-----irrelevant
B-----irrelevant
C-----irrelevant
D-----support
E----- right
没注意physical 和cognitive 是相对的…
======================================================
Time: 31”
Background Information: Cultivated wheat has been found in Europe and Asia. Wild wheat has been found in narrow strip in Asia.
Premise: Oldest remains of cultivated emmer wheat are found from same strip
Conclusion: Emmer wheat was first domesticated in that strip.
Prephrase: Condition for cultivation
A-----irrelevant
B-----irrelevant
C-----irrelevant
D-----right
E-----irrelevant
======================================================
Time: 30”
Background Information: Predation or disease causes the number of sea otters declining.
Premise: Seals and sea lion are killed by disease, such disease will infect sea otters
Conclusion: Disease
Prephrase: Predation
A-----right
B-----irrelevant
C-----irrelevant
D-----irrelevant
E-----irrelevant
===========================================================
Time: 36”
Background Information: Beverage containers lose weight for the adoption of recycling aluminum and glass.
Premise: A is more widely used
Conclusion: The weight of G declined by a greater percentage than the weight of A
Prephrase: G weights heavier than A
A-----right
B-----weaken
C-----irrelevant
D-----irrelevant
E-----irrelevant
题目没说明两者数量关系,所以A不能
==========================================================
Time: 31”
Background Information: when TB captures insect, it will intermittently stop and then resume attack.
Premise: Cannot maintain pace or go blind and stop
Conclusion: support one and undermine the other
A-----both weaken
B-----both weaken
C-----right
D-----irrelevant
E-----irrelevant
谢谢lz~
作者: ElenW 时间: 2012-10-5 10:01
1.(17'2)
Bg:Humanbeings have cognitive faculties.
pre:humans become aware of these
con:they can't be made happy by other things
that does't involve gratification of these faculties.
(weaken)
Pre:A->B IF A happened,then B would happen
I need to attack that even if A happens, B stil has not
any change.Or,the A is not the only reason.
Ans:B
I think A.C.D.E are irrelevant choices.
逻辑链
21(35's)
Bg:Traces of cultivated emmer wheat have been found among the earliest agricultural remains of many archaeological sites in Europe and Asia.
pre:the oldest remains of cultivated E.W yet found are form village sites in the same narrow strip
con:E.W. was first domesticated somwhere in that strip.
(strengthen)
Pre:Although the oldest E.W. was found in the same narrow strip,it's not enough.If I was the aracheologist,I would say why the result is correct becouse
the other factors which have influence on the result such as climatic or wet have little changes.
Ans![](static/image/smiley/default/biggrin.gif)
A:irrelevant.
B is not the point .The stimulus did not discuss the temporary way of cultivate E.W.
C:irrevelvant
E:weaken
22(33'1s)
Bg:In the late 1980s, the population of sea otters in the North Pacific began to decline.
pre:Two reasons:predation(killer whales) & disease
Disease is the more likely.
con:diseases that infect these creatures(seals & sea lion) are likely to be able to infect sea otters also
(weaken)
Pre:This is false.Sea otters may also be attacked by disease. But,if the people investigate the sea condition, finding that the sea environment is well,and
survey a sampile they will find that the sea otters are not infected by disease.
Ans:A
B:strengthen
C:irrelevant
D:contradiction
E:irrelevant.
今天起来晚了 先做完了这几个......
作者: srafcatt 时间: 2012-10-6 13:13
精炼 43s weaken
BG:human cognitive faculties(HC)>those of other animals(TA)
premise:if human know HC and HC is not satisfied
conclusion:they will not happy
set an counter example
choice:B
A It has nothing to do with animal's cognitive faculties,the topic is about human.
B this means even if HC is not satisfied they will still be happy.this is correct.
C someone is a special case,not convincing
D maybe they are satisfied with HC,so they are happy.
E that does not necessarily mean that their cognitive faculties are not satisfied.
逻辑链
1.22s support
premise:because both the remains and the cultivating wild E wheat are found in certain narrow strip
conclusion:the earliest E wheat is cultivated there.
no earlier E wheat has been found
choice![](static/image/smiley/default/biggrin.gif)
A weaken
B irrelevant
C irrelevant
D support this choice emphasizes that this program of cultivating E wheat is practical
E weaken
2.41s weaken
BG
tter---decline, reason1:predation or disease, reason2:more likely disease
premise:sea lions and seals decline due to disease
conclusion:reason2:more likely disease
otters are not living together with lions or seals.
choice:A
A strenghten reason 1
B irrelevant information
C repeat the question stem in a different way
D irrelevant information
E strenghten reason 2
3.37s paradox
one situation: the recycling of cans are more practical than that of glass bottle
the opposite phenomenence:the weight of glass bottles declined by greater percentage
maybe glass bottle's total amount is low.
choice:C
A irrelevant information
B irrelevant
C correct
D strenghten the conflict
E irrelevant
4.58s two-speaker stimulus
hypothesis1;Tiger beetles need to pause to have a rest
hypothesis2:tiger beetles need time to see things
the key point is pause for rest not seeing things or the opposite
choice:C
A weaken both,because the prey runs to it is just the same as it runs to the prey,and
B weaken both,if the beetles cannot see,it should pause more frequently when down,if it needs some rest it should pause less frequently when down
C support 1 weaken 2
D weaken both
E irrelevant information
作者: hanhan1991 时间: 2012-11-19 15:59
1.weaken
1)time:27s/1''09
2)logical lines:
Premise:Human have cognitive faculties that are superior to those of other animals.
Conclusion:Once human realise this, they cannot be happy through anything that does not contain grafication of these faculities.
3)Prediction:
Something that does not contain grafication of the cognititive faculties such as sporting can still make people happy.
4)Answer:B
A.other animals are irrelevant to the conclusion, since we only focus on the human beings.
B.people realise the coginitive faculties but still enjoy the thing that does not involve these coginitvie.this weakens the conclusion.
C.this is apparently irrelevant.
D.this does not contain the information whether the athletes realise their superior cognitive faculties.
E.this is apparently irrelevant.
2.strenthen
1)time:54s/2''28(too long)
2)logiacal lines:
Background:Traces of cultivated emmer wheat have been found in many sites in Europe and Asia. The only place where the wild form of emmer wheat has been found growing is a narrow strip of Asia.
Premise:The oldest emmer wheat yet found are from vilage sites in the same narrow strip.
Conclusion:Emmer wheat was frist domesticated in that strip.
3)Predication:
The narrow strip is favorable for emmer wheat cultivation.
****there is a gap between found and first cultivated. It is possible that though the oldest remains of cultivated emmer wheat found in the strip, the wheat was in fact first cultivated in Europe.
4)Answer:B
A.einkorn wheat is irrelevant.
B.If the wild emmer wheat can easily be domesticated, the place where the wild corps growing can be the place where they were first cultivated.
****the choice does not invlove any information about the strip.
C.nutrition is irrelevant.
D.this can be a contender.But whether the weather changed a lot before the devolpment of agriculture seems to be irrelevant.
****if the weather has not changed a lot, the wild wheat found today existed before the devolpment of agriculture.
E.the difficulty of testing is irrelevant.
——————————————————————————————加强题不一定是充分必要条件,但一定要有关。
3.weaken
1)time:39s/1''29
2)logical lines:
Background:The dicline in the number of sea otters can be explained by two reasons--predation or disease.
Premise:The decline in populations of seals and sea lion is due to disease, and the disease can infect sea otters also.
Conclusion
isease is the possible explanation.
3)Prediction
The cause of the dicline in population of sea otters is not disease but preditation. the dicline in populations of seals and sea lions may cause the rise in the number of sea otters predators that seals and sea lions eat.
4)Answer:A
A.the reason is predation but not disease.
B.this opposites the background.
C.this repeated tbe background.
D.the increase in the populations of other animals is irrelevant.
E.the coverage is irrelevant.
4.explain
1)time:40s/2''29(too long)
2)logical lines:
Background: The percentage of household garbage of beverage container declined in 1978-1988 due to the practice of recycing aluminum and glass.
Facts:The aluminum recycing was more widely practiced
Paradox:The weight of glass declined by a greater percentage
3)Prediction:
People use much more aluminum bottles than glass bottles in 1978-1988.
4)Answer:A
A.even though people recycle more aluminum cans, the weitht of glass might declined by a greater percentage.
****this can be a contender, but notice that the conclusion talk about percentage change so the weight is irrelevant.
B.irrelevant,since we only conern about glass bottles.
****this aggravate the paradox
C.maunfacture is irrelevant.
****if maunfacture produced fewer glass bottles, people use fewer glass bottles and the total weight of glass bottles would declined by a greater percentage.
D.this may aggrevates the paradox
E.irrelevant.
5.
1)time:52s/3''50(too long!!)
2)logical lines:
Background:When running for capturing insects, Tiger beeltes interimttenly stop and then resume their attack.
Explanation1:They can not maintian their pace and need to rest for a moment.
Explanation2:They can not process the resulting rapidly changing visual information, and go blind and stop.
3)Predication:
Personally, I think explanation2 is groudless, if the beeltes go blind how can they resume their attack?
4)Answer:B
A.this weakens both the explanaions.
B.this weakens the secondbut strenghen the frist.
***this choice weakens both hypothses, for beetles need longer intervals to stop when the chaises are in upward incline.
C.sounds irrelevant?
***this strenghten the second but weakens the first.
D.irrelevant.
E.not very sure about this answer.
****this support the second but not weakens the first.
作者: CD用户825193 时间: 2012-11-24 18:31
标题: Daily CR-6_2012-11-24
1. 1' !!!(Weak)
P: human's congnitive facuties are superior to other species'
P:
C: human can be made happy without including those congnitive facuties
Pre: No direct relation between being made happy and by anything without congnitive faculties
Answers:
(A) Not relevant to the logic chain
(B) CORRECT,physical pleasure has no congnitive facuties, but people enjoy it more
(C) Both types of music are congnitive
(D) Irrelevant
(E) Irrelevant: nothing about being made happy
2. 36"+27" (Support)
P: Cultivated emmer wheat was found in agriculture remains in Europe and Asia
P: The only place wild emmer wheat was found in the narrow strip of southwest Asia, and earliest cultivated emmer wheat yet found was also in that same strip.
C: Domesticated emmer wheat was first in the narrow strip
Pre: [Basically no clue...] cultivated emmer wheat found elsewhere has big time gap with in the strip? not the same early time?
Answers:
(A) another wild wheat is irrelevant
(B) easily to be domesticated and yield is irrelevant
(C) emmer wheat being the most nutritious at that time is irrelevant
(D) CORRECT [Guess, not very clear]
(E) difference between wild emmer wheat and other wild wheat is irrelevant
3. 45" (Weak)
P: sea otter declined with two possible reasons: predattion or disease
P: concurrent decline of seals and sea lions was caused by disease which likely to effect otters
C: otters decline was caused by disease
Pre: could be both <- whales increased / other fish whates eat declined
otters r immune to the disease
Answers:
(A) CORRECT: less seals and sea lions -> killer whales ate other prey as otters -> less otters by predation
(B) Irrelevant: support the two guesses only
(C) nothing about the reasoning
(D) result of otters decline, not reason
(E) nothing about the reasoning
4. 1'31" (Explaination)
P: beverage containers account for great weight of household garbage
P: widespread recycling of al and glass container, and more al recycling practiced
Missing Premise
C: % reduced weight of glass is higher than al
Pre: % unequal to quantity (Other reasons reduced the glass container in garbage)
Answers:
(A) doen't explain the %
(B) products other than beverage is irrelevant
(C) CORRECT: manufacters' less use of glass reduced much more glass garbage
(D) opposite
(E) opposite, should be more glass garbage if sorting is harder
5. 55" (Weak one Support another)
P: the beatle runs fast after insects, but stop for a while after high speed then resume
P:
C: to maintain the pace so get rest / got blind and need time to recover
Pre: ...
Answers:
(A) undermine the blind theory but didn't approve the pace
(B) undermind the rest theory but didn't approve being blind
(C)
(D)
(E)
作者: annieliu830 时间: 2012-12-27 23:42
12.26
1.
1)逻辑链
原因:人类有比其他任何动物都更高超的认知能力
结论:人们一旦认识到这一点,当其他事物不需要使用这些特质时,他们就会不高兴
2)推测
削弱
他们不高兴,不是因为题目中提到的那个原因,而是有其他原因
3)选项分析
A无关,其他动物能不能与人们的反应无关
B正确
C无关
D并不能说明这些运动员不做智力思考,所以不能说明他们的快乐是因为体力还是智力
E无关
2.
1)逻辑链
原因:培育小麦种植的痕迹在欧洲和亚洲的最早的农业种植区被发现的;
唯一的野生小麦被发现种植的地方是在亚洲西南部分狭窄的一条带上发现的;
最久远的小麦种植遗迹也是在那条狭窄的带上的村庄发现的
结论:小麦最开始肯定是在那条带上的某个地方被培育的
2)推测
加强
填补了某个gap吧
三个原因都是单独的
数据是真的
3)选项分析
A无关
B削弱
C无关
D正确,加强
E削弱
3
1)逻辑链
现象:sea otter的数量下降
推论:鲸鱼的捕食以及疾病。而由于seal和sea lion数量的下降是被认为与疾病有关,因而传染seal和sea lion的疾病也非常有可能会感染sea otter
2)推测
削弱
鲸鱼本来是吃seal和sea lion,而现在这些数目下降了,因而来捕食sea otter了
3)选项分析
A正确
B直接否认的是现象,而不应该否认现象,应该否认推理过程才对
C只是对题目中的现象重述而已,无意义
D无关,我们关心的是sea otter数量变化的原因,而不是sea otter数量变化后对其他动物的影响
E无关
4.
1)逻辑链
矛盾题
铝回收的数量应该高于玻璃回收的数量
但在家居废品中,玻璃重量的减少的比重却高于铝重量减少的比重
2)推测
Explain
玻璃的重量比较重,故虽然减少的数目比较少,但重量比较多
啤酒瓶通常是玻璃的,而不是铝的
3)选项分析
A正确
B与题目中说铝回收的更多无关
C削弱
D无关
E无关
错因分析
c他因,说明玻璃重量减少的原因不是因为回收了,而是因为制造商制造的本来就少了
a玻璃瓶重量减少的比重 与 铝重量减少的比重 相比。所以即使玻璃瓶重量高,但与他自己本身相比,铝也是与他自己本身质量相比,这样其实就无关自己本身质量了,只要减少得少,那么就始终少
5.
1)逻辑链
现象:TB是跑得非常快的动物,几乎可以追上所有不飞的昆虫。但跑的过程中总要间接性地停一停,再继续追
解释:1.无法保持速度,因而要停下
2.不能处理快速变化的信息
2)推测
Evaluation
要有一个神马东西能加强一个,然后削弱另外一个
3)选项分析
A否认体力和视力论
B否认体力和视力论
C支持体力论,否认视力论,正确
D无关
E既不否认视力论,也不否认体力论
作者: zxppx 时间: 2013-1-9 10:57
1.
Human beings have cognitive faculties that are
superior to those of other animals, and once humans
become aware of these, they cannot be made happy
by anything that does not involve gratification of
these faculties.
Which one of the following statements, if true, most
calls into question the view above?
———Bible Weaken Question Problem Set NO.1
(A) Certain animals—dolphins and chimpanzees,
for example—appear to be capable of
rational communication.
There is no relationship about other animals.
(B) Many people familiar both with intellectual
stimulation and with physical pleasures
enjoy the latter more.
It successfully points out the weak in the argument. Physical pleasures are not included in the cognitive ability. So it is the best answer.
(C) Someone who never experienced classical
music as a child will usually prefer popular
music as an adult.
It does not call into question the view above.
(D) Many people who are serious athletes consider
themselves to be happy.
The same to C.
(E) Many people who are serious athletes love
gourmet food.
Although the choice seems to be valid at first glance, a more thorough reveals that it does not hurt the argument above. For instance, if these athletes did not eat gourmet food previously, when they have the opportunity to enjoy gourmet food, they would love them naturally.
21.(28551-!-item-!-188;#058&003690)
Traces of cultivated emmer wheat have been found among the earliest agricultural remains of many archaeological sites in Europe and Asia. The only place where the wild form of emmer wheat has been found growing is a relatively narrow strip of southwest Asia. Since the oldest remains of cultivated emmer wheat yet found are from village sites in the same narrow strip, it is clear that emmer wheat was first domesticated somewhere in that strip.
Which of the following, if true, most strengthens the argument?
(A) The present day distribution of another wild wheat, einkorn, which was also domesticated early in the development of agriculture, covers a much larger area of southwest Asia.
There is no relationship with condition of the present day.
(B) Modern experiments show that wild emmer wheat can easily be domesticated so as to yield nearly as well as traditionally domestic strains.
It does not solve the difference between wild emmer wheat and traditionally domestic strains.
(C) At the time when emmer wheat was first cultivated, it was the most nutritious of all the varieties of grain that were then cultivated.
There is no relationship with the nutrition of the wild emmer wheat.
(D) In the region containing the strip where wild emmer wheat has been found, climatic conditions have changed very little since before the development of agriculture.
It helps to prove that emmer wheat can survive during the long term. So it is the best answer.
(E) It is very difficult, without genetic testing, to differentiate the wild form of emmer wheat from a closely related wild wheat that also grows in southwest Asia.
It weakens the argument.
22.(28829-!-item-!-188;#058&003748)
In the late 1980s, the population of sea otters in the North Pacific began to decline. There are two plausible explanations for the decline: predation, possibly by killer whales, or disease. Of these two, disease is the more likely, since a concurrent sharp decline in populations of seals and sea lions is believed to have been caused by disease, and diseases that infect these creatures are likely to be able to infect sea otters also.
Which of the following, if true, most seriously weakens the reasoning?
(A) Killer whales in the North Pacific usually prey on seals and sea lions but will, when this food source is scarce, seek out other prey.
It is important for us to evaluate the argument, because if killer whales have not sufficient food, they will prey on otters. BA
(B) There is no indication that the sea otter population at any North Pacific location declined in the 1980s because of substantial numbers of sea otters migrating to other locations.
It just tells us that the decline in the population of otters is not caused by their migration.
(C) Along the Pacific coast of North America in the 1980s, sea otters were absent from many locations where they had been relatively common in former times.
It just reveals that the population of otters has declined.
(D) Following the decline in the population of the sea otters, there was an increase in the population of sea urchins, which are sea otters' main food source.
It just shows that the result of the decline in the population of otters.
(E) The North Pacific populations of seals and sea lions cover a wider geographic area than does the population of sea otters.
It does not help to weaken the argument.
23.(29107-!-item-!-188;#058&003854)
From 1978 to 1988, beverage containers accounted for a steadily decreasing percentage of the total weight of household garbage in the United States. The increasingly widespread practice of recycling aluminum and glass was responsible for most of this decline. However, although aluminum recycling was more widely practiced in this period than glass recycling, it was found that the weight of glass bottles in household garbage declined by a greater percentage than the weight of aluminum cans.
Which of the following, if true of the United States in the period 1978 to 1988, most helps to account for the finding?
(A) Glass bottles are significantly heavier than aluminum cans of comparable size.
Although the choice seems to be valid at first glance, a more thorough analysis reveals that if there is less glass recycled or if there is more aluminum recycled, the argument cannot be justified.
(B) Recycled aluminum cans were almost all beverage containers, but a significant fraction of the recycled glass bottles had contained products other than beverages.
It does not help solve the contradiction.
(C) Manufacturers replaced many glass bottles, but few aluminum cans, with plastic containers.
Because weight of plastic containers is much less than that of glass containers, the choice is sufficient for us to solve the contradiction. BA
(D) The total weight of glass bottles purchased by households increased at a slightly faster rate than the total weight of aluminum cans.
Actually, this choice makes the argument more contradictory.
(E) In many areas, glass bottles had to be sorted by color of the glass before being recycled, whereas aluminum cans required no sorting.
There is no relationship with sorting.
24.(29155-!-item-!-188;#058&003857)
Tiger beetles are such fast runners that they can capture virtually any nonflying insect. However, when running toward an insect, the beetles intermittently stop, and then, a moment later, resume their attack. Perhaps they cannot maintain their pace and must pause for a moment's rest; but an alternative hypothesis is that while running tiger beetles are unable to process the resulting rapidly changing visual information, and so quickly go blind and stop.
Which of the following, if discovered in experiments using artificially moved prey insects, would support one of the two hypotheses and undermine the other?
(A) When a prey insect is moved directly toward a beetle that has been chasing it, the beetle immediately turns and runs away without its usual intermittent stopping.
Because the beetle does not stop, we know nothing about the reason why the beetle would stop during prey.
(B) In pursuing a moving insect, the beetles usually respond immediately to changes in the insect's direction, and pause equally frequently whether the chase is up or down an incline.
It helps to demonstrate that the beetles can see their prey, but does not prove that the pause is due to blind or rest.
(C) The beetles maintain a fixed time interval between pauses, although when an insect that had been stationary begins to flee, the beetle increases its speed after its next pause.
It helps to demonstrate that the pause is caused by rest, rather than blind. BA
(D) If, when a beetle pauses, it has not gained on the insect it is pursuing, the beetle generally ends its pursuit.
We only know the result of pauses, rather than the reason.
(E) When an obstacle is suddenly introduced just in front of running beetles, the beetles sometimes stop immediately, but they never respond by running around the barrier.
It helps to demonstrate that the beetles can see their prey, but does not prove that the pause is due to blind or rest.
作者: pennyz 时间: 2013-2-8 21:58
28sweaken
background:human>animal faculty of cognitive
premise
nce human found his or her ability to coginition]
conclusion:things with cognition would be boring
B
(A) Certain animals—dolphins and chimpanzees,
for example—appear to be capable of
rational communication.-----talk about human,not animal
(B) Many people familiar both with intellectual
stimulation and with physical pleasures
enjoy the latter more.----correct(give counterexample)
(C) Someone who never experienced classical
music as a child will usually prefer popular
music as an adult.----not related to cogonition
(D) Many people who are serious athletes consider
themselves to be happy.---irrelevant
(E) Many people who are serious athletes love
gourmet food.----irrelevant
1;04
background:the wild grain with wide ... is found in west asia
the remain early is found in earope
premise;the first plant grain resemble the wild one in the same narrow strip
conclusion:the emmer wheat was first domesticated tn that strip
rephrase:no early remain was found in other places
D
45s
weaken
background:the mumber of u declined,and the cause may be prayer or disease
premise:the population of dolphin declines for the disease
this disease may also affect u
conclusion:the decline attribute to disease
rephrase:there is not way to contact other sick species
A
explain
40s
background:the garbage reduce mainly because of the cycling of al and grass
premise:although the cycling of al is more popular
conclusion:the grass in garbage reduced more quickly
rephrase:more stuffs are made of al than grass
c
48s
support one,weaken the other
fact:b is fast ,can catch insect not moving
but
catching the moving ones, it will pause for a second
theory 1:can not persist have to take a break
theory 2:too fast can be belind temporarily
rephrase:if we give them insect near enough and they can catch them without pause
A
作者: 一粒黄豆。 时间: 2013-3-10 10:27
3/10 【1-6】
22'
background:human beings have cognitive faculties
conclusion
nce they become aware of these,they cannot be made happy by 不包含这些功能带来的快乐
prephrase:cite evidence
A irrelevant
B strengthen
C strengthen
D weaken
E stengthen
the point is :people are more likely to be made happy by something they never touched, so the athletes who is familiar with his/her career should not be happy.
BCE are strengthening the point.
answer![](static/image/smiley/default/biggrin.gif)
58'
background:emmer wheat have been found among the earliest agricultural remains
premise:wild emmer only found in strip;oldest remains of cultivated emmer were were found also here
conclusion:emmer was first domesticated(引进) in that strip
prephrase:??? I found the argument quite rational...
A 取非 irrelevant
B 取非 irrelevant
C 取非 irrelevant
D ???
E weaken
不懂。。。今天的题目好难
44’
premise:seals and sea lions dead for disease
conclusion:diseases could infect sea otters
prephrase:seals, sea lions are also the predators of killer whales,decrease in these two creatures caused killer whales to kill more sea otters.
A
44'
info1:aluminum recycling was more widely practiced in this period than glass recycling
info2:weight of glass bottles delined by a greater percentage than the weight of aluminum cans.
prephrase:i dont know....
C both input and output have something to do with declining
1'04'
background:tiger beetles can capture any nonflying insect,but wen running toward an insect, they will stop for a moment before resume their attack
hypo1:cannot maintain their pace
hypo2:too quickly to process changing visual info
prephrase:find a insect flying in an constant direction.
A irrelevant
B if the beetles are blind ,they cannot respond immediately to changes in direction,this is likely to be answer
C fixed time interval... ?
D irrelevant
E irrelevant
B
今天的都好难啊![](/static/legacy-emoticon/1.gif)
还是因为我早上起来脑子不清楚呢?
作者: 一粒黄豆。 时间: 2013-3-10 10:42
精炼的题干理解错了,point在于“人能否从cognitive faculties以外的东西获得快乐”所以逻辑80%的重点还是在题干的提炼啊![](/static/legacy-emoticon/1.gif)
作者: 一粒黄豆。 时间: 2013-3-10 10:46
Traces of cultivated emmer wheat have been found among the earliest agricultural remains of many archaeological sites in Europe and Asia.The only place where the wild form of emmer wheat has been found growing is a relatively narrow strip of southwest Asia.Since the oldest remains of cultivated emmer wheat yet found are from village sites in the same narrow strip, it is clear that emmer wheat was first domesticated somewhere in that strip.
premise:1在最早的农耕遗迹中发现了种植的ew
2只在strip发现野生的ew
3最早发现的ew也在这个strip
conclusion:ew最早是在这里被引进的
完全找不到gap。。。。
作者: 一粒黄豆。 时间: 2013-3-10 10:47
最后一题的关键,在于审题
would support one of the two hypotheses and undermine the other?
必须要support一个的同时undermine一个!!
这个题目不典型~ 所以审题还是要仔细
作者: 纽约我爱你 时间: 2013-3-23 23:02
1~30s
Premise: human has the c f better than any other animals
Conclusion: we won’t be happy by anything without f once we understand it.
Weaken: the fact doesn’t mean that we can be happy only by f
2~37s
Premise: 1. some wheat are found at a remained tool
2.the wheat was first found at a place
2. the tool was found at the same place
Conclusion: the wheat was first grown at the place
Support: the tool was used to grow the wheat
3~31s
Premise: 1.the sea seals are dying because of a disease
2. the disease can affect otters
Conclusion: the otters are dying because of the disease
Weaken: 1. the ill seals are from a narrow place but all the otters are declining
2. the number of whale are growing sharply there years
4~55s
factor: 1.the a is more widely recycled
2.the weight of the g found in the garbage was declining more by percentage
Explanation: the use of a was too wide
5~42s
Premise: t runs fast
2.it always stops when chasing the prey
Reason:1.it can’t keep pace
2.it gets blind and must rest
Support: 1.when it’s not chasing prey, it doesn’t need to stop
2. it can see the obstacle in front of it
今天错了最后两个,题干说实话都不长,但是词汇很讨厌!好多都不认识啊,还是要加紧练习
作者: okplokpl0714 时间: 2013-3-30 21:45
精练:
Pre:Human cognitive faculties>other animals
Con
nce humans aware of these--->they cannot be made happy...
Weaken:猜是B吧。。。
我去。。。前两天认真分析没分析对,这回倒蒙对了。。汗。。不过其它选项看起来没有
B像,本来想选D的,但是经过前两天的吃亏,我吸取教训了。。不能选。。。
其实这题我根本没看懂。因为有生词。。。。。
逻辑链:
21.
Trace of cultived emmer wheat found among the earliset agricultural E and A
Only place wild emmer wheat found is narrow strip of southwest Asia.
The oldest emmer wheat found in same narrow strip-->emmer wheat first in that strip
加强:他因,没有其他地方发现older emmer
Answer:D
22.
late 1980s,sea otters in NP began decline.
Two explanations for the decline:
1)killer whales' predation
2)disease
Concurrent sharp decline in seals and sea lions is believed to caused by disease and disease able to infect sea otters--->Disease is more likely
削弱:加强原因1),即加强killer whales’predation
Answer:A
23.
1978--1988,beverage containers steadily decreasing percentage of total garbage in US
The increasingly widespread recycling aluminum and glass is the reason
BUT!!
aluminum recycling wider>glass recycling
aluminum cans declined<glass bottles declined in household
解释:他因,导致玻璃瓶在日常垃圾中的比重减少的更多了
Answer:C
24.
Tb fast-->Tb can capture any nonflying insect
BUT!!
when running an insect,Tb stop and amoment later attack,perhaps Tb cannot maintain their pace
But an alternative hypothesis is that:Tb unable to change visual info.-->go blind to stop
弱一强一:
这种问法还真不多见啊~~
两种解释分别是:
1)Tb不能保持它的速度,必须停下来休息一下---能不能相当于有爆发力没有体力
2)当Tb奔跑时不能处理高速转换的视觉信号,致盲并且停了下来
则猜测:
加强一:Tb体力很足
加强二:Tb反应能力很好。
Answer:B
好吧。。我又短路了。。。答案是C,那就是加强了解释一,休息是为了爆发
作者: haohao8701 时间: 2013-5-22 22:07
哪里能校对答案啊
作者: Feelalive 时间: 2013-7-7 13:12
路过~~~~~~~~~~~~
作者: Mint静默 时间: 2013-8-5 10:56
中间隔了几个星期又回来跟逻辑小分队了
【精练】
1) 计时:15s
2) 逻辑链
Background Information:
Human beings have cognitive faculties that are superior to those of other animals
Premise:
好像找不到P。。
Conclusion:
Human cannot be made happy by anything that does not involve gratification of these faculties.
3) 推测:Weaken
答案应该是举Conclusion的反例
4) 选项分析:排除法选B
(A) Certain animals—dolphins and chimpanzees, for example—appear to be capable of rational communication. 本文关注的是human而非animals
(B) Many people familiar both with intellectual stimulation and with physical pleasures enjoy the latter more. 其实也没看懂这个选项
(C) Someone who never experienced classical music as a child will usually prefer popular music as an adult. 支持,not weaken。 Never experience classical music - prefer popular music
(D) Many people who are serious athletes consider themselves to be happy. 无关
(E) Many people who are serious athletes love gourmet food. 无关
【逻辑链】
21. 这题没怎么懂,求问B为什么不对
1) 计时:62s
2) 逻辑链
Premise:
Both the wild form and cultivated form of emmer wheat has been found growing in a relatively narrow strip of southwest Asia.
Conclusion:
Emmer wheat was first domesticated somewhere in that strip.
3) 推测:
排它因或者直接加强
4) 选项分析:纠结B&D 最后选B
(A) The presentday distribution of another wild wheat, einkorn, which was also domesticated early in the development of agriculture, covers a much larger area of southwest Asia. 无关
(B) Modern experiments show that wild emmer wheat can easily be domesticated so as to yield nearly as well as traditionally domestic strains. 排除wild emmer在其他地方也长过(narrow strip of southwest Asia is not the only place where the wild form of emmer wheat has been found growing.)只不过没活下来的可能性。
(C) At the time when emmer wheat was first cultivated, it was the most nutritious of all the varieties of grain that were then cultivated. 无关
(D) In the region containing the strip where wild emmer wheat has been found, climatic conditions have changed very little since before the development of agriculture. 无关
(E) It is very difficult, without genetic testing, to differentiate the wild form of emmer wheat from a closely related wild wheat that also grows in southwest Asia. Weaken
22.
1) 计时:18s
2) 逻辑链
Background Information:
In the late 1980s, the population of sea otters inthe North Pacific began to decline.
Premise:
a concurrent sharp decline in populations of seals and sea lions isbelieved to have been caused by disease
Conclusion:
diseases that infect these creatures are likely to be able to infect sea ottersalso.
3) 推测:
选项会引导我们推测出真正原因是killer whales
4) 选项分析:
(A) Killer whales in the North Pacificusually prey on seals and sea lions but will, when this food source is scarce,seek out other prey. 加强了是killerwhale捕杀sea otters的推论,correct
(B) There is no indication that the seaotter population at any North Pacific location declined in the 1980s because of substantial numbers of sea otters migrating toother locations. 无关
(C) Along the Pacific coast of NorthAmerica in the 1980s, sea otters were absent from many locations where they hadbeen relatively common in former times. 重复P,无关
(D) Following the decline in the populationof the sea otters, there was an increase in the population of sea urchins, which are sea otters' main food source. 无关
(E) The North Pacific populations of seals and sea lions cover awider geographic area than does the population of sea otters. 无关比较
23.
1) 计时:38s
2) 逻辑链
Background Information:
From 1978 to 1988, beverage containers accountedfor a steadily decreasing percentage of the total weight of household garbagein the United States.
Premise:
aluminum recycling was more widely practiced in this period than glassrecycling
Contrary Conclusion:
the weight of glass bottles in household garbage declined by a greaterpercentage than the weight of aluminum cans.
3) 推测:
glass bottles的基数很小
4) 选项分析:
(A) Glass bottles are significantly heavierthan aluminum cans of comparable size. C中说的是percentage,已经消除了这个因素
(B) Recycled aluminum cans were almost allbeverage containers, but a significant fraction of the recycled glass bottleshad contained products other than beverages. 无关
(C) Manufacturers replaced many glassbottles, but few aluminum cans, with plastic containers. 它因,correct
(D) The total weight of glass bottlespurchased by households increased at a slightly faster rate than the totalweight of aluminum cans. 反,基数变大
(E) In many areas, glass bottles had to be sorted by color of the glass before being recycled,whereas aluminum cans required no sorting. 无关
24.
1) 计时:32s
2) 逻辑链
Background Information:
when running toward an insect, the beetlesintermittently stop, and then, a moment later, resume their attack.
Hypothesis 1:
they cannot maintain their pace and must pause for a moment's rest;
hypothesis 2:
while running tiger beetles are unable to process the resulting rapidlychanging visual information, and so quickly go blind and stop.
3) 推测:
问题说的很清楚了。。支持某一hypothesis
4) 选项分析:纠结B&C
(A) When a prey insect is moved directlytoward a beetle that has been chasing it, the beetle immediately turns and runsaway without its usual intermittent stopping. 反P
(B) In pursuing a moving insect, thebeetles usually respond immediately to changes in the insect'sdirection, and pause equally frequently whether the chase is up or down anincline.支持假设1,Correct
(C) The beetles maintain a fixed timeinterval between pauses, although when an insect that had been stationarybegins to flee, the beetle increases its speed after its next pause. 可能为假设1or2
(D) If, when a beetle pauses, it has notgained on the insect it is pursuing, the beetle generally ends its pursuit. 无关
(E) When an obstacle is suddenly introducedjust in front of running beetles, the beetles sometimes stop immediately, butthey never respond by running around the barrier.无关
作者: Elisha728 时间: 2013-8-28 07:41
DDACB
9'05''
作者: Elisha728 时间: 2013-8-28 07:44
corneliaflower 发表于 2011-11-24 22:29 ![](static/image/common/back.gif)
逻辑链KEY DACC精练解释Question #1. Weaken. June 1999 LSAT, Section #2, #15. The correct answer choic ...
错了两个啊!!!![](static/image/smiley/default/cry.gif)
作者: Elisha728 时间: 2013-8-28 22:13
okplokpl0714 发表于 2013-3-30 21:45 ![](static/image/common/back.gif)
精练:
Pre:Human cognitive faculties>other animals
Connce humans aware of these--->they cannot be ...
最后一题为什么选C不选B? 大神帮忙解答呗![](static/image/smiley/default/biggrin.gif)
作者: m1nt 时间: 2013-9-3 17:49
1. 20’
Pre: human beings have cognitive faculties that are superior to those of other animals
Con: once humans become aware of these, they cannot be made happy by anything that does not involve gratification of these faculties.
Weaken: some are happy by things that does not involve gratification of these faculties.
2. 20’
Pre: EW found earliest in E and A, wild EA found in narrow strip of SA
Con: EW first domesticated in narrow strip of SA
Support: the condition didn’t change much
ABC out of scope
D right
E weaken
3. 28’
Pre: disease and killer whales lead the SO to decline
Con: disease is more likely, S and SL got disease and infect SO
Weaken: kill whales lead the decline
A 正确
4. 25’
Pre: recycling A and G led to the decline of weight of the garbage
Con: A recycling is more widely than G, weight of G declined by a greater% than weight of A
Explanation: less use of G
C right
B E 无关
D support
5. 30’
Pre:TB run fast and stop then resume the attack
Con: 1 cant maintain pace and have to stop
2 unable to process the changing visual info, go blind and stop
Support one, weaken another
作者: meckyona 时间: 2013-9-9 23:10
15s
B: human have cognitive faculties that are superior to other animals
P: once people aware
C: the can’t be made happy except gratification
问weaken
Gratification 满意 cognitive 认知的
选B
21 。29s
B traces of ew have been found among earlist agriculture sites
P: the only place ew has been found growing is in narrow strp, the oldest remains of ew are also narrow strip
C: ew was first domesticated in that strip
问加强,也许有考古证据..?
A 无关
B 无关
C谁管你营养成分啊
D right
E irrelevant
30s
B population of sea otters bagan to drop, two reasons, predation by killer whales or disease
P: disease more likely since seals and sea lions also declined by disease
C disease
Seriously weaken 也许是支持另一个原因
A 很有可能,right
B 无关
C 无关
D 当然会这样= =
E 无关
Sea otters 海獭
B beverage decrease steadily of the total weight of garage, aluminum can recycling is widely practice, but glass bottle decrease more rapidly
Support idea 很可能是因为商家更多使用aluminum
A 无关
B 无关
C right
D 不支持
E 不支持
最后一题做过但是还是不确定QAQ
作者: 览物之情 时间: 2013-10-8 02:46
10月7号
1.19s
Premise: Human beings have cognitive faculties that are
superior to those of other animals, and once humans
become aware of these
Conclusion: they cannot be made happy
by anything that does not involve gratification of
these faculties.
因果推理 人类有比动物更高的认知能力,如果这种认知能力不能被满足,人就不会感觉开心。也就是说人不会被不认识的东西取悦。
Which one of the following statements, if true, most
calls into question the view above?
P these faculties has no relation to happiness
choice B
A) Certain animals—dolphins and chimpanzees, for example—appear to be capable of
rational communication. __________it’s neither about happy nor about these faculties.
(B) Many people familiar both with intellectual stimulation and with physical pleasures
enjoy the latter more. ____________it mentions happy and congnition may be correct
(C) Someone who never experienced classical music as a child will usually prefer popular
music as an adult. _______________ not about happy so irrelevant..
(D) Many people who are serious athletes consider themselves to be happy. ______only mention happy so irrelevant.
(E) Many people who are serious athletes love gourmet food._______ it’s neither about happy nor about these faculties so irrelevant.
2 32s
Premise: Traces of cultivated emmer wheat have been found among the earliest agricultural remains of many archaeological sites in Europe and Asia.The only place where the wild form of emmer wheat has been found growing is a relatively narrow strip of southwest Asia. Since the oldest remains of cultivated emmer wheat yet found are from village sites in the same narrow strip,
Conclusion: it is clear that emmer wheat was first domesticated somewhere in that strip.
因为最古老的种植的麦子在strip发现,野生的也只有那里有。所以那里是最早种植麦子的地方。相关因果
Which of the following, if true, most strengthens the argument?
P:There are no strips that grow wheat that have not been found
(A) The presentday distribution of another wild wheat, einkorn, which was also domesticated early in the development of agriculture, covers a much larger area of southwest Asia ._________Because it’s another wheat so it is irrelevant
(B) Modern experiments show that wild emmer wheat can easily be domesticated so as to yield nearly as well as traditionally domestic strains. _____irrelevant
(C) At the time when emmer wheat was first cultivated, it was the most nutritious of all the varieties of grain that were then cultivated.______irrelevant
(D) In the region containing the strip where wild emmer wheat has been found, climatic conditions have changed very little since before the development of agriculture._______觉得其他不太对就选了这个
(E) It is very difficult, without genetic testing, to differentiate the wild form of emmer wheat from a closely related wild wheat that also grows in southwest Asia._______ weaken
3 30s
Background:In the late 1980s, the population of sea otters in the North Pacific began to decline.There are two plausible explanations for the decline:predation, possibly by killer whales, or disease.
Premise: a concurrent sharp decline in populations of seals and sea lions is believed to have been caused by disease, and diseases that infect these creatures are likely to be able to infect sea otters also.
Conclusion: Of these two, disease is the more likely to be the cause
相关因果:sea lions的减少是由disease导致,因此sea otters的减少也是由disease导致。
Which of the following, if true, most seriously weakens the reasoning
P 相关性不存在 sea otters are immune to the disease or it is not a deadly disease.它因导致 the number of killer whales increased a lot
(A) Killer whales in the North Pacific usually prey on seals and sea lions but will, when this food source is scarce, seek out other prey._______possible because the number of sea lions reduced so it might prey sea otters.correct
(B) There is no indication that the sea otter population at any North Pacific location declined in the 1980s because of substantial numbers of sea otters migrating to other locations.____________________rulling out other explanation actually strength the argument.
(C) Along the Pacific coast of North America in the 1980s, sea otters were absent from many locations where they had been relatively common in former times.___________just repeat the background so it is irrelevant
(D) Following the decline in the population of the sea otters, there was an increase in the population of sea urchins, which are sea otters' main food source.__________irrelevant
(E) The North Pacific populations of seals and sea lions cover a wider geographic area than does the population of sea otters.____irrelevant
4 32s
Premise: However, although aluminum recycling was more widely practiced in this period than glass recycling
Outcome: the weight of glass bottles in household garbage declined by a greater percentage than the weight of aluminum cans.
现象解释
P any possible reason as long as it explain the result can be oK. Such as people use more A so that they use less G.
Which of the following, if true of the United States in the period 1978 to 1988, most helps to account for the finding?
(A) Glass bottles are significantly heavier than aluminum cans of comparable size.______更矛盾
(B) Recycled aluminum cans were almost all beverage containers, but a significant fraction of the recycled glass bottles had contained products other than beverages.______irrelevant
(C) Manufacturers replaced many glass bottles, but few aluminum cans, with plastic containers._____possible because manufactures use less G correct.
(D) The total weight of glass bottles purchased by households increased at a slightly faster rate than the total weight of aluminum cans._____not explain the result but make it more complex.
(E) In many areas, glass bottles had to be sorted by color of the glass before being recycled, whereas aluminum cans required no sorting._____irrelevant.
5 42s
Background: Tiger beetles are such fast runners that they can capture virtually any nonflying insect.
Premise: However, when running toward an insect, the beetles intermittently stop, and then, a moment later, resume their attack.
Conclusion 1 they cannot maintain their pace and must pause for a moment's rest;
Conclusion2 while running tiger beetles are unable to process the resulting rapidly changing visual information, and so quickly go blind and stop.
现象解释
Which of the following, if discovered in experiments using artificially moved prey insects, would support one of the two hypotheses and undermine the other?
There is a cap between the premise and the conclusion. P to support 2and undermine1 it can be when the insects moves straight, the beetles do not stop. To support 1 and undermine 2 it can be when the insects move straight it also stop. Any way it should mention two things
Choice B
(A) When a prey insect is moved directly toward a beetle that has been chasing it, the beetle immediately turns and runs away without its usual intermittent stopping. Undermine1 但不能support 2 因为是directly.
(B) In pursuing a moving insect, the beetles usually respond immediately to changes in the insect's direction, and pause equally frequently whether the chase is up or down an incline. Undermine 2 and support 1
(C) The beetles maintain a fixed time interval between pauses, although when an insect that had been stationary begins to flee, the beetle increases its speed after its next pause. Only mention 1
(D) If, when a beetle pauses, it has not gained on the insect it is pursuing, the beetle generally ends its pursuit. Irrelevant
(E) When an obstacle is suddenly introduced just in front of running beetles, the beetles sometimes stop immediately, but they never respond by running around the barrier. Only support conclusion2
这题不知道。
作者: w.melhere 时间: 2013-10-20 10:57
今天又错了两道...好桑心。觉得十二月底一战真是超级没把握。
1.22‘
2.36’ 做错了,做题的时候没有注意到 ‘The only place where the wild form of emmer wheat has been found growing is a relatively narrow strip of southwest Asia.Since the oldest remains of cultivated emmer wheat yet found are from village sites in the same narrow strip,’是在比较现在和过去Asia都有E的存在,这样的话D选项就很好理解了。
3.39‘
4.55’ 第二遍做,还是错了。原文逻辑是,广泛使用AL,则AL所占比例应该大于glass,而观察则是glass更大,这里是一个paradox。 A选项只考虑了重量因素,没有考虑数量;若数量相差足够大,则不能说明问题了。 C选项说因为塑料代替了glass,由于被替换所以gl少了,所以下降比例大。 (这道题我要好好记住!!)
5.34‘
作者: 花生张 时间: 2013-10-21 19:12
计时:20S
逻辑链:premise: humans become aware that Human beings have cognitive faculties that are
superior to those of other animals
Conclusion: they cannot be made happy by anything that does not involve gratification of these faculties.
预测:weaken?
除了involve gratification 之外,还有其他的可以使人happy的吗?
选项分析:选C
A 无关
B 无关
C 保留
D 无关
E 无关
错误原因分析:选C的原因是因为我觉得C说明了小孩子在对一个事物没有认知之前,没有gratification的标准,没有标准的时候也会觉得happy。错误原因是没有看懂题目,cognitive是指心理,与之相对的是physical.
计时:42S
逻辑链:the wild form of emmer wheat is in a narrow strip--- the oldest remains of cultivated emmer wheat yet found are from village sites in the same narrow strip ---emmer wheat was first domesticated somewhere in that strip.
推测:strengthen?
有其他证据证明emmer wheat 发源于这个地方
选项分析:选B
A 无关
B 加强
C 无关
D 加强 气候条件利于种植
E 削弱
B D相比 B更直接的加强结论
计时:30S
逻辑链:SO数量下降---两个解释:捕食或疾病—疾病的可能性更大—因为其他的动物因为一种疾病数量下降了
推测:weaken? 找SO和其他动物的不相似的地方,或找能增强捕食的可能性的答案
选项分析:选C
A 削弱 保留
B 无关
C 集体性性下降 保留
D 无关
E 无关
C 选项涉及到不相似性,A虽然说会找其他的代替食物,但不一定就是SO
计时:39S
逻辑链:饮料包装在家庭垃圾中所占的重量下降—可回收包装是原因所在—虽然A 使用量更大,但玻璃的重量下降更多
推测:explanation? 玻璃的基数少?玻璃本来就比铝重
选项分析:选C
A 保留
B 无关, 讨论的是饮料
C 玻璃被塑料替代 重量自然下降
D 无关
E 无关
计时:50S
逻辑链:TB在抓小虫子的时候会突然停一下---假设之一是它们不能一直跑,要休息一下—假设之二是它们在跑的时候,看不到快速移动的物体
选项分析:选B
作者: cecidefok 时间: 2013-10-26 19:37
精练题
P: 人类有优于动物的认知能力
C: 人类认识到自己的认知能力(HB)------->/HNI (人类为与能力相关的事情而感到开心)
问削弱
(A)说了动物的情况,但没有weaken人类的情况
(B)人类更欣赏自己的体能特征,weaken了
(C)没有说明由于对自己能力的认知的信息
(D)没有weaken
(E)无关
选B
21.
P:人工种植的EW在欧洲和亚洲的考古地址中的最早的农业残迹中被发现
野生的EW唯一被发现的位置是某strip
人工种植的EW的残迹也在某strip中被发现
C:EW首次在某strip中被人工种植
推测:
问加强
A.无关.与另外一种wheat无关
B......无感
C.没有提及到wheat的营养程度
D.天气条件没有大变,识得野生的wheat,可以人工种植,变得更加合理,strengthen
E.提到分辨出野生和人工种植的...没联系..
选D
22.
选A
23.
选D
24.
选C
作者: irenetopia 时间: 2013-11-19 12:12
真的好痛苦……今天错了好多……还有你们是怎么做到30S-50S一题的……
【精练】
1. Human beings have cognitive faculties that are
superior to those of other animals, and once humans
become aware of these, they cannot be made happy
by anything that does not involve gratification of
these faculties.
Which one of the following statements, if true, most
calls into question the view above?
———Bible Weaken Question Problem Set NO.1
逻辑链:人类比动物优在有感知能力——当他发现了他有这个能力——他就不会感到高兴如果不涉及到令他们满意的这个faculities
问削弱
思路:就找即使没有faculities他也不会不高兴。
Certain animals—dolphins and chimpanzees,
for example—appear to be capable of
rational communication.——扯到动物,排除
(B) Many people familiar both with intellectual
stimulation and with physical pleasures
enjoy the latter more.——intellectual stimulation和faculities异曲同工,身体和感知能力有点属于精神和肉体的关系,人们更喜欢后者,削弱命题。
(C) Someone who never experienced classical
music as a child will usually prefer popular
music as an adult.——这个如果用到F上来,只能解释如果一个人没有意识到国F,他会更喜欢B。而题目中是,他意识到了F,那么没有F他就不高兴。不一样。
(D) Many people who are serious athletes consider
themselves to be happy.——运动员认为他们很开心……是我打开方式不对?
(E) Many people who are serious athletes love
gourmet food.——这两个选项没看懂,我pass了
【逻辑链】
21.1:54min
Traces of cultivated emmer wheat have been found among the earliest agricultural remains of many archaeological sites in Europe and Asia.The only place where the wild form of emmer wheat has been found growing is a relatively narrow strip of southwest Asia.Since the oldest remains of cultivated emmer wheat yet found are from village sites in the same narrow strip, it is clear that emmer wheat was first domesticated somewhere in that strip.
Which of the following, if true, most strengthens the argument?
逻辑链:野生EW只在SA的一个地方被发现过,因为最古老的人工培育EW也在同样的地方发现过——所以EW是在那个地方被培育出来的
问加强
思路:一切可能改变的因素都不存在,看选项吧其实我也不知道
The presentday distribution of another wild wheat, einkorn, which was also domesticated early in the development of agriculture, covers a much larger area of southwest Asia.削弱
(B) Modern experiments show that wild emmer wheat can easily be domesticated so as to yield nearly as well as traditionally domestic strains.提到可以被轻易繁殖——削弱
(C) At the time when emmer wheat was first cultivated, it was the most nutritious of all the varieties of grain that were then cultivated.——一个小麦的属性,无关
(D) In the region containing the strip where wild emmer wheat has been found, climatic conditions have changed very little since before the development of agriculture.算是加强吧——好牵强
(E) It is very difficult, without genetic testing, to differentiate the wild form of emmer wheat from a closely related wild wheat that also grows in southwest Asia.——削弱
57S
In the late 1980s, the population of sea otters in the North Pacific began to decline.There are two plausible explanations for the decline:predation, possibly by killer whales, or disease.Of these two, disease is the more likely, since a concurrent sharp decline in populations of seals and sea lions is believed to have been caused by disease, and diseases that infect these creatures are likely to be able to infect sea otters also.Which of the following, if true, most seriously weakens the reasoning?
逻辑链:海豹数量下降——因为病或者被捕食——因为病的比较可能因为同地区的海象啥的都得病了
问削弱:找出质疑,也就是相关于被捕的几率大
Killer whales in the North Pacific usually prey on seals and sea lions but will, when this food source is scarce, seek out other prey.——bingo!因为虎鲸没有海象次了,只能次海豹
(B) There is no indication that the sea otter population at any North Pacific location declined in the 1980s because of substantial numbers of sea otters migrating to other locations.——无关啊
(C) Along the Pacific coast of North America in the 1980s, sea otters were absent from many locations where they had been relatively common in former times.——栖息地比较,无关
(D) Following the decline in the population of the sea otters, there was an increase in the population of sea urchins, which are sea otters' main food source.——无关
(E) The North Pacific populations of seals and sea lions cover a wider geographic area than does the population of sea otters.——无关,都是在说属性而不是影响因素
23.1.57S
From 1978 to 1988, beverage containers accounted for a steadily decreasing percentage of the total weight of household garbage in the United States.The increasingly widespread practice of recycling aluminum and glass was responsible for most of this decline.However, although aluminum recycling was more widely practiced in this period than glass recycling, it was found that the weight of glass bottles in household garbage declined by a greater percentage than the weight of aluminum cans.Which of the following, if true of the United States in the period 1978 to 1988, most helps to account for the finding?
逻辑链:饮料罐重量总数下降因为回收——铝罐比玻璃的回收使用率高——但是玻璃在垃圾中的比率小了
问原因:没有告诉总重量,比率小了只可能从来推测是玻璃制品少了,找出改变玻璃制品的因素
Glass bottles are significantly heavier than aluminum cans of comparable size.——基数不知道,排除
(B) Recycled aluminum cans were almost all beverage containers, but a significant fraction of the recycled glass bottles had contained products other than beverages.——只能说明玻璃瓶重了啊
(C) Manufacturers replaced many glass bottles, but few aluminum cans, with plastic containers.——bingo!因为很多玻璃瓶都被换成塑料的了
(D) The total weight of glass bottles purchased by households increased at a slightly faster rate than the total weight of aluminum cans.——和增长速率无关
(E) In many areas, glass bottles had to be sorted by color of the glass before being recycled, whereas aluminum cans required no sorting.——分类也不能改变数量
这题一开始选了A,注意注意。
24. 1:54
Tiger beetles are such fast runners that they can capture virtually any nonflying insect.However, when running toward an insect, the beetles intermittently stop, and then, a moment later, resume their attack.Perhaps they cannot maintain their pace and must pause for a moment's rest; but an alternative hypothesis is that while running tiger beetles are unable to process the resulting rapidly changing visual information, and so quickly go blind and stop.Which of the following, if discovered in experiments using artificially moved prey insects, would support one of the two hypotheses and undermine the other?
逻辑链:TB捕捉昆虫的时候,飞过去会停顿一会再飞——因为1,他刹不住2.因为视力调节跟不上——求解释
思路:如果是1,刹不住,那么当静止的
When a prey insect is moved directly toward a beetle that has been chasing it, the beetle immediately turns and runs away without its usual intermittent stopping.——跟掉头跑没什么关系吧
(B) In pursuing a moving insect, the beetles usually respond immediately to changes in the insect's direction, and pause equally frequently whether the chase is up or down an incline.——我原来选的是B,因为这样就能知道不是因为视力影响的——求解释
(C) The beetles maintain a fixed time interval between pauses, although when an insect that had been stationary begins to flee, the beetle increases its speed after its next pause.——是正确答案,但是不懂,求解释
(D) If, when a beetle pauses, it has not gained on the insect it is pursuing, the beetle generally ends its pursuit.——他放弃追虫子了,无关
(E) When an obstacle is suddenly introduced just in front of running beetles, the beetles sometimes stop immediately, but they never respond by running around the barrier.——两个都削弱了,视力又好又刹得住……好吧
作者: irenetopia 时间: 2013-11-19 12:17
qiuhua01234567 发表于 2011-11-25 21:54 ![](static/image/common/back.gif)
欢迎小花~~-- by 会员 balapupu (2011/11/25 21:40:55)
是我嘛?自恋下可不可以 ...
弱弱的问,你们都是怎么提高速度的,好痛苦啊我
作者: goodluck22 时间: 2014-3-1 19:54
Background: Human beings have cognitive faculties that are superior to those of other animals
Conclusion: once humans be aware of these, they cannot be happy by anything that does not involve grarification
Prephrase: other faculties can make people happy
ANS: B
Background: E wheat found among the earliest remains in EU and Asia
Premise: wild form of e wheat in SA, the oldest remains of wheat are found in narrow strip
Conclusion: first domesticated in strip
Prephrase: growing – domesticated
A. another – out of scope
B. compare to yield Irrelevant
C. Irrelevant
D. Correct – exclude the factor that will affect
E. Irrelevant
Background: late 1980s, population of sea otters decline
Premise: predation or disease. Disease is more likely. Because the seals and seal lion caused by disease , and disease will infect sea otters also
Prephrase: predator cause the population decline or disease will not cause sea otters died
A. Correct, predator kill sea otters
B. Irrelevant
C. Irrelevant
D. Out of scope
E. Not discuss about the geographic areas
Background: total weigh of household garbage decline
Premise: practice of recycling aluminum and glass was responsible
Conclusion: widespread : A> G weight decline: G>A
Prephrase: G recycle more
A. Irrelevant
B. Irrelevant
C. correct
D. Irrelevant
E. Irrelvant
Background: Tb run fast can capture all nonflying insects
Premise: two theories : 1. Stop and resume- cannot maintain the pace
2. unable to process the rapidly changing visual information, go blind and stop
ANS: C
作者: cyndichiang 时间: 2014-4-4 14:16
精炼:
1、27‘’
humans
Premise:human become aware of that they have cognitive faculties that are superior to those of other animals
Conclusion
nly gratification of these faculties can make people happy
Q:weaken:
prephrase: 它因:其他因素也可能使people feel happy,
二者没有关系:有因无果,无因有果的例证,比如有一些人没有意识到他们认知能力好他们也很快乐,意识到了不一定快乐。。
选项分析:
(A) Certain animals—dolphins and chimpanzees,
for example—appear to be capable of
rational communication.------irrelevant,结论在于人,说动物无关
(B) Many people familiar both with intellectual
stimulation and with physical pleasures
enjoy the latter more.-------CORRECT,无因有果,他们不是因为认知能力好而快乐,而是因为身体好而快乐
(C) Someone who never experienced classical
music as a child will usually prefer popular
music as an adult.----irrelevant,时间上的对比与文章中的无关..理解错了:music也是一种认知能力,但感觉是没有提到他们意识到之后感到快乐,与结论无关,只是纵向比较,没有起到削弱作用
(D) Many people who are serious athletes consider
themselves to be happy.-----irrelevant, 试图证明无因有果,但是不明显,因为没说运动员是否知道他们感知方面的优越性,同样无关(不确定。。自己瞎YY的。)
(E) Many people who are serious athletes love
gourmet food.------irrelevant,这个选项太主观,而且说是否喜欢某种事物与文章内容无关。 out of scope。。理解错了,答案说:slightly strengthen。。那种食物是要经过感知能力才能prefer的。
逻辑链
2.36‘’
Premise:the wild form of emmer wheat only has been found in a relatively narrow strip
Premise:the oldest remains of cultivated emmer wheat yet found are in the same narrow strip
Conclusion: emmer wheat was first domesticated somewhere in that strip.
Q:Strengthen:
Prephrase: 证明在emmer之前没有其他作物被种植
3.34‘’
BG: There are 2 explanations for the decline of sea otter: predation and diseases
Premise: the number of other animals such as seal and sea lion also declined
Conclusion: Diseases lead to the decline.
Q:Weaken
prephrase: 不是因为diseases而是因为predator:比如kill shark predate seal and sea lion also eat sea otter
kill shark的数量反而增加,如果因为disease那么数量应该减少?
4.59‘’题干一长我就捉急,一着急就看不下去,一看不下去就老回看,一回看时间就长![](static/image/smiley/default/cry.gif)
BG:beverage containers accounted for a steadily decreasing percentage of the total weight of household garbage because of recycling aluminum and glass
Premise:回收铝比回收玻璃更普遍
Conclusion:但是,家庭垃圾中(除去回收的),玻璃重量却减少比铝减重的多
Q:explain
prephrase: 因的特点:比如在回收时和不回收时,玻璃的相差比较大,而吕相差不大
5.34‘’
BG: 有一种虫子?他追捕猎物的时候会突然停顿一下,有两种解释,累了?和暂时失明?
Premise:???
conclusion:支持其中一种反对另一种
Prephrase: 1.累了但是视力没问题2. 视力有问题但是不需要休息
这个还是看选项吧,自己YY不出来。。。= =
作者: simonleo 时间: 2014-4-29 13:17
BBAAA
1.
Time: 29s
Premise: Human have cognitive faculties that are superior to those of other animals.
Conclusion: Once human become aware of these, they cannot be made happy by anything else that does not involve gratification of these faculties.
Prephrase:削弱。任何答案 只要可以显示出一个human 在意识到自己cognitive faculties强、好了以后 还会因为something else that doesn't involce gratification of these faculties而开心的话 即可以削弱arg。
(A) Certain animals—dolphins and chimpanzees,
for example—appear to be capable of
rational communication. 无关 要考虑人的问题
(B) Many people familiar both with intellectual
stimulation and with physical pleasures
enjoy the latter more. countender 表明了除了认知观能以外 另外的physical pleasures更让其感到快乐
(C) Someone who never experienced classical
music as a child will usually prefer popular
music as an adult. 无关 讨论 没意识到的事情 无法削弱
(D) Many people who are serious athletes consider
themselves to be happy. 无法得知这些athletes有没有 aware of their cognitive faculties are superior。
(E) Many people who are serious athletes love
gourmet food. 同D serious athletes 不表明其认知到了他自身的优势。
即B是正解。
2.Time:1:14
P: the only place where wild form of e wheat has been found growing is a narrow strip.
the oldest remains of cultivated e wheat yet found are from village sites in the same narrow strip.
C: it is clear that e wheat was first domesticated somewhere in the strip.
Pre:加强 野生的最早发现在这个地方,这个地方也是培养这个的最早的遗址,要得出家养的首先也在这个地方,则没有其他地方先自己培养了但是还没有被发现。
A) The presentday distribution of another wild wheat, einkorn, which was also domesticated early in the development of agriculture, covers a much larger area of southwest Asia. 其他的distribution涵盖的范围无关,只要求考虑时间上的早晚
(B) Modern experiments show that wild emmer wheat can easily be domesticated so as to yield nearly as well as traditionally domestic strains. countender,填补了arg里面的gap,证明了wild可以容易的被domesticated,那则可以说明最早wild等同于最早domesticated
(C) At the time when emmer wheat was first cultivated, it was the most nutritious of all the varieties of grain that were then cultivated. 无关
(D) In the region containing the strip where wild emmer wheat has been found, climatic conditions have changed very little since before the development of agriculture. 无关
(E) It is very difficult, without genetic testing, to differentiate the wild form of emmer wheat from a closely related wild wheat that also grows in southwest Asia. 无关 closely related wild wheat 无关
不太会prephrase。。。 B选项的逻辑加强也不是特别清晰。。。
3. Time:46s
P:the population of sea otters decline
two explanations:1.predation
2.disease
populations of seals and sea lions concurrently sharp decline is believed to have been caused by disease
C:disease is the more likely reason.
Pre:削弱。 同时不代表同地,也许seal和sea lions生长在一个和sea otter不同的区域。
总之 说出感染它俩的病不一定会感染sea otters。
(A) Killer whales in the North Pacific usually prey on seals and sea lions but will, when this food source is scarce, seek out other prey. countender, 可以说明sea otters 是因为predation而下降的
(B) There is no indication that the sea otter population at any North Pacific location declined in the 1980s because of substantial numbers of sea otters migrating to other locations. 无关,只考虑predation和disease因素
(C) Along the Pacific coast of North America in the 1980s, sea otters were absent from many locations where they had been relatively common in former times.无关
(D) Following the decline in the population of the sea otters, there was an increase in the population of sea urchins, which are sea otters' main food source.无关
(E) The North Pacific populations of seals and sea lions cover a wider geographic area than does the population of sea otters. countender 涵盖更大区域 则比较容易生病,则sea otters 不一定因为生病。。。
相比较下认为A 更合理,可以很好的削弱,不是disease,而是predation,两者的对比突出了。
4 Time:51s
B:beverage containers accounted for a steadily decreasing percentage of the total weight of household garbage in the United States.
P:increasingly widespread practice of recycling aluminum and glass was responsible for most of this decline.
although aluminum recycling was more widely practiced in this period than glass recycling
C-P:it was found that the weight of glass bottles in household garbage declined by a greater percentage than the weight of aluminum cans.
Pre:solve the paradox, aluminum的重量比glass的重量小,则虽然多一些,其减少的重量的百分比可能会小一些。
(A) Glass bottles are significantly heavier than aluminum cans of comparable size. 正解
(B) Recycled aluminum cans were almost all beverage containers, but a significant fraction of the recycled glass bottles had contained products other than beverages. 无关
(C) Manufacturers replaced many glass bottles, but few aluminum cans, with plastic containers. 无关
(D) The total weight of glass bottles purchased by households increased at a slightly faster rate than the total weight of aluminum cans.无关
(E) In many areas, glass bottles had to be sorted by color of the glass before being recycled, whereas aluminum cans required no sorting. 无关
5.Time:57s
p:when running toward an insect, the beetles intermittently stop, and then, a moment later, resume their attack
C:1. they cannot maintain their pace and must pause for a moment's rest
2.while running tiger beetles are unable to process the resulting rapidly changing visual information, and so quickly go blind and stop.
pre:evaluate.如果artificially moved prey insects不转弯 一直直行,beetles停则1对,beetles不停则2对;或者让其一直转弯,经常停则2对。
(A) When a prey insect is moved directly toward a beetle that has been chasing it, the beetle immediately turns and runs away without its usual intermittent stopping. 正解 直行的时候它就不停顿
(B) In pursuing a moving insect, the beetles usually respond immediately to changes in the insect's direction, and pause equally frequently whether the chase is up or down an incline. up and down是无关的
(C) The beetles maintain a fixed time interval between pauses, although when an insect that had been stationary begins to flee, the beetle increases its speed after its next pause.不能加强削弱 不知道是不是转弯引起的
(D) If, when a beetle pauses, it has not gained on the insect it is pursuing, the beetle generally ends its pursuit.无关
(E) When an obstacle is suddenly introduced just in front of running beetles, the beetles sometimes stop immediately, but they never respond by running around the barrier. 有障碍和转向无关
异常纠结 今天居然错了三个。。。
作者: 采药去 时间: 2014-10-22 17:26
我去····上班实习偷偷做的。。。错爆多, 看看也是醉了
B
B-->D
A
A-->C
E-->C
精炼
注意 faulty --能力;gratification--满意;喜悦;
P:人类有优于其他动物的cognitive faculties
C:一旦他们意识到这点,他们就再不会被任何没有involve 这种能力的事情而取悦了
---》言下之意, 人们只会被涉及到认知能力的事情取悦
weaken
A.irrelevant
B.人们更喜欢physical pleasure(并不涉及cognitive)--correct
C.没听过古典音乐的小孩更喜欢popular music?--irrelevant
D.挺迷惑的。。。。;但是还是觉得 不太妥当 serious athelets 感觉吧范围缩小了,求指教?
E.irrelevant
逻辑链
BG:野生emmer wheat只生长在亚洲西南的strip上
P:最古老的remains of emmer wheat被发现在相同的strip
C:emmer wheat was first domesticated somewhere in that strip.
support
A.irrelevant
B.产量--irrelevant
C.nurient--irrelevant
D.气候变化不明显--易于生长--correct
E.irrelevant
3.
BG:sea otters 数量下降;可能是因为 predation or disease
P:与此同时, seals and sea lions的数量也下降了,被认为是这个疾病所致;这个疾病也感染sea otters
C: 很有可能是diseases
weaken
A. other causes--correct
B. indications about the migration--irrelevant
C. location--irrelevant
D. food上升--》otters 也应该上升--》违背BG
E.cover a wild area又咋样, 也不一定会感染sea otters, 缺乏有力支撑
4.
BG:饮料瓶可以解释家庭垃圾总重量的稳定下降--AL和玻璃的广泛应用
situation:用al更多,可是。。。。
【the weight of glass bottles in household garbage declined by a greater percentage than the weight of aluminum cans.】
这句话啥意思?
莫不是。。。
比起铝易拉罐, 家庭垃圾中的玻璃瓶的重量下降的更多
解释这个现象
不明白为啥A错了
5.TB抓昆虫的时候,会间歇性停止之后再攻击
可能是因为
1.他们不能维持他们的节奏,过一会必须要休息
或者 不能迅速的change visual information ,因此会失明然后停止
找一个答案 削弱一个增强一个
A.否定BG 错
B.两者都weaken
C.削弱第一个解释
D.irrelevant
E.没解释
B也感觉不太好 不过相比之下B 更好?
还是好多没明白的感觉。。。![](static/image/smiley/default/cry.gif)
作者: spencerchan 时间: 2015-8-11 15:55
20’’
P: Once human aware they have superior cognitive faculties than other animals
C: People would not be happy by anything that does not involve gratification of these faculties
Prep: the key may provide a possibility that people can be happy when something does not involve gratification of the faculties
A: irrelevant
B: irrelevant
C: irrelevant??
D: CORRECT?? just guess it correct T.T
E: irrelevant
1’06’’ 一开始没有读懂。。
P: The only place whereto wild form of emmer wheat has been found in Asia
P: The oldest remains of cultivated emmer wheat found at the same place
C: Emmer wheat was first domesticated at that place
Prep: The key may provide a fact to support the conclusion OR eliminate the weaken conditions
A: weaken
B: irrelevant
C: irrelevant
D: CORRECT
E: weaken
49’’
P: Population of seals and sea lions decline is caused by disease
P: The disease can infect sea otters
C: The population decline of sea otters is caused by disease
Prep: Other reasons
A: irrelevant?? OR weaken??
B: CORRECT There is NO??, I am doubt it
C: irrelevant
D: irrelevant, and it is common sense
E: strengthen
1’20’’
P: Aluminum recycling was more widely practiced than glass recycling
C: The weight of glass bottles in household garbage is declined greater than aluminum bottles
Prep: I cannot understand the logic. I think recycling was more widely practiced has no relationship with weight declined.
A: CORRECT
B: it cannot explain the conclusion
C: irrelevant
D: it cannot explain the conclusion
E: irrelevant
48’’
P: The tiger beetles have to stop
C1: They cannot maintain their pace, so need a break
C2: They will be blind a moment, so have to stop
Prep: The key shows a supporting fact
A: irrelevant
B: This option shows that tiger beetles are not blind to fast, but it not supports the other reason
C: CORRECT
D: irrelevant
E: irrelevant
让我哭会。。。
作者: MIA926 时间: 2015-8-31 16:09
CR小分队20150831-6
【精练】
1. Human beings have cognitive faculties that are superior to those of other animals, and once humans become aware of these, they cannot be made happy by anything that does not involve gratification of these faculties.
Which one of the following statements, if true, most calls into question the view above?
———Bible Weaken Question Problem Set NO.1
计时:
逻辑链:人类的认知能力比其他动物都要好,而且一旦人类意识到这点,他们就不能对任何无法满足这些faculty的食物感到开心
(A) Certain animals—dolphins and chimpanzees,
for example—appear to be capable of
rational communication. 无关 讨论范围是人类
(B) Many people familiar both with intellectual
stimulation and with physical pleasures
enjoy the latter more. 无关
(C) Someone who never experienced classical
music as a child will usually prefer popular
music as an adult. 无关 讲的是happy的问题
(D) Many people who are serious athletes consider
themselves to be happy. 正确
(E) Many people who are serious athletes love
gourmet food. 无关 同C
【逻辑链】
21.(28551-!-item-!-188;#058&003690)
Traces of cultivated emmer wheat have been found among the earliest agricultural remains of many archaeological sites in Europe and Asia.The only place where the wild form of emmer wheat has been found growing is a relatively narrow strip of southwest Asia.Since the oldest remains of cultivated emmer wheat yet found are from village sites in the same narrow strip, it is clear that emmer wheat was first domesticated somewhere in that strip.
逻辑链:某种耕作小麦在欧洲和亚洲的考古地点都有发现。但只有在西南亚才发现了野生的这种小麦,而且最早的这种小麦的remains也是来自于这个地方,所以这个地方是最早种植小麦的。
Which of the following, if true, most strengthens the argument?
(A) The presentday distribution of another wild wheat, einkorn, which was also domesticated early in the development of agriculture, covers a much larger area of southwest Asia. 无关
(B) Modern experiments show that wild emmer wheat can easily be domesticated so as to yield nearly as well as traditionally domestic strains. 无关
(C) At the time when emmer wheat was first cultivated, it was the most nutritious of all the varieties of grain that were then cultivated. 无关
(D) In the region containing the strip where wild emmer wheat has been found, climatic conditions have changed very little since before the development of agriculture. 正确
(E) It is very difficult, without genetic testing, to differentiate the wild form of emmer wheat from a closely related wild wheat that also grows in southwest Asia. 削弱
22.(28829-!-item-!-188;#058&003748)
In the late 1980s, the population of sea otters in the North Pacific began to decline.There are two plausible explanations for the decline:predation, possibly by killer whales, or disease.Of these two, disease is the more likely, since a concurrent sharp decline in populations of seals and sea lions is believed to have been caused by disease, and diseases that infect these creatures are likely to be able to infect sea otters also.
Which of the following, if true, most seriously weakens the reasoning?
逻辑链:otter population decline , two reasons, disease more likely, because seals and sea lions populations also decline because of the disease.
(A) Killer whales in the North Pacific usually prey on seals and sea lions but will, when this food source is scarce, seek out other prey. 正确
(B) There is no indication that the sea otter population at any North Pacific location declined in the 1980s because of substantial numbers of sea otters migrating to other locations. 加强
(C) Along the Pacific coast of North America in the 1980s, sea otters were absent from many locations where they had been relatively common in former times. 无关
(D) Following the decline in the population of the sea otters, there was an increase in the population of sea urchins, which are sea otters' main food source. 无关
(E) The North Pacific populations of seals and sea lions cover a wider geographic area than does the population of sea otters. 无关
23.(29107-!-item-!-188;#058&003854)
From 1978 to 1988, beverage containers accounted for a steadily decreasing percentage of the total weight of household garbage in the United States.The increasingly widespread practice of recycling aluminum and glass was responsible for most of this decline.However, although aluminum recycling was more widely practiced in this period than glass recycling, it was found that the weight of glass bottles in household garbage declined by a greater percentage than the weight of aluminum cans.
Which of the following, if true of the United States in the period 1978 to 1988, most helps to account for the finding?
逻辑链:recycle more aluminum than glass but glass bottle decline by a greater percentage than aluminum.
(A) Glass bottles are significantly heavier than aluminum cans of comparable size. 无关
(B) Recycled aluminum cans were almost all beverage containers, but a significant fraction of the recycled glass bottles had contained products other than beverages. 无关
(C) Manufacturers replaced many glass bottles, but few aluminum cans, with plastic containers. 正确
(D) The total weight of glass bottles purchased by households increased at a slightly faster rate than the total weight of aluminum cans. 无关
(E) In many areas, glass bottles had to be sorted by color of the glass before being recycled, whereas aluminum cans required no sorting. 无关
24.(29155-!-item-!-188;#058&003857)
Tiger beetles are such fast runners that they can capture virtually any nonflying insect.However, when running toward an insect, the beetles intermittently stop, and then, a moment later, resume their attack.Perhaps they cannot maintain their pace and must pause for a moment's rest; but an alternative hypothesis is that while running tiger beetles are unable to process the resulting rapidly changing visual information, and so quickly go blind and stop.
Which of the following, if discovered in experiments using artificially moved prey insects, would support one of the two hypotheses and undermine the other?
逻辑链: 两种解释,做实验证明一种对一种错
(A) When a prey insect is moved directly toward a beetle that has been chasing it, the beetle immediately turns and runs away without its usual intermittent stopping. 无关
(B) In pursuing a moving insect, the beetles usually respond immediately to changes in the insect's direction, and pause equally frequently whether the chase is up or down an incline. 无关
(C) The beetles maintain a fixed time interval between pauses, although when an insect that had been stationary begins to flee, the beetle increases its speed after its next pause. 正确
(D) If, when a beetle pauses, it has not gained on the insect it is pursuing, the beetle generally ends its pursuit. 无关
(E) When an obstacle is suddenly introduced just in front of running beetles, the beetles sometimes stop immediately, but they never respond by running around the barrier. 无关
作者: 啊云苏苏 时间: 2015-9-2 16:02
1-6-1-question
1)计时:27.29
2)逻辑链:
-P
CF>animals'
-C
Once people are aware of these,things do not involve CF are unable to make them happy
3)Prephrase
可以因为很多原因happy,不一定非要因为CF;
4)选项分析:
A irrelevant,再怎么交流也不如人类,无法攻击P
B bingo!!!人们可以因为physical而快乐
C irrelevant
D irrelevant,和CF无关
E irrelevant
1-6-2-strengthen
1)计时:55.39
2)逻辑链:
-B
CEW are found in the earliest AR of E&A
-P1
the only WEW are found in XX
-P2
the oldest CEW are found in XX
-C
EW was first domesticated in XX
3)Prephrase
别的地方的EW培育种子是来自于XX;后续也没有发现比XX更早的培育trace了
4)选项分析:
A irrelevant,结论说的是emmer不是einkorn
B irrelevant
C irrelevant
D(correct,气候变化小,剔除了另一个变量,说明WEW可以在这个区域存在很久,而不是最近才出现的,strengthen了结论)
E(这题应该是削弱,很难区分导致人们容易弄混,但是基因还是有区别的,不是一种wheat。不要错误的理解为没有区别)
错了,选D
1-6-3-weaken
1)计时:40.88
2)逻辑链:
-B
SO↓,because of Predation or Disease
-P
concurrent Seals↓,SL↓is believed by D
the D can infect SO
-C
D is more likely
3)Prephrase
seal和Sl的死亡发生在别的海域,隔得太远无法传染;这个d对SO来说不一定致命
4)选项分析:
A bingo!是因为seal和SL得病死太多,SW只能吃SO,所以SO减少了
B irrelevant,说的是总量减少
C 重复B,没有意义
D 按结论分析下去逻辑对,但是irrelevant
E support
1-6-4-reason
1)计时:56.33
2)逻辑链:
-B
BC↓,recycling↑是一个主要原因
-P
AR>GR,however GB<AB
3)Prephrase
厂家用glass包装的少了;人们买glass包装的少了
4)选项分析:
A 题目说的比重下降
B opposite
C bingo!!!
D opposite
E opposite,所以没人回收glass,glass应该↑
1-6-5-support one,undermine another
1)计时:55.03
2)逻辑链:
-B
TB can run quickly to capture any insect(nonflying)
抓的时候中途偶尔会停一下,再重新开始
-H1
cannot maintain pace,need a rest
-H2
cannot change VI rapidly,so quickly go blind and stop
3)Prephrase
如果AMI直线逃跑,就不用转变视野,TB持续追捕,就可以否定H1支持H2
重点是要有一种情况TB不停下来
4)选项分析:
A(同时削弱了两个H)
B 削弱了2,但也没有支持1
C(correct,削弱了2,支持了1。这个选项说的是方向没变,它还是要停一下休息)
D irrelvant
作者: kexiner 时间: 2016-2-17 22:54
1-6
【精练】
1. Human beings have cognitive faculties that are
superior to those of other animals, and once humans
become aware of these, they cannot be made happy
by anything that does not involve gratification of
these faculties.
Which one of the following statements, if true, most
calls into question the view above?
———Bible Weaken Question Problem Set NO.1
逻辑链: 人类有cognitive faculties 比其他动物高级,
premise 一旦他们be aware of these,
conclusion 他们将不会因为其他不包含these faculties满足的事而开心
weaken
preprase 不involve这些faculties的事情也能让人开心
Cognitive means “relating to the mental process of knowing,
including reasoning and judgment.” In other words, cognitive faculties are thinking and analyzing, etc.
(A) Certain animals—dolphins and chimpanzees,
for example—appear to be capable of
rational communication.
irrelevant
(B) Many people familiar both with intellectual
stimulation and with physical pleasures
enjoy the latter more.
right
faculties包含Physica了pleasures吗
(C) Someone who never experienced classical
music as a child will usually prefer popular
music as an adult.
classical music和popularmusic对此题无关
(D) Many people who are serious athletes consider
themselves to be happy.
-if the athletes are happy because of the love of sports they play~right
(E) Many people who are serious athletes love
gourmet food.
food 无关
【逻辑链】
21.(28551-!-item-!-188;#058&003690)
Traces of cultivated emmer wheat have been found among the earliest agricultural remains of many archaeological sites in Europe and Asia.The only place where the wild form of emmer wheat has been found growing is a relatively narrow strip of southwest Asia.Since the oldest remains of cultivated emmer wheat yet found are from village sites in the same narrow strip, it is clear that emmer wheat was first domesticated somewhere in that strip.
逻辑链:对于cultivated emmer wheat的起源被发现在最早的农业remains在考古地在欧洲和亚洲。 the only 地点 where野生emmer wheat 被发现生长在亚洲西南部的narrow strip。 因为最古老的remains of 种植的 emmer wheat被发现在同样的narrow strip 。
结论: 很清楚的是 emmer wheat 是第一个被demesticated 在那个 strip
Which of the following, if true, most strengthens the argument?
The presentday distribution of another wild wheat, einkorn, which was also domesticated early in the development of agriculture, covers a much larger area of southwest Asia.
weaken
(B) Modern experiments show that wild emmer wheat can easily be domesticated so as to yield nearly as well as traditionally domestic strains.
(C) At the time when emmer wheat was first cultivated, it was the most nutritious of all the varieties of grain that were then cultivated.
(D) In the region containing the strip where wild emmer wheat has been found, climatic conditions have changed very little since before the development of agriculture.
说明这样的climate适合耕作? 加强? right
(E) It is very difficult, without genetic testing, to differentiate the wild form of emmer wheat from a closely related wild wheat that also grows in southwest Asia.
weaken
22.(28829-!-item-!-188;#058&003748)
In the late 1980s, the population of sea otters in the North Pacific began to decline.There are two plausible explanations for the decline:predation, possibly by killer whales, or disease.Of these two, disease is the more likely, since a concurrent sharp decline in populations of seals and sea lions is believed to have been caused by disease, and diseases that infect these creatures are likely to be able to infect sea otters also.
Which of the following, if true, most seriously weakens the reasoning?
逻辑链 sea otter 数量减少。 有两种可能性解释:1.捕食2.疾病
疾病是更加可能的,因为同时期发生的seal和sea lion的减少是由于疾病,
结论: 疾病会感染这类creatures的也可能感染sea otter
pre predation比疾病更容易感染sea otter
Killer whales in the North Pacific usually prey on seals and sea lions but will, when this food source is scarce, seek out other prey.
seal 和sea lion由于疾病减少 因此whale就吃seaotter。 right
(B) There is no indication that the sea otter population at any North Pacific location declined in the 1980s because of substantial numbers of sea otters migrating to other locations.
strengthen 证明不是migration的原因导致减少
(C) Along the Pacific coast of North America in the 1980s, sea otters were absent from many locations where they had been relatively common in former times.
former times 属于无关
(D) Following the decline in the population of the sea otters, there was an increase in the population of sea urchins, which are sea otters' main food source.
sea urchin 增加 代表 seaotter减少 无关
(E) The North Pacific populations of seals and sea lions cover a wider geographic area than does the population of sea otters.
23.(29107-!-item-!-188;#058&003854)
From 1978 to 1988, beverage containers accounted for a steadily decreasing percentage of the total weight of household garbage in the United States.The increasingly widespread practice of recycling aluminum and glass was responsible for most of this decline.However, although aluminum recycling was more widely practiced in this period than glass recycling, it was found that the weight of glass bottles in household garbage declined by a greater percentage than the weight of aluminum cans.
Which of the following, if true of the United States in the period 1978 to 1988, most helps to account for the finding?
逻辑链:beverage container是家庭垃圾下降的分量中占有很大比重。 回收铝和玻璃的广泛使用是这种下降的主要原因。 然而, 尽管回收铝的使用比玻回收玻璃的使用更加广泛,研究发现,玻璃瓶在家庭垃圾里的下降比铝制品的下降比重要多。
(A) Glass bottles are significantly heavier than aluminum cans of comparable size.
(B) Recycled aluminum cans were almost all beverage containers, but a significant fraction of the recycled glass bottles had contained products other than beverages.
right
whether the glass bottles contain beverage or not, they will discarded as garbage.
(C) Manufacturers replaced many glass bottles, but few aluminum cans, with plastic containers.
right ???
(D) The total weight of glass bottles purchased by households increased at a slightly faster rate than the total weight of aluminum cans.
irrelevant
(E) In many areas, glass bottles had to be sorted by color of the glass before being recycled, whereas aluminum cans required no sorting.
24.(29155-!-item-!-188;#058&003857)
Tiger beetles are such fast runners that they can capture virtually any nonflying insect.However, when running toward an insect, the beetles intermittently stop, and then, a moment later, resume their attack.Perhaps they cannot maintain their pace and must pause for a moment's rest; but an alternative hypothesis is that while running tiger beetles are unable to process the resulting rapidly changing visual information, and so quickly go blind and stop.
Which of the following, if discovered in experiments using artificially moved prey insects, would support one of the two hypotheses and undermine the other?
逻辑链 tiger beetle 可以基本上抓住任何 nonflying insect。然而,当朝着一个insect跑的时候,beetle间歇停止,然后 过一会 回复攻击。 可能他们不能保持他们的速率and 必须停止来休息一下;但是一种替换的解释是由于他们不能变换视觉信息,然后很快go blind
When a prey insect is moved directly toward a beetle that has been chasing it, the beetle immediately turns and runs away without its usual intermittent stopping.
weaken both
(B) In pursuing a moving insect, the beetles usually respond immediately to changes in the insect's direction, and pause equally frequently whether the chase is up or down an incline.
(C) The beetles maintain a fixed time interval between pauses, although when an insect that had been stationary begins to flee, the beetle increases its speed after its next pause.
right
(D) If, when a beetle pauses, it has not gained on the insect it is pursuing, the beetle generally ends its pursuit.
(E) When an obstacle is suddenly introduced just in front of running beetles, the beetles sometimes stop immediately, but they never respond by running around the barrier.
没有weaken第一个
作者: Geranium_Bayy 时间: 2016-3-25 17:31
(做错)1. Human beings have cognitive faculties that are superior to those of other animals, and once humans become aware of these, they cannot be made happy by anything that does not involve gratification of these faculties.
Which one of the following statements, if true, most calls into question the view above?
———Bible Weaken Question Problem Set NO.1
15’’ 逻辑链:人类比其他动物认知能力更好。一旦人们意识到这些,他们就不会对任何没有包含满足的东西感到开心。
削弱:a phenomenon(/蕴含着一个道理) 即使没有包含满足的东西也能让他们感到开心。
(A) Certain animals—dolphins and chimpanzees, for example—appear to be capable of rational communication. Rational communication [not=] cognitive ability
Even though they have this communication ability, human cognitive faculties can still be superior.
(B) Many people familiar both with intellectual stimulation and with physical pleasures enjoy the latter more. 无关
This is the correct answer, and a somewhat risqué one at that. By showing that many
people enjoy the physical more than the cognitive, the answer shows that people can be made happy by
gratification of something other than cognitive faculties. Cognitive faculties, being mental in nature, are of
course distinct from physical pleasures.
(C) Someone who never experienced classical music as a child will usually prefer popular music as an adult. 无关
A preference for a certain type of music is likely a cognition-driven preference, and
this preference is expressed by an adult who would certainly be aware of cognitive faculties. And, since no
suggestion is made that individuals can be made happy without gratification of the cognitive faculties, this
answer is incorrect.
(D) Many people who are serious athletes consider themselves to be happy.
This can be an attractive answer at first, but it depends on the assumption that the
serious athletes are happy due to their athletic endeavors. However, that connection is not explicitly stated,
and it could be that the serious athletes are happy because of some gratification of their cognitive faculties,
in their respective sport or otherwise.
(E) Many people who are serious athletes love gourmet food.
This answer is similar to answer choice (D). A gourmet is a connoisseur of food and
drink, and a connoisseur is a person with deep or special knowledge of a subject. In this sense, there
would be a cognitive element to the enjoyment of gourmet food. As such, this answer may serve to slightly
strengthen the argument because it shows that an individual with experience with the non-cognitive still
retains a love of the cognitive.
21.(28551-!-item-!-188;#058&003690)
Traces of cultivated emmer wheat have been found among the earliest agricultural remains of many archaeological sites in Europe and Asia. The only place where the wild form of emmer wheat has been found growing is a relatively narrow strip of southwest Asia. Since the oldest remains of cultivated emmer wheat yet found are from village sites in the same narrow strip, it is clear that emmer wheat was first domesticated somewhere in that strip.
Which of the following, if true, most strengthens the argument?
24’’ E物最早是在欧洲和亚洲发现的。唯一一个地方发现的野生E物是在一个相对比较狭隘的带状地区找到的。因为最早发现的E物同样是在一个狭隘的带状地区找到的。因此,E物最早是生长在带状地区。
加强:排除strip地区各方面不好的因素/排除可能让E物质外移的因素
(A) The presentday distribution of another wild wheat, einkorn, which was also domesticated early in the development of agriculture, covers a much larger area of southwest Asia. 无关,新信息einkorn
(B) Modern experiments show that wild emmer wheat can easily be domesticated so as to yield nearly as well as traditionally domestic strains. 无关
(C) At the time when emmer wheat was first cultivated, it was the most nutritious of all the varieties of grain that were then cultivated.无关
(D) In the region containing the strip where wild emmer wheat has been found, climatic conditions have changed very little since before the development of agriculture. correct
(E) It is very difficult, without genetic testing, to differentiate the wild form of emmer wheat from a closely related wild wheat that also grows in southwest Asia. 无关
22.(28829-!-item-!-188;#058&003748)
In the late 1980s, the population of sea otters in the North Pacific began to decline.There are two plausible explanations for the decline:predation, possibly by killer whales, or disease.Of these two, disease is the more likely, since a concurrent sharp decline in populations of seals and sea lions is believed to have been caused by disease, and diseases that infect these creatures are likely to be able to infect sea otters also.
Which of the following, if true, most seriously weakens the reasoning?
35’’ 逻辑链:SO数量下降——有两种因素:predation & disease——最可能是disease:seals 和sea lions 数量是被disease影响的,SO也有可能。
削弱:1、S和SL住的不是一个地方(切断S,SL与SO之间的联系)
2、killer whales 的数量在急剧上升
(A) Killer whales in the North Pacific usually prey on seals and sea lions but will, when this food source is scarce, seek out other prey. Correct【source is scarce=S & SO have disease, killers can’t eat】
(B) There is no indication that the sea otter population at any North Pacific location declined in the 1980s because of substantial numbers of sea otters migrating to other locations. 无关
(C) Along the Pacific coast of North America in the 1980s, sea otters were absent from many locations where they had been relatively common in former times. 无关,并没有提到地点
(D) Following the decline in the population of the sea otters, there was an increase in the population of sea urchins, which are sea otters' main food source. SO decline导致的结果本来就是SO的food数量会增加
(E) The North Pacific populations of seals and sea lions cover a wider geographic area than does the population of sea otters. 无关
(做错)23.(29107-!-item-!-188;#058&003854)
From 1978 to 1988, beverage containers accounted for a steadily decreasing percentage of the total weight of household garbage in the United States.The increasingly widespread practice of recycling aluminum and glass was responsible for most of this decline. However, although aluminum recycling was more widely practiced in this period than glass recycling, it was found that the weight of glass bottles in household garbage declined by a greater percentage than the weight of aluminum cans.
Which of the following, if true of the United States in the period 1978 to 1988, most helps to account for the finding?
27’’ 逻辑链:家庭垃圾重量减少——归功于对铝和对玻璃的回收——回收铝的范围比回收玻璃的范围大——玻璃垃圾重量的减少大于铝的
解释:玻璃比铝重
(A) Glass bottles are significantly heavier than aluminum cans of comparable size. Correct
题中涉及的是percentage。Aluminum recycling wider>glass recycling
aluminum cans declined<glass bottles declined in household
解释:他因,导致玻璃瓶在日常垃圾中的比重减少的更多了这样才能削弱原文。题目中讲到percentage的时候不要拿质量&数量来作比较,因为并不知道基数。只能拿此类物种在此占得权重是增加了还是减少了。
(B) Recycled aluminum cans were almost all beverage containers, but a significant fraction of the recycled glass bottles had contained products other than beverages.
(C) Manufacturers replaced many glass bottles, but few aluminum cans, with plastic containers.
(D) The total weight of glass bottles purchased by households increased at a slightly faster rate than the total weight of aluminum cans. Slightly & significantly(题中)
(E) In many areas, glass bottles had to be sorted by color of the glass before being recycled, whereas aluminum cans required no sorting. 无关
24.(29155-!-item-!-188;#058&003857)
Tiger beetles are such fast runners that they can capture virtually any nonflying insect.However, when running toward an insect, the beetles intermittently stop, and then, a moment later, resume their attack.Perhaps they cannot maintain their pace and must pause for a moment's rest; but an alternative hypothesis is that while running tiger beetles are unable to process the resulting rapidly changing visual information, and so quickly go blind and stop.
Which of the following, if discovered in experiments using artificially moved prey insects, would support one of the two hypotheses and undermine the other?
27’’ 逻辑链:TB跑的很快因此他能抓住所有不是飞行类的虫子——但是当他抓捕的时候,他要停下来一会再抓——可能是因为他不能保持这样的速度所以要停下来休息一会——另外一个假说是说TB在高速奔跑的时候不能适应迅速变化的视觉信息,所以它就像眼瞎了似得,停了。
Support one/undermine another:
(A) When a prey insect is moved directly toward a beetle that has been chasing it, the beetle immediately turns and runs away without its usual intermittent stopping.
(B) In pursuing a moving insect, the beetles usually respond immediately to changes in the insect's direction, and pause equally frequently whether the chase is up or down an incline.
(C) The beetles maintain a fixed time interval between pauses, although when an insect that had been stationary begins to flee, the beetle increases its speed after its next pause. 这种类型的问题一定要记得控制变量。变量控制:rapidly changing visual information,但是TB还是要停下来休息.
(D) If, when a beetle pauses, it has not gained on the insect it is pursuing, the beetle generally ends its pursuit. 无关
(E) When an obstacle is suddenly introduced just in front of running beetles, the beetles sometimes stop immediately, but they never respond by running around the barrier. 无关
作者: fesche 时间: 2016-5-8 00:58
Human has cognitive faculties, other animals don’t
human can not be happy once they aware this
not happy without involve gratification faculties
Weaken
因:有cognitive faculties
果:没有的话不开心
因果不对,因果颠倒,他因
B
(A) Certain animals—dolphins and chimpanzees, for example—appear to be capable of rational communication.
(B) Many people familiar both with intellectual stimulation and with physical pleasures enjoy the latter more.
(C) Someone who never experienced classical music as a child will usually prefer popular music as an adult.
(D) Many people who are serious athletes consider themselves to be happy.
(E) Many people who are serious athletes love gourmet food.
EW was found in eu and asia
the only place is in narrow strip in asia
因:the oldest one was found in the narrow strip
果: first dominated in this narrow strip
Strengthen
因果,有因有果没有他因,因果没有颠倒
资料表明那里就是最早的
其他地方发现的都没有那里早
D
A) The present day distribution of another wild wheat, einkorn, which was also domesticated early in the development of agriculture, covers a much larger area of southwest Asia.
(B) Modern experiments show that wild emmer wheat can easily be domesticated so as to yield nearly as well as traditionally domestic strains.
(C) At the time when emmer wheat was first cultivated, it was the most nutritious of all the varieties of grain that were then cultivated.
(D) In the region containing the strip where wild emmer wheat has been found, climatic conditions have changed very little since before the development of agriculture.
(E) It is very difficult, without genetic testing, to differentiate the wild form of emmer wheat from a closely related wild wheat that also grows in southwest Asia.
Otter decline
disease is likely
seal lion disease
disease can infect otter
weaken
因:seal and seal lion有病,病可以从他们传到otter
果:otter也是有病
因果之间有gap,因没有果
otter有病死不了
seal他们有病死了,所以killer shark没东西食只可以食otter
A
(A) Killer whales in the North Pacific usually prey on seals and sea lions but will, when this food source is scarce, seek out other prey.
(B) There is no indication that the sea otter population at any North Pacific location declined in the 1980s because of substantial numbers of sea otters migrating to other locations.
(C) Along the Pacific coast of North America in the 1980s, sea otters were absent from many locations where they had been relatively common in former times.
(D) Following the decline in the population of the sea otters, there was an increase in the population of sea urchins, which are sea otters' main food source.
(E) The North Pacific populations of seals and sea lions cover a wider geographic area than does the population of sea otters.
Beverage containers in trash declines, because of recycle program
A was used more widely, but G by greater percentage
关于数量的,凡是有数的都要考虑是否正确
可能厂家不出G,而是推出A的多
大家都把G在家里用做花瓶等
C
(A) Glass bottles are significantly heavier than aluminum cans of comparable size.
(B) Recycled aluminum cans were almost all beverage containers, but a significant fraction of the recycled glass bottles had contained products other than beverages.
(C) Manufacturers replaced many glass bottles, but few aluminum cans, with plastic containers.
(D) The total weight of glass bottles purchased by households increased at a slightly faster rate than the total weight of aluminum cans.
(E) In many areas, glass bottles had to be sorted by color of the glass before being recycled, whereas aluminum cans required no sorting.
When attack has to stop
one: stop for rest
second: go too fast, blind, stop to change visual info
有个移动的,怎么增强一个
在移动不而不stop就不是第二个
B-C
(A) When a prey insect is moved directly toward a beetle that has been chasing it, the beetle immediately turns and runs away without its usual intermittent stopping.
(B) In pursuing a moving insect, the beetles usually respond immediately to changes in the insect's direction, and pause equally frequently whether the chase is up or down an incline. 前半段讲了即使移动没有停止,所以是看到的。第二个移动停止是休息
(C) The beetles maintain a fixed time interval between pauses, although when an insect that had been stationary begins to flee, the beetle increases its speed after its next pause.
(D) If, when a beetle pauses, it has not gained on the insect it is pursuing, the beetle generally ends its pursuit.
(E) When an obstacle is suddenly introduced just in front of running beetles, the beetles sometimes stop immediately, but they never respond by running around the barrier.
作者: hedyail 时间: 2016-8-12 11:36
D6 错2对3,关键在于理解题意
精炼
T 31S
PRE:human has better congitive faculties
CON:human cannot be happier by anything that does not involve gratification of these faculties
WEAKEN
(A) Certain animals—dolphins and chimpanzees,
for example—appear to be capable of
rational communication.- not related to other animals
(B) Many people familiar both with intellectual
stimulation and with physical pleasures
enjoy the latter more. - 没对比
(C) Someone who never experienced classical
music as a child will usually prefer popular
music as an adult. - support
(D) Many people who are serious athletes consider
themselves to be happy. - 没有前后功能的对比
(E) Many people who are serious athletes love
gourmet food. -correct
E c
逻辑链
T2 28s
PRE:traces of cultivated EW are found in Europe and Asia. Wild form of EW can only be found in a strip of southwest asia.
oldest remains of cultivated EW also in the same village in the strip
CON:EW was first domesticated in the strip
SUPPORT
D ACD无关讨论
T3 40s
PRE:sea otters decreased a lot in 1980s. Two main reasons:predation and disease. The diseases that infect other creatures like sea seals can also cause sea otters to get infected
CON
isease is a more likely reason
WEAKEN
A B有点support,CDE无关
T4
BG:1978-1989, beverage containers accounted less percentage of total weight of household garbage in USA. Recycling is the main reason.
PRE:A recycling was more widely practiced than glass recycling.
CON:the weight of glass declined by a greater percentage than A
EXPLAIN
C D有support,E无关,AB有诱惑
T5 42s
PRE:TB are fast runners. When running to an insect, TB may suddenly stop and the nresume their attack.
Hypothese:
1)cannot maintain their pace and need a rest
2)cannot process the changing visual information and go blind to stop
Support 1 hypothese but undermine another one
A 不太确定 c
作者: 宋痂岛__ 时间: 2017-12-8 14:04
B
D
A
C
C
作者: May97 时间: 2018-1-19 09:52
占
作者: echo-LUO 时间: 2018-9-12 19:57
精练:
题目提供两个facts:Human beings have cognitive faculties that are superior to those of other animals;
once humans become aware of these, they cannot be made happy by anything that does not involve gratification of these faculties.
(A) Certain animals—dolphins and chimpanzees,
for example—appear to be capable of
rational communication.
(B) Many people familiar both with intellectual
stimulation and with physical pleasures
enjoy the latter more.
(C) Someone who never experienced classical
music as a child will usually prefer popular
music as an adult.
(D) Many people who are serious athletes consider
themselves to be happy.
(E) Many people who are serious athletes love
gourmet food.
好难,这道题没有看懂
这篇文章的结论是,人们必需要被包含已经了解的认知能力的事物取悦,所以包含两个条件,第一是become aware of,第二个是cognitive faculties。B选项就说明,除了cognitive faculties之外,其它的也可以带来愉悦
逻辑链
21. HELP
人类种植的we存在在欧洲和亚洲。但野生的ew唯一被发现在sa,且因为发现的最早的人类种植的ew也在这里,所以we就是在这里最早被改变成人类种植的东西
(A) The presentday distribution of another wild wheat, einkorn, which was also domesticated early in the development of agriculture, covers a much larger area of southwest Asia.
(B) Modern experiments show that wild emmer wheat can easily be domesticated so as to yield nearly as well as traditionally domestic strains.
(C) At the time when emmer wheat was first cultivated, it was the most nutritious of all the varieties of grain that were then cultivated. 因为有很高的价值,所以人们才会种植?
(D) In the region containing the strip where wild emmer wheat has been found, climatic conditions have changed very little since before the development of agriculture.为啥选...
(E) It is very difficult, without genetic testing, to differentiate the wild form of emmer wheat from a closely related wild wheat that also grows in southwest Asia.
22. HURT
造成so死亡的原因有两种,天敌或者疾病;但一些动物因为疾病死亡,且这些动物和so一样都很容易感染,所以得出结论,so更可能是因为疾病死亡
漏洞:容易感染不代表会感染。这个病毒可能无法在so生活的地方存在、so的食物可能可以解毒etc
(A) Killer whales in the North Pacific usually prey on seals and sea lions but will, when this food source is scarce, seek out other prey.
(B) There is no indication that the sea otter population at any North Pacific location declined in the 1980s because of substantial numbers of sea otters migrating to other locations.
(C) Along the Pacific coast of North America in the 1980s, sea otters were absent from many locations where they had been relatively common in former times.
(D) Following the decline in the population of the sea otters, there was an increase in the population of sea urchins, which are sea otters' main food source.
(E) The North Pacific populations of seals and sea lions cover a wider geographic area than does the population of sea otters.距离远,所以不易传播
23. HELP
1978-1988,饮料罐垃圾越来越少,因为人们开始尝试回收铝罐和玻璃罐。但是尽管铝罐的回收范围更大,玻璃罐重量比铝罐重量减少的更快
原因:可能玻璃罐的单位重量更大啊,基数更小啊,玻璃罐更容易被收集啊etc
(A) Glass bottles are significantly heavier than aluminum cans of comparable size.
(B) Recycled aluminum cans were almost all beverage containers, but a significant fraction of the recycled glass bottles had contained products other than beverages.
(C) Manufacturers replaced many glass bottles, but few aluminum cans, with plastic containers.生产商提供的玻璃瓶少了,自然产生的垃圾也少了
(D) The total weight of glass bottles purchased by households increased at a slightly faster rate than the total weight of aluminum cans.
(E) In many areas, glass bottles had to be sorted by color of the glass before being recycled, whereas aluminum cans required no sorting.
24. HELP&HURT
tb可以快速捕捉猎物,但总是要在发动攻击前停顿。一种假说是tb很难保持高速的节奏,所以需要停顿来休息;一种假说是tb的视力需要适应剧烈的速度变化
(A) When a prey insect is moved directly toward a beetle that has been chasing it, the beetle immediately turns and runs away without its usual intermittent stopping.
(B) In pursuing a moving insect, the beetles usually respond immediately to changes in the insect's direction, and pause equally frequently whether the chase is up or down an incline.说明第一能看到,第二,歇息是固定的,多半是累了
(C) The beetles maintain a fixed time interval between pauses, although when an insect that had been stationary begins to flee, the beetle increases its speed after its next pause.在昆虫不动的时候,也有固定的休息时间,并且之后加速,说明是因为看不到,而不是pace
(D) If, when a beetle pauses, it has not gained on the insect it is pursuing, the beetle generally ends its pursuit.
(E) When an obstacle is suddenly introduced just in front of running beetles, the beetles sometimes stop immediately, but they never respond by running around the barrier.
欢迎光临 ChaseDream (https://forum.chasedream.com/) |
Powered by Discuz! X3.3 |